You are on page 1of 188

MAT 171: Differential Calculus

Student’s Activity Sheet #1

Name: _________________________________________________ Class number: ______


Section: ____________ Schedule: __________________________ Date: _____________

Lesson title: INTRODUCTION TO DIFFERENTIAL Materials: Ballpoint, Notebook, Calculator


CALCULUS Text book: Calculus by Ron Larson,11th Ed
Lesson Objectives: References:
At the end of the lesson, you should be able to: 1..The Calculus 7, by Louis Leithold
1. Define Differential Calculus and related terms 2. Calculus 7th Ed. By James Stewart
2. Classify Functions and their Graphs 3. Differential and Integral Calculus
3. Evaluate Functions by Clyde E. Love and Earl Rainville

Productivity Tip: Most things in life don’t come easy. You have to
make some tough decisions. Make some smart choices.
Being prepared is the only way to know that you are going to win,

A. LESSON PREVIEW/REVIEW

Introduction
In order to pass the course, you shall be oriented on the following:

1. Course Outline
Differential Calculus is an introductory course covering the core concepts of limits, continuity and
differential of functions involving one or more variables. This also includes: the application of
differential calculations in solving problems on optimization, rates of change, related rates,
tangent and normal lines, partial differentiation and transcendental curve tracing

2. Grading System
The Final Grade is computed as follows:
Final Grade = 0.33(1st Periodical Grade) + 0.33(2nd Periodical Grade) + 0.34(3rd Periodical Grade)

1st Periodical Grade = 50% (Class Standing) + 50% (1st Periodic Exam)
Class Standing = 60%(Quiz) + 40%(Class Participation)
Class Participation = Seatwork, Problem Set, Students’ Activity Sheets
2nd Periodical Grade = 50% (Class Standing) + 50% (2nd Periodic Exam)
= 50% CS + 50% Exam
Class Standing = 60% Quiz + 40% Class Participation
Class Participation = Seatwork, Problem Set, Students’ Activity Sheets
3rd Periodical Grade = 50% (Class Standing) + 50% (3rd Periodic Exam)
= 50% CS + 50% Exam
Class Standing = 60% Quiz + 40% Class Participation
Class Participation = Seatwork, Problem Set, Students’ Activity Sheets

Final Exam covers all topics. Passing score is 50.

This document is the property of PHINMA EDUCATION


1
MAT 171: Differential Calculus
Student’s Activity Sheet #1

Name: _________________________________________________ Class number: ______


Section: ____________ Schedule: __________________________ Date: _____________

3. Course Map

3. Due Date on the Submission of Students’ Activity Sheets


You are required to submit the Students’ Activity Sheet on time. The rating is based on
the following:
3.1 Accuracy = 80%
3.2 Timeliness = 20%

4. The first lesson will cover the following:


4.1 Definition of Differential Calculus and related terms
4.2 Classification of Functions and their Graphs
4.3 Evaluation of Functions

This document is the property of PHINMA EDUCATION


2
MAT 171: Differential Calculus
Student’s Activity Sheet #1

Name: _________________________________________________ Class number: ______


Section: ____________ Schedule: __________________________ Date: _____________

Activity 1: What I Know Chart, part 1

What I Know Questions: What I Learned (Activity 4)

1 Do you have any knowledge of


differential calculus?

2 How about functions and their


graphs?

3 Can you evaluate a function?

B. MAIN LESSON

Activity 2: Content Notes

DEFINITIONS

Differential Calculus is the mathematics of the variation of a function with respect to changes in
independent variables; the study of slopes of curves, accelerations, maxima and minima, by
means of derivatives and differentials.

A function is a set of ordered pairs of numbers (𝑥, 𝑦) in which no two distinct ordered pairs have
the same first number. The set of all admissible values of 𝑥 is called the domain of the function,
and the set of all resulting values of 𝑦 is called the range of the function.

The symbols 𝑥 and 𝑦 denote variables. Because the value of 𝑦 is dependent on the choice of
x, 𝑥 denotes the independent variable and 𝑦 denotes the dependent variable.

A function may be denoted as 𝑦 = 𝑓(𝑥) which is read “y equals the function of x” or briefly as “y
equals f of x.” The other symbols used frequently are: 𝑦 = ℎ(𝑥), 𝑦 = 𝑔(𝑥), 𝑦 = 𝜃(𝑥). If we are
given a function 𝑓(𝑥), the value of the function when 𝑥 = 𝑎 is denoted by 𝑓(𝑎).

This document is the property of PHINMA EDUCATION


3
MAT 171: Differential Calculus
Student’s Activity Sheet #1

Name: _________________________________________________ Class number: ______


Section: ____________ Schedule: __________________________ Date: _____________

GRAPHS OF SIX (6) BASIC FUNCTIONS

𝒇(𝒙) = 𝒙 𝒇(𝒙) = 𝒙𝟐

(1) Identity or Linear Function (2) Quadratic Function

𝒇(𝒙) = 𝒙𝟑 𝒇(𝒙) = +√𝒙

(3) Cubic Function (4) Radical or Square Root Function

This document is the property of PHINMA EDUCATION


4
MAT 171: Differential Calculus
Student’s Activity Sheet #1

Name: _________________________________________________ Class number: ______


Section: ____________ Schedule: __________________________ Date: _____________

𝟏
𝒇(𝒙) = |𝒙| 𝒇(𝒙) = 𝒙

(5) Absolute Value Function (6) Rational Function

CLASSIFICATION OF FUNCTIONS

All functions are classified as either algebraic or transcendental. A function is algebraic if the operations
involved in the function are combinations of the six fundamental algebraic operations, namely, addition,
subtraction, multiplication, division, evolution (process of taking the nth roots of a number), and involution
(the operation of raising a number to a power).

The elementary transcendental functions are trigonometric functions (sine, cosine, tangent, cosecant,
secant, cotangent) and inverse trigonometric functions (arcsine, arccosine, arctangent, arc cotangent,
arc secant, arc cosecant) and the exponential and logarithmic functions.

This document is the property of PHINMA EDUCATION


5
MAT 171: Differential Calculus
Student’s Activity Sheet #1

Name: _________________________________________________ Class number: ______


Section: ____________ Schedule: __________________________ Date: _____________

EVALUATION OF A FUNCTION

In evaluating a function, we will input or substitute a value of the independent variable to find the
value of the function. Examples are given for easy comprehension.

Example 1. If 𝑓(𝑥) = 3𝑥 2 − 5,
find: a) 𝑓(1)
b) 𝑓(−3)
c) 𝑓(𝑎)
d) 𝑓(4ℎ − 1)

Solution: Given, 𝑓(𝑥) = 3𝑥 2 − 5 Write the given function

a) 𝑓(1) = 3(1)2 − 5 Substitute 1 for x


=3−5 Simplify
= −2

b) 𝑓(−3) = 3(−3)2 − 5 Substitute (-3) for x


= 3(9) − 5 Simplify
= 27 − 5
= 22

c) 𝑓(𝑎) = 3(𝑎)2 − 5 Substitute a for x


= 3𝑎2 − 5 Simplify

d) 𝑓(4ℎ + 1) = 3(4ℎ + 1)2 − 5 Substitute (4h+1) for x


= 3(16ℎ2 + 8ℎ + 1) − 5 Expand the binomial
= 48ℎ2 + 24ℎ + 3 − 5 Simplify
= 48ℎ2 + 24ℎ − 2

Take note that different values of the


function will be obtained for different
input in the independent variable.

This document is the property of PHINMA EDUCATION


6
MAT 171: Differential Calculus
Student’s Activity Sheet #1

Name: _________________________________________________ Class number: ______


Section: ____________ Schedule: __________________________ Date: _____________

3𝑥+2
Example 2. If 𝑓(𝑥) =
𝑥−1
𝑥+2
and 𝑔(𝑥) =
𝑥−3

Find 𝑓[𝑔(𝑥)]

Solution:

Write
3𝑥 + 2
𝑓(𝑥) =
𝑥−1
𝑥+2
Substitute 𝑔(𝑥) = 𝑥−3 to x of 𝑓(𝑥)

𝑥+2
3 [ 𝑥 − 3] + 2
𝑓[𝑔(𝑥)] =
𝑥+2
[𝑥 − 3 ] − 1

Simplify

3(𝑥 + 2) + 2(𝑥 − 3)
𝑓[𝑔(𝑥)] = 𝑥−3
(𝑥 + 2) − 1(𝑥 − 3)
𝑥−3

3(𝑥 + 2) + 2(𝑥 − 3) 𝑥−3


𝑓[𝑔(𝑥)] = [ ][ ]
𝑥−3 (𝑥 + 2) − (𝑥 − 3)

3(𝑥 + 2) + 2(𝑥 − 3)
𝑓[𝑔(𝑥)] =
(𝑥 + 2) − (𝑥 − 3)

3𝑥 + 6 + 2𝑥 − 6
𝑓[𝑔(𝑥)] =
𝑥+2−𝑥+3
5𝑥
𝑓[𝑔(𝑥)] =
5

𝑓[𝑔(𝑥)] = 𝑥

This document is the property of PHINMA EDUCATION


7
MAT 171: Differential Calculus
Student’s Activity Sheet #1

Name: _________________________________________________ Class number: ______


Section: ____________ Schedule: __________________________ Date: _____________

Example 3. If 𝑔(𝑦) = 𝑐𝑜𝑠 2𝑦 − 2𝑠𝑖𝑛 𝑦


find: a) 𝑔(𝜋),
𝜋
b) 𝑔( ),
2
c) 𝑔(0),
d) [𝑔(𝑥) + 𝑔(−𝑥)]

Solution: From the definition of 𝑔(𝑦), it follows that,

a) 𝑔(𝜋) = 𝑐𝑜𝑠 2(𝜋) − 2𝑠𝑖𝑛(𝜋) Substitute 𝜋 for y


=1−0 Simplify
=1

𝜋 𝜋 𝜋 𝜋
b) 𝑔 ( 2 ) = 𝑐𝑜𝑠 2 ( 2 ) − 2𝑠𝑖𝑛 ( 2 ) Substitute 2
for y
= −1 − 2 Simplify
= −3

c) 𝑔(0) = 𝑐𝑜𝑠 2(0) − 2𝑠𝑖𝑛 (0) Substitute 0 for y


=1−0 Simplify
=1

d) [𝑔(𝑥) + 𝑔(−𝑥)]

since: 𝑔(𝑥) = 𝑐𝑜𝑠 2(𝑥) − 2𝑠𝑖𝑛( 𝑥) ,

and 𝑔(−𝑥) = 𝑐𝑜𝑠 2(−𝑥) − 2sin(−𝑥) Note: s𝑖𝑛 (−𝑥) = −𝑠𝑖𝑛 𝑥


= 𝑐𝑜𝑠 (−2𝑥) − 2sin(−𝑥) 𝑐𝑜𝑠 (−𝑥) = 𝑐𝑜𝑠𝑥
= 𝑐𝑜𝑠 2𝑥 + 2𝑠𝑖𝑛 𝑥

Therefore:
[𝑔(𝑥) + 𝑔(−𝑥)] = [𝑐𝑜𝑠 2𝑥 − 2𝑠𝑖𝑛𝑥] + [𝑐𝑜𝑠 2𝑥 + 2𝑠𝑖𝑛𝑥] Substitution
= 2 𝑐𝑜𝑠2𝑥 Simplify

Take note: When dealing trigonometric functions with


pi (𝜋);

Change the mode of your calculators into “radians


mode”
180
Or convert radians in degrees by multiplying it by, 𝜋

This document is the property of PHINMA EDUCATION


8
MAT 171: Differential Calculus
Student’s Activity Sheet #1

Name: _________________________________________________ Class number: ______


Section: ____________ Schedule: __________________________ Date: _____________

𝑏−𝑏 2
Example 4. If ℎ(𝑏) = , find: a) ℎ(0)
1+𝑏 2
1
b) ℎ(2)
c) ℎ(𝑡𝑎𝑛 𝑥)

0−0
Solution: a) ℎ(0) = Substitute 0 for b
1+0
0
= Simplify
1
=0

1 1 2
1 −( )
2 2 1
b) ℎ( ) = 1 Substitute 2 for b
2 1+( )2
2
1 1

2 4
= 1 Simplify
1+4
1
4
= 5
4
1
= 5

t𝑎𝑛 𝑥− 𝑡𝑎𝑛2 𝑥
𝑐) ℎ(t𝑎𝑛 𝑥) = 1+ 𝑡𝑎𝑛2 𝑥

𝑠𝑖𝑛 𝑥 𝑠𝑖𝑛 𝑥 2
−( )
𝑐𝑜𝑠 𝑥 𝑐𝑜𝑠 𝑥
= 𝑠𝑖𝑛 𝑥 2
1+ ( )
𝑐𝑜𝑠 𝑥

𝑠𝑖𝑛 𝑥 𝑠𝑖𝑛2 𝑥

𝑐𝑜𝑠 𝑥 𝑐𝑜𝑠2 𝑥
= 𝑠𝑖𝑛2 𝑥
1+
𝑐𝑜𝑠2 𝑥

𝑐𝑜𝑠 𝑥 𝑠𝑖𝑛 𝑥 − 𝑠𝑖𝑛2 𝑥


= 𝑐𝑜𝑠 2 𝑥
𝑐𝑜𝑠 𝑥 + 𝑠𝑖𝑛2 𝑥
2

𝑐𝑜𝑠 2 𝑥
𝑐𝑜𝑠 𝑥 𝑠𝑖𝑛 𝑥− 𝑠𝑖𝑛2 𝑥
= but: 𝑐𝑜𝑠 2 𝑥 + 𝑠𝑖𝑛2 𝑥 = 1
𝑐𝑜𝑠2 𝑥+ 𝑠𝑖𝑛2 𝑥

𝑐𝑜𝑠 𝑥 𝑠𝑖𝑛 𝑥− 𝑠𝑖𝑛2 𝑥


=
1

h(tan x) = 𝑠𝑖𝑛 𝑥(𝑐𝑜𝑠 𝑥 − 𝑠𝑖𝑛 𝑥) factor out sin x

This document is the property of PHINMA EDUCATION


9
MAT 171: Differential Calculus
Student’s Activity Sheet #1

Name: _________________________________________________ Class number: ______


Section: ____________ Schedule: __________________________ Date: _____________

Example 5. If 𝑓(𝑣) = 𝑣 3 ,
𝑓(𝑥+∆𝑥) − 𝑓(𝑥)
𝑓𝑖𝑛𝑑: , ∆𝑥 ≠ 0
∆𝑥
𝑓(𝑥 + ∆𝑥) − 𝑓(𝑥)
∆𝑥

Separate evaluation for 𝑓(𝑥 + ∆𝑥) and 𝑓(𝑥)

Considering 𝑓(𝑥 + ∆𝑥)


Substitute: 𝑥 + ∆𝑥 for v
Write: 𝑓(𝑥 + ∆𝑥) = (𝑥 + ∆𝑥)3
Expand 𝑓(𝑥 + ∆𝑥) = (𝑥)3 + 3 (𝑥)2 (∆𝑥) + 3(𝑥) (∆𝑥)2 + (∆𝑥)3
Simplify 𝑓(𝑥 + ∆𝑥) = 𝑥 3 + 3𝑥 2 ∆𝑥 + 3𝑥∆𝑥 2 + ∆𝑥 3

Then, considering: 𝑓(𝑥)


Substitute: 𝑥 for v
Write 𝑓(𝑥) = (𝑥)3
Simplify 𝑓(𝑥) = 𝑥 3

Substitute 𝑓(𝑥 + ∆𝑥) = 𝑥 3 + 3𝑥 2 ∆𝑥 + 3𝑥∆𝑥 2 + ∆𝑥 3 and 𝑓(𝑥) = (𝑥)3

𝑓(𝑥+∆𝑥) − 𝑓(𝑥)
To
∆𝑥

𝑓(𝑥+∆𝑥)−𝑓(𝑥) [𝑥 3 +3𝑥 2 ∆𝑥+3𝑥∆𝑥 2 +∆𝑥 3 ]−[𝑥 3 ]


=
∆𝑥 ∆𝑥

Since we have +x3 and -x3, therefore omit x3

𝑓(𝑥+∆𝑥)−𝑓(𝑥) 3𝑥 2 ∆𝑥+3𝑥∆𝑥 2 +∆𝑥 3


=
∆𝑥 ∆𝑥

Factor ∆x in the numerator

𝑓(𝑥+∆𝑥)−𝑓(𝑥) ∆𝑥(3𝑥 2 +3𝑥∆𝑥 +∆𝑥 2 )


=
∆𝑥 ∆𝑥
Cancel ∆x in the numerator and denominator

𝑓(𝑥+∆𝑥)−𝑓(𝑥)
= 3𝑥 2 + 3𝑥∆𝑥 + ∆𝑥 2
∆𝑥

This document is the property of PHINMA EDUCATION


10
MAT 171: Differential Calculus
Student’s Activity Sheet #1

Name: _________________________________________________ Class number: ______


Section: ____________ Schedule: __________________________ Date: _____________

Activity 3: Skill-building Activities

Directions: Perform the indicated operations in each exercise. Simplify the results.

1. 𝐼𝑓 𝑓(𝑥) = 𝑥 2 − 4𝑥, 𝑓𝑖𝑛𝑑: 𝑎) 𝑓(−5)


𝑏) 𝑓(𝑦 2 + 1)
𝑐) 𝑓(𝑥 + ∆𝑥)
𝑑) 𝑓(𝑥 + 1) − 𝑓(𝑥 − 1)

𝜋
2. 𝐼𝑓 ℎ(𝑦) = 𝑐𝑜𝑠 𝑦 − 𝑠𝑖𝑛 𝑦, 𝑠ℎ𝑜𝑤 𝑡ℎ𝑎𝑡 ℎ ( + 𝑥) = ℎ(𝜋 − 𝑥) = −ℎ(−𝑥)
2

Activity 4: What I Know Chart, part 2

What I Learned

1. ______________________________________________________________________________
______________________________________________________________________________
2. ______________________________________________________________________________
______________________________________________________________________________
3. ______________________________________________________________________________
______________________________________________________________________________

Activity 5: Check for Understanding

Directions: Perform the indicated operations in each exercise. Simplify the results.

1. 𝐼𝑓 𝑓(𝑥) = √𝑥 2 + 4, 𝑓𝑖𝑛𝑑: 𝑎) 𝑓(−2) DO YOUR


𝑏) 𝑓(2) BEST

𝑐) 𝑓(𝑥 + 𝑏𝑥)

2. 𝐼𝑓 𝑓(𝑥) = 𝑐𝑜𝑠 𝑥 𝑓𝑖𝑛𝑑: 𝑎) 𝑓(0)


𝑏) 𝑓(𝜋)
𝑐) 𝑓(𝜋 − 𝑦)

This document is the property of PHINMA EDUCATION


11
MAT 171: Differential Calculus
Student’s Activity Sheet #1

Name: _________________________________________________ Class number: ______


Section: ____________ Schedule: __________________________ Date: _____________

B. LESSON WRAP-UP

Activity 6: Thinking about Learning

A. Work Tracker
You are done with this session! Let’s track your progress. Shade the session number you just
completed.

B. Think about your Learning

1. What motivated you to finish the lesson today?


_____________________________________________________________________________
_____________________________________________________________________________

2. What could you have done better to improve your learning today?
_____________________________________________________________________________
_____________________________________________________________________________

FAQs

1. What are the applications of differential calculus?

The applications of differential calculus are : (1)Calculation of profit and loss with
respect to business using graphs (2) calculation of the rate of change of temperature, (3)
calculation of speed or distance (4) to derive many Physics equations.

In Electrical Engineering, it used to determine the length of cable from one station to
another.

2. What are the pre-requisites of calculus?

You must have prior knowledge in Algebra, Geometry and Trigonometry.

This document is the property of PHINMA EDUCATION


12
MAT 171: Differential Calculus
Student’s Activity Sheet #1

Name: _________________________________________________ Class number: ______


Section: ____________ Schedule: __________________________ Date: _____________

KEY TO CORRECTIONS

Activity 3: 1.a) 45

b) 𝑦 4 − 2𝑦 2 − 3

c) (𝑥 + ∆𝑥)[𝑥 + ∆𝑥 − 4]

d) 4𝑥 − 8

2 −[𝑐𝑜𝑠 𝑥 + 𝑠𝑖𝑛 𝑥 ]

Activity 5.

1. a) 2√2

b) 2√2

c) √𝑥 2 + 2𝑏𝑥 + 𝑏 2 𝑥 2 + 4

2. a) 1

b) −1

c) −c𝑜𝑠 𝑦

This document is the property of PHINMA EDUCATION


13
MAT 171: Differential Calculus
Student’s Activity Sheet #1

Name: _________________________________________________ Class number: ______


Section: ____________ Schedule: __________________________ Date: _____________

This document is the property of PHINMA EDUCATION


14
MAT 171: Differential Calculus
Student’s Activity Sheets #2

Name:__________________________________________________________ Class number: _______


Section: ____________ Schedule:___________________________________ Date: ______________

Lesson Title: LIMITS OF FUNCTIONS Materials: Ballpoint, Notebook, Calculator

Lesson Objectives: Textbook: Calculus by Ron Larson, 11th Ed.


At the end of the lesson, you should be able to: References:
1. Define Limits of Functions and Familiarize the 1. The Calculus 7 by Louis Leithold
Theorems 2. Differential and Integral Calculus
1. 2. Evaluate the Limit of Functions by Clyde E. Love and Earl Rainville
2.

Productivity Tip:
Successful people are not gifted. They just work hard,
then succeed on purpose. If you can’t figure out your
purpose, figure out your passion. For your passion will
lead you right into your purpose. Allow your passion to
become your purpose, and it will one day become your
profession.

A. LESSON PREVIEW/REVIEW

Introduction
GOOD DAY! The lesson is your springboard to calculus.
The following topics shall be accomplished in this module:
1. Define limits of functions and familiarize the theorems
2. Evaluate the limit of functions

1) Activity 1: What I Know Chart, part 1

What I Know Questions: What I Learned (Activity 4)

1 Any idea about limits of


functions?

2 How about the theorems on


limits?

3 Do you know how to evaluate


the limits of functions?

This document is the property of PHINMA EDUCATION


1
MAT 171: Differential Calculus
Student’s Activity Sheets #2

Name:__________________________________________________________ Class number: _______


Section: ____________ Schedule:___________________________________ Date: ______________

B. MAIN LESSON

DEFINITION OF A LIMIT

 Let 𝑓(𝑥) be a function of 𝑥 and let 𝑎 be constant. If there is a number 𝐿 such that, in order to
make the value of 𝑓(𝑥) as close to 𝐿 as may be desired. It is sufficient to choose 𝑥 close enough
to 𝑎, but different from 𝑎. Then we say that the limit of 𝑓(𝑥) as 𝑥 approaches 𝑎, is 𝐿.

 We write, lim 𝑓(𝑥) = 𝐿 , which is read . . . “the limit of 𝑓(𝑥) as 𝑥 approaches 𝑎, is 𝐿.”
𝑥→𝑎

 In finding the limit of a given expression, the first step is to substitute the value of 𝑎.
If the computed value is defined, then that is the limit;
If not, then other property may be tried.

 Let us consider lim (3𝑥 + 1) .


𝑥→2
Here, 𝑓(𝑥) = 3𝑥 + 1 and the constant 𝑎, which 𝑥 will approach, is 2.

 To determine the limit of 𝑓(𝑥) as 𝑥 approaches 2. We will find the value of 𝑓(𝑥) . . .
as 𝑥 approaches 2 from the left, and,
as 𝑥 approaches 2 from the right.

 To do this, let us evaluate 𝑓(𝑥) for several values of 𝑥, near but not equal to 2.
To approach 2 from the left, we use values less than 2,
such as 𝑥 = 1.600, 1.700, 1.950, 1.999….
Whereas to approach 2 from the right, we use values greater than 2,
such as, 𝑥 = 2.400, 2.300, 2.100, 2.009….

The following table gives the values of 𝑓(𝑥) that corresponds to different values of 𝑥.
𝑥 1.600 1.700 1.950 1.999 2 2.009 2.100 2.300 2.400
𝑓(𝑥) 5.800 6.100 6.850 6.997 7.027 7.300 7.900 8.200

 The table shows that when 𝑥 is near 2, whether a little less or a little greater than 2,
𝑓(𝑥) = 3𝑥 + 1 is nearer 7.

 In other words, “3𝑥 + 1 approaches the number 7 as a limit when 𝑥 approaches 2.”
The abbreviated symbolic form for this statement is,
3𝑥 + 1 → 7 𝑎𝑠 𝑥 → 2.

 We may also say that “the limit of 3𝑥 + 1 as 𝑥 approaches 2 is 7.”


In symbols, we write this as,

lim (3𝑥 + 1) = 7
𝑥→2

This document is the property of PHINMA EDUCATION


2
MAT 171: Differential Calculus
Student’s Activity Sheets #2

Name:__________________________________________________________ Class number: _______


Section: ____________ Schedule:___________________________________ Date: ______________

THEOREMS ON LIMITS

We shall need theorems to evaluate the limits of functions. Examples are given for understanding.

Theorem 1: “The limit of a constant is equal to the constant itself”


𝐥𝐢𝐦 𝑪 = 𝑪
𝒙→𝒂

Example: 1.1. 𝐥𝐢𝐦 𝟐 = 2


𝒙→𝟏

𝟏. 𝟐. 𝐥𝐢𝐦 𝟓 = 5
𝒙→𝟎

Theorem 2: “The limit of the variable x, as x approaches a constant a, is equal to a”


𝐥𝐢𝐦 𝒙 = 𝒂
𝒙→𝒂

Example: 2.1. 𝐥𝐢𝐦 𝒙 = 1 Substitute 1 in x


𝒙→𝟏

2.2 𝐥𝐢𝐦 𝑥 2 = (2)2 =4 Substitute 2 in x


𝒙→𝟏

Theorem 3: “The limit of the sum (algebraic sum) of two or more functions is equal to the
sum of their limits”
𝐥𝐢𝐦 [𝒇(𝒙) + 𝒈(𝒙)] = 𝒇(𝒙) + 𝒈(𝒙)
𝒙→𝒂

Example: 3.1 𝐥𝐢𝐦 (𝒙𝟐 + 𝟓) = 𝐥𝐢𝐦 𝒙𝟐 + 𝐥𝐢𝐦 𝟓 Apply theorems 1, 2, 3


𝒙→𝟐 𝒙→𝟐 𝒙→𝟐
2
= 2 +5
= 9

3.2 𝐥𝐢𝐦 (3𝑥 2 + 4𝑥) = 𝐥𝐢𝐦 3𝑥 2 + 𝐥𝐢𝐦 4𝑥 Apply theorems 1, 2, 3


𝒙→𝟏 𝒙→𝟏 𝒙→𝟏
2
= 3(1) + 4(1)
= 3+4
= 7

This document is the property of PHINMA EDUCATION


3
MAT 171: Differential Calculus
Student’s Activity Sheets #2

Name:__________________________________________________________ Class number: _______


Section: ____________ Schedule:___________________________________ Date: ______________

Theorem 4: The limit of the product of two or more functions is equal to the product of their
limits”
𝐥𝐢𝐦 [𝒇(𝒙) ∙ 𝒈(𝒙)] = [ 𝐥𝐢𝐦 𝒇(𝒙) ] [ 𝐥𝐢𝐦 𝒈(𝒙) ]
𝒙→𝒂 𝒙→𝒂 𝒙→𝒂

Example: 4.1 Evaluate: 𝐥𝐢𝐦 (3𝑥 2 )(2𝑥 − 7)


𝒙→𝟒
Solution: 𝐥𝐢𝐦 (3𝑥 2 )(2𝑥 − 7) = [𝐥𝐢𝐦 3𝑥 2 ] [𝐥𝐢𝐦 (2𝑥 − 7) ]
𝒙→𝟒 𝒙→𝟒 𝒙→𝟒
= [3(4)2 ] [2(4) − 7] Substitute 4 in x
= (48)(1) Simplify
= 48

Theorem 5: “The limit of the quotient of two fractions is equal to their limits, provided
that the limit of the denominator is not zero”
𝒇(𝒙) 𝐥𝐢𝐦 𝒇(𝒙)
𝒙→𝒂
𝐥𝐢𝐦 = where: lim g(x)≠ 𝟎
𝒙→𝒂 𝒈(𝒙) 𝐥𝐢𝐦 𝒈(𝒙)
𝒙→𝒂

𝑥 3 −4
Example: 5.1 Evaluate 𝐥𝐢𝐦 (2𝑥−1 )
𝒙→𝟑
𝑥 3 −4 𝐥𝐢𝐦 (𝑥 3 −4)
𝒙→𝟑
Solution: 𝐥𝐢𝐦 ( ) =
𝒙→𝟑 2𝑥−1 𝐥𝐢𝐦 (2𝑥−1)
𝒙→𝟑
(3)3 −4
= Substitute 3 in x
2(3)−1
27−4
= 6−1
Simplify
23
= 5

𝑥 3 −𝑥 2 +𝑥−6
Example: 5.2 Evaluate 𝐥𝐢𝐦 𝑥−2
𝒙→𝟏
𝑥 3 −𝑥 2 +𝑥−6 𝑥 3 −𝑥 2 +𝑥−6
Solution: 𝐥𝐢𝐦
𝑥−2
= 𝑥−2
𝒙→𝟏
13 −12 +1−6
= 1−2
Substitute 1 in x
−5
= −1
Simplify
=5

Note that the limits of functions in the


above examples can be obtained by
straight substitution.

This document is the property of PHINMA EDUCATION


2
MAT 171: Differential Calculus
Student’s Activity Sheets #2

Name:__________________________________________________________ Class number: _______


Section: ____________ Schedule:___________________________________ Date: ______________

𝒙𝟑 −𝒙𝟐 +𝒙−𝟔
Example: 5.3 Evaluate 𝐥𝐢𝐦
𝒙→𝟐 𝒙−𝟐

Solution: Repeat the application of Theorems 1, 2, and 3,

𝑥 3 −𝑥 2 +𝑥−6 8−4+2−6
lim
𝑥−2
= 2−2
𝑥→2
0
= 0
,

The result is in the indeterminate form.


Hence, Theorem 5 cannot be applied at this stage.

In synthetic division of polynomials,


𝑥 = 2 is a zero of the function 𝑥 3 − 𝑥 2 + 𝑥 − 6
yields 𝑥 3 − 𝑥 2 + 𝑥 − 6 = (𝑥 − 2)(𝑥 2 + 𝑥 + 3).

By synthetic division:

1 −1 1 −6 Լ2
2 2 6
____________________
1 1 3 0

Recall that in the definition of a limit 𝑥 → 𝑎,


𝑥 is barred from taking on the value of 𝑎, that is 𝑥 ≠ 𝑎. For 𝑥 ≠ 2, then:

𝑥 3 −𝑥 2 +𝑥−6 (𝑥−2)(𝑥 2 +𝑥+3).


lim
𝑥−2
= (𝑥−2)
𝑥→2
2
= 𝑥 +𝑥+3

Therefore, to evaluate the limit of the given function of this case, we proceed as follows:

𝑥 3 −𝑥 2 +𝑥−6 (𝑥−2)(𝑥 2 +𝑥+3)


lim
𝑥−2
= lim (𝑥−2)
𝑥→2 𝑥→2

= lim ( 𝑥 2 + 𝑥 + 3 )
𝑥→2

= 22 + 2 + 3 Substitute 2 in x
= 9 Simplify

𝟎
Note: The indeterminate form 𝟎 can be solved by L’Hopital’s Rule

This document is the property of PHINMA EDUCATION


3
MAT 171: Differential Calculus
Student’s Activity Sheets #2

Name:__________________________________________________________ Class number: _______


Section: ____________ Schedule:___________________________________ Date: ______________

𝑥 2 +4𝑥−21
Example: 5.4 Evaluate lim
𝑥→3 𝑥 2 −4𝑥+3

Solution: Repeat application of theorems 1, 2, and 3,

𝑥 2 +4𝑥−21 (3)2 +4(3)−21


Lim =
𝑥→3 𝑥 2 −4𝑥+3 (3)2 −4(3)+3

9+12−21 0
= = indeterminate form.
9−12+3 0

Since, 𝑥 2 − 4𝑥 + 3 = 0 Theorem 5 cannot be applied.


The limit may be expressed as,
𝑥 2 +4𝑥−21 (𝑥−3)(𝑥+7)
lim = lim (𝑥−3)(𝑥−1) by factoring
𝑥→3 𝑥 2 −4𝑥+3 𝑥→3
(𝑥+7)
= lim (𝑥−1)
𝑥→3
3+7
=
3−1
10
= 2
=5
𝑥 2 +4𝑥−21
Thus, lim 2 =5
𝑥→3 𝑥 −4𝑥+3

√𝑥−2
Example: 5.5. Evaluate lim
𝑥→4 𝑥−4

√𝑥−2 √4−2
Solution: lim = .
𝑥→4 𝑥−4 4−4

0
= 0
indeterminate form

We rationalize the numerator by multiplying both the


numerator and the denominator by (√𝑥 + 2).

√𝑥−2 √𝑥−2 √𝑥+2


lim = lim ( )
𝑥→4 𝑥−4 𝑥→4 𝑥−4 √𝑥+2
(𝑥−4)
= lim (𝑥−4)(
𝑥→4 √𝑥+2)
1
=
√2+2
1
=
2+2
√𝑥−2 1
Thus, lim =
𝑥→4 𝑥−4 4

This document is the property of PHINMA EDUCATION


4
MAT 171: Differential Calculus
Student’s Activity Sheets #2

Name:__________________________________________________________ Class number: _______


Section: ____________ Schedule:___________________________________ Date: ______________

Two special trigonometric limits (Calculus by Larson, p. 86)

𝑠𝑖𝑛 𝛼
Theorem 6: “ If α is measured in radians, lim 𝛼
=1
𝑎→0

1−𝑐𝑜𝑠 𝑥
Theorem 7: lim
𝑥
= 0
𝑥→0

𝑠𝑖𝑛 3𝜃
Example: 6.1. Evaluate lim
𝜃→0 𝜃

𝑠𝑖𝑛 3𝜃 (3)(𝑠𝑖𝑛 3𝜃)


lim
𝜃
= lim (3)𝜃
𝜃→0 𝜃→0
𝑠𝑖𝑛 3𝜃 s𝑖𝑛 3𝜃
Lim
𝜃
= ( lim 3) ( lim 3𝜃
) We take α = 3𝜃 in theorem 6,
𝜃→0 𝜃→0 𝜃→0
= 3(1)
= 3

Activity 3: Skill-building Activities (with answer key)

Evaluate the limits:

1. lim (𝑥 2 + 2𝑥 − 7)
𝑥→2
2. lim (𝑥 2 + 5𝑥 − 3)
𝑥→3
3. lim (2𝑥 − 1)(𝑥 2 + 5𝑥 − 3)
𝑥→3
2𝑡 2 +1
4. lim 𝑡 2 +3𝑡−5 GOOD
𝑡→0
2𝑥 2 −𝑥−3
5. lim 3𝑥2 +5𝑥+2 LUCK
𝑥→1
𝑦 3 −13𝑦+12
6. lim
𝑦→2 𝑦 3 −14𝑦+15
𝑠𝑖𝑛 𝑎 2
7. lim
𝑎→0 𝑎 2
𝑠𝑖𝑛 𝑘𝑥
8. lim
𝑥→0 𝑥

1) Activity 4: What I Know Chart, part 2

What I Learned
1. _________________________________________________________________________________
_________________________________________________________________________________
2. ________________________________________________________________________________
_________________________________________________________________________________
3. _________________________________________________________________________________
_________________________________________________________________________________

This document is the property of PHINMA EDUCATION


5
MAT 171: Differential Calculus
Student’s Activity Sheets #2

Name:__________________________________________________________ Class number: _______


Section: ____________ Schedule:___________________________________ Date: ______________

2) Activity 5: Check for Understanding


Evaluate the limits:
𝑥 2 −1
1. lim
𝑥→1 𝑥 2 +3𝑥−4

𝑦 3 −𝑦 2 −𝑦−2
2. lim
𝑦→2 3 −5𝑦 2 +5𝑦−6
2𝑦

B. LESSON WRAP-UP

Activity 6: Thinking about Learning

A. Work Tracker

B. Think about your Learning

1. What motivated you to finish the lesson today?


________________________________________________________________________
________________________________________________________________________

2. What could you have done better to improve your learning today?
_________________________________________________________________________
_________________________________________________________________________

FAQs

1. Do all functions have limits?


Some functions do not have any kind of limit as x approaches to infinity. For example, consider
the function f(x) = x sin x. This function does not get close to any particular real number as x
gets large, because we can always use a value of x to make f(x) larger than any number we
choose.

2. What is the importance of limits?


Limits allow us to study a number from afar. That is, we can study the points around it so we can
better understand the given value we want to know. Especially in derivatives, where change in
position is purely relative, the points around a given value are critically important.

This document is the property of PHINMA EDUCATION


6
MAT 171: Differential Calculus
Student’s Activity Sheets #2

Name:__________________________________________________________ Class number: _______


Section: ____________ Schedule:___________________________________ Date: ______________

KEY TO CORRECTIONS

Activity 3:
1. 1
2. 21
3. 105
1
4. −
5

5. 5
14
6.
13

7. 1

8. 𝑘

Activity 5:
2
1.
5

7
2. 9

This document is the property of PHINMA EDUCATION


7
MAT 171: Differential Calculus
Student’s Activity Sheets #2

Name:__________________________________________________________ Class number: _______


Section: ____________ Schedule:___________________________________ Date: ______________

This document is the property of PHINMA EDUCATION


8
MAT 171: Differential Calculus
Student’s Activity Sheets #3

Name: __________________________________________________________ Class number: ________


Section: ____________ Schedule:___________________________________ Date: _______________

Lesson Title: CONTINUITY Materials: Ballpoint, Notebook, Calculator

Lesson Objectives: Textbook: Calculus by Ron Larson, 11th Ed.


At the end of the lesson, you should be able to: References:
1. Enumerate and Illustrate Continuous and 1. The Calculus 7 by Louis Leithold
Discontinuous Function at a Point 2. Differential and Integral Calculus
1. 2. Interpret Graphically the Infinite Limits and by Clyde E. Love and Earl Rainville
2. Points of Discontinuity

Productivity Tip

People aren’t always going to be there for you, that’s why


you need to learn to handle things on your own.

If you want to be strong, learn how to fight alone.

If you want success, then don’t rely on other people to do,


what you can do!

A. LESSON PREVIEW/REVIEW

1) Introduction
GOOD DAY! The following topics shall be accomplished in this lesson:
1. Enumerate continuous and discontinuous functions
2. Illustrate continuity and discontinuity at a point
3. Graph the infinite limits
4. Interpret the points of discontinuity

2) Activity 1: What I Know Chart, part 1

What I Know Questions: What I Learned (Activity 4)


1 Are you familiar with continuous
and discontinuous functions?

2 Do you know how to graph


them?

3 Can you interpret their graphs?

This document is the property of PHINMA EDUCATION


1
MAT 171: Differential Calculus
Student’s Activity Sheets #3

Name: __________________________________________________________ Class number: ________


Section: ____________ Schedule:___________________________________ Date: _______________

B.MAIN LESSON

CONTINUITY

 A function f(x) is said to be continuous at 𝒙 = 𝒂 if the following conditions are all satisfied:

1. f(a) exists
2. lim 𝑓(𝑥) exists
𝑥→𝑎

3. lim 𝑓(𝑥) = 𝑓(𝑎)


𝑥→𝑎

 If any of the conditions 1, 2, 3 is not satisfied, then, the function is said to be discontinuous at
𝒙 = 𝒂.

 When f(a) exists and lim 𝑓(𝑥) = 𝑓(𝑎), the function f(x) is said to have a right-hand
𝑥→𝑎 +
continuity at 𝒙 = 𝒂.

 On the other hand, if f(a) exist and lim− 𝑓(𝑥) = 𝑓(𝑎), then f(x) is said to have a left-hand
𝑥→𝑎
continuity at 𝒙 = 𝒂.

Continuity, like limit of a function implies both right-hand and left-hand continuity.

 A function f(x) is said to be continuous in an interval if it is continuous for every value of x in


the interval. The graph of the function is “unbroken” over the interval. That is, the graph of f(x)
can be drawn without lifting the pencil from the paper. To illustrate:

Example 1. 𝒇(𝒙) = 𝒙𝟐 is continuous at x=2 because


in applying the definition of a continuous function on
𝑓(𝑥) = 𝑥 2 with x=2, we have,

1) 𝑓(2) = 22 = 4
2) lim 𝑓(𝑥) = lim 𝑥 2 = 4
𝑥→2 𝑥→2

3) lim 𝑓(𝑥) = 𝑓(2)


𝑥→2

𝒇(𝒙) = 𝒙𝟐
Thus, the function is continuous at x=2. In fact, it is
Continuous for all finite values of x. Refer to the graph
of the function.

This document is the property of PHINMA EDUCATION


2
MAT 171: Differential Calculus
Student’s Activity Sheets #3

Name: __________________________________________________________ Class number: ________


Section: ____________ Schedule:___________________________________ Date: _______________
𝟏
Example 2. The function 𝒇(𝒙) = is continuous at x=3.
𝒙
Using the definition of a continuous function, we have,
1
1) 𝑓(𝑥) = 3
1 1 𝟏
2) lim 𝑓(3) = 3
𝒇(𝒙) = 𝒙
𝑥→3
1
3) lim 𝑓(3) = 𝑓(3)
𝑥→3

It is however discontinuous at x=0 since


1) f(0) is not defined (has zero denominator)
2) lim 𝑓(𝑥) does not exist (equals ∞)
𝑥→0

The graph of this function, shown in the figure contains a “break” at x=0. The function is continuous
everywhere except at x=0 where it is said to have an infinite discontinuity.

With regards to rational algebraic functions, the question of continuity is completely covered by the
following theorems:

Theorem 1. A polynomial is continuous for all values of x

Example 3. The graph of the polynomial function, 𝒇(𝒙) = 𝒙𝟑 − 𝟔𝒙𝟐 + 𝟗𝒙 + 𝟏 is shown below. The
function is continuous for all values of x.

𝒇(𝒙) = 𝒙𝟑 − 𝟔𝒙𝟐 + 𝟗𝒙 + 𝟏

This document is the property of PHINMA EDUCATION


3
MAT 171: Differential Calculus
Student’s Activity Sheets #3

Name: __________________________________________________________ Class number: ________


Section: ____________ Schedule:___________________________________ Date: _______________

Theorem 2. A rational algebraic function is continuous except for


those values of x for which the denominator becomes zero
𝟏
Example. The rational function 𝒇(𝒙) = 𝒙−𝟐
is discontinuous at x=2.

𝟏
𝒇(𝒙) = 𝒙−𝟐

Activity 3: Skill-building Activities (with answer key)


Find the point of discontinuity of the following functions:
3𝑥
1. 𝑓(𝑥) = DO YOUR
𝑥−5
BEST
𝑥 2 +3
2. 𝑓(𝑥) = 𝑥 2 −16

6𝑥
3. 𝑓(𝑥) = 𝑥 2 −9

3𝑥+2
4. 𝑓(𝑥) = 𝑥 2 −6𝑥+9

Activity 4: What I Know Chart, part 2

What I Learned
1. ______________________________________________________________________________
______________________________________________________________________________

2. ______________________________________________________________________________
______________________________________________________________________________

3. ______________________________________________________________________________
______________________________________________________________________________

This document is the property of PHINMA EDUCATION


4
MAT 171: Differential Calculus
Student’s Activity Sheets #3

Name: __________________________________________________________ Class number: ________


Section: ____________ Schedule:___________________________________ Date: _______________

2) Activity 5: Check for Understanding

Find the point of discontinuity of the function:


3𝑥+2
1. 𝑓(𝑥) =
𝑥 2 −8𝑥+15

C. LESSON WRAP-UP
1) Activity 6: Thinking about Learning

A. Work Tracker
You are done with this session! Let’s track your progress. Shade the session number you just
completed.

A. Think about your Learning

1. What motivated you to finish the lesson today?


________________________________________________________________________
________________________________________________________________________

2. What could you have done better to improve your learning today?
_________________________________________________________________________
_________________________________________________________________________

FAQs
1. How do you know when a function is continuous?
A function is continuous at x=a if and only if it meets three conditions:
1) The function is defined at x=a.
2) The limit of the function as x approaches a exists.
3) The limit of the function as x approaches a is equal to the function value of f(a).

2. What is the concept of continuity?


Continuity of a function is sometimes expressed by saying that if the x-values are close
together, then the y-values of the function will also be close. A function varies with no abrupt
breaks or jumps.

This document is the property of PHINMA EDUCATION


5
MAT 171: Differential Calculus
Student’s Activity Sheets #3

Name: __________________________________________________________ Class number: ________


Section: ____________ Schedule:___________________________________ Date: _______________

KEY TO CORRECTIONS

Activity 3: 1. 𝒙 = 𝟓
2. 𝒙 = 𝟒, 𝒙 = −𝟒
3. 𝒙 = 𝟑, 𝒙 = −𝟑
4. 𝒙 = −𝟑

Activity 5: 1. 𝒙 = 𝟓, 𝒙 = 𝟑

This document is the property of PHINMA EDUCATION


6
Mat 171: Differential Calculus
Student’s Activity Sheets # 4

Name: _______________________________________________________ Class number: ________


Section: ____________ Schedule: _________________________________ Date: _______________

Lesson Title: DERIVATIVE OF ALGEBRAIC Materials: Ballpoint, Notebook,


FUNCTIONS using “Three-Step Rule” Calculator

Lesson Objectives: Textbook: Calculus by Ron Larson


At the end of the lesson, you should be able to: 11th Ed.
1. Define and illustrate increment of variable References:
1. 2. Find the derivative of a function using the 1. The Calculus 7 by Louis Leithold
2. “3-step rule” 2. Differential and Integral Calculus
3. 3. Relate the concept of slope and rate of change to by Clyde E. Love and Earl Rainville
4. the derivative using the “3-step rule”

Productivity Tip:
It’s a slow progress, but quitting won’t make it any faster. A
little progress each day adds up to big results. No matter
how many mistakes you make or how slow your progress is,
you are still ahead of everyone who isn’t trying.

A. LESSON PREVIEW/REVIEW

1) Introduction
Welcome! The following topics skills should be accomplished
in this lesson:
1. Defining and illustrating the increment of variable
2. Finding the derivative of a function using the “3-step rule”
3. Relating the slope and rate of change to the derivative using
the “3-step rule”

2) Activity 1: What I Know Chart, part 1

What I Know Questions: What I Learned (Activity 4)


1. Will you define and illustrate
the increment of a variable?

2. Do you know the “3-step


rule”? can you use this rule in
finding the derivative of a
function?

3 How about relating the slope


and rate of change to the
derivative?

This document is the property of PHINMA EDUCATION


1
Mat 171: Differential Calculus
Student’s Activity Sheets # 4

Name: _______________________________________________________ Class number: ________


Section: ____________ Schedule: _________________________________ Date: _______________

B.MAIN LESSON

The process of finding the derivative of a function (algebraic, trigonometric, exponential or


logarithmic) is called differentiation and the branch of calculus dealing with this process is
Differential Calculus.

INCREMENTS

An increment of x, denoted by the symbol ∆𝑥, read as “delta x”, is the change in x as it
increases or decreases from one value x=x0 to another value x=x1, that is ∆𝑥 = 𝑥1 − 𝑥0 .
Similarly, ∆𝑦 denotes an increment of y.

Suppose 𝑦 = 𝑓(𝑥). A change ∆𝑥 in x produces a corresponding ∆𝑦 in y. That is,

𝑦 + ∆𝑦 = 𝑓(𝑥 + ∆𝑥)
∆𝑦 = 𝑓(𝑥 + ∆𝑥) − 𝑦f\

But since 𝑦 = 𝑓(𝑥), then we have the relation

∆𝑦 = 𝑓(𝑥 + ∆𝑥) − 𝑓(𝑥)

This notation is graphically illustrated in the figure below.

Example 1. If 𝑦 = 𝑥 2, then 𝑦 + ∆𝑦 = (𝑥 + ∆𝑥)2


and ∆𝑦 = (𝑥 + ∆𝑥)2 − 𝑥 2 .

Example 2. If 𝐴 = 𝜋𝑟 2 , then 𝐴 + ∆𝐴 = 𝜋(𝑟 + 𝜋𝑟)2


and ∆𝐴 = 𝜋(𝑟 + 𝜋𝑟)2 − 𝜋𝑟 2 .

This document is the property of PHINMA EDUCATION


2
Mat 171: Differential Calculus
Student’s Activity Sheets # 4

Name: _______________________________________________________ Class number: ________


Section: ____________ Schedule: _________________________________ Date: _______________

THE DERIVATIVE OF FUNCTION

 Let the function f given by 𝑦 = 𝑓(𝑥) be a continuous function of x in some interval containing x.
Recall that in the preceding section, we said that a change ∆𝑥 in the independent variable x
produces a corresponding change ∆𝑦 in the independent variable y; thus,
𝑦 + ∆𝑦 = 𝑓(𝑥 + ∆𝑥)
or ∆𝑦 = 𝑓(𝑥 + ∆𝑥) − 𝑓(𝑥)

if we divide both members by ∆𝑥, we have,


∆𝒚 𝒇(𝒙 + ∆𝒙) − 𝒇(𝒙)
=
∆𝒙 ∆𝒙

 Note that this ratio is the difference of the function values divide by the difference of the x-
values. For this reason, it is often referred to as the difference quotient. It also denotes the
∆𝑦
average rate of change of 𝑓(𝑥) between x and (x +∆𝑥). That is, ∆𝑥
is a measure of the rate at

which y is the change with respect to x for the interval ∆𝑥.

 Suppose we regard x as fixed and then let ∆𝑥 vary and approach zero. Then we define the
∆𝒚 𝒇(𝒙+∆𝒙)−𝒇(𝒙)
expression, 𝐥𝐢𝐦 𝑜𝑟, 𝐥𝐢𝐦 as the instantaneous rate of change of 𝑓(𝑥) at x or
∆𝒙→𝟎 ∆𝒙 ∆𝒙→𝟎 ∆𝒙

simply the rate of change of 𝑓(𝑥) at x.

FUNDAMENTAL DEFINITION
The derivative of y with respect to x is the limit of
∆𝒚
the ratio ∆𝒙 when ∆𝒙 approaches zero.

𝑑𝑦
The derivative is designated by the symbol 𝑑𝑥
.

𝒅𝒚 ∆𝒚 𝒇(𝒙+∆𝒙)−𝒇(𝒙)
= 𝐥𝐢𝐦 = 𝐥𝐢𝐦
𝒅𝒙 ∆𝒙→𝟎 ∆𝒙 ∆𝒙→𝟎 ∆𝒙

𝑑𝑦 𝑑(𝑦) 𝑑
Other symbols for derivative for 𝑑𝑥 are 𝑦 ′ , 𝑓 ′ (𝑥), 𝑑𝑥
, 𝑑𝑥
[𝑓(𝑥)].

This document is the property of PHINMA EDUCATION


3
Mat 171: Differential Calculus
Student’s Activity Sheets # 4

Name: _______________________________________________________ Class number: ________


Section: ____________ Schedule: _________________________________ Date: _______________

“THE “ 3-STEP RULE”


In finding the derivative of a function 𝑦 = 𝑓(𝑥). Based on the definition of the derivative,
𝒅𝒚 ∆𝒚 𝒇(𝒙+∆𝒙)−𝒇(𝒙)
𝒅𝒙
= 𝐥𝐢𝐦 = 𝐥𝐢𝐦 we can write the steps in solving the derivative.
∆𝒙→𝟎 ∆𝒙 ∆𝒙→𝟎 ∆𝒙

These are as follows:

Step 1. Write down the expression for 𝑓(𝑥 + ∆𝑥) − 𝑓(𝑥) ,


and simplify.
𝑓(𝑥+∆𝑥)−𝑓(𝑥)
Step 2. Divide the result in Step 1 by ∆𝑥, that is, ∆𝑥
,

again simplify.
Step 3. Find the limit of the result in Step 2 as ∆𝑥 approaches zero.
The obtained limit is the derivative.
𝑑𝑦 ∆𝑦 𝑓(𝑥+∆𝑥)−𝑓(𝑥)
𝑑𝑥
= lim = lim
∆𝑥→0 ∆𝑥 ∆𝑥→0 ∆𝑥

Example 1. Find the derivative of 𝑓(𝑥) = 𝑥 2 − 5𝑥 using the “three-step rule”.

Solution:
Step 1. Write down 𝑓(𝑥 + ∆𝑥) − 𝑓(𝑥)

Note, 𝑓(𝑥 + ∆𝑥) = (𝑥 + ∆𝑥)2 − 5(𝑥 + ∆𝑥 ) is obtained by


Replacing each x in the original equation by (𝑥 + ∆𝑥).

Therefore,
𝒇(𝒙 + ∆𝒙) − 𝒇(𝒙) = [(𝒙 + ∆𝒙)𝟐 − 𝟓(𝒙 + ∆𝒙)] − (𝒙𝟐 − 𝟓𝒙)
= 𝒙𝟐 + 𝟐𝒙∆𝒙 + (∆𝒙)𝟐 − 𝟓𝒙 − 𝟓∆𝒙 − 𝒙𝟐 + 𝟓𝒙 Simplify
= 𝟐𝒙∆𝒙 + (∆𝒙)𝟐 − 𝟓∆𝒙
= ∆𝒙(𝟐𝒙 + ∆𝒙 − 𝟓) factor out by ∆𝑥

Step 2. Divide the last equation by ∆𝑥 and simplify

𝒇(𝒙 + ∆𝒙) − 𝒇(𝒙) ∆𝒙(𝟐𝒙 + ∆𝐱 − 𝟓)


=
∆𝒙 ∆𝒙
𝒇(𝒙 + ∆𝒙) − 𝒇(𝒙)
= 𝟐𝒙 + ∆𝐱 − 𝟓
∆𝒙

This document is the property of PHINMA EDUCATION


4
Mat 171: Differential Calculus
Student’s Activity Sheets # 4

Name: _______________________________________________________ Class number: ________


Section: ____________ Schedule: _________________________________ Date: _______________

Step 3. Evaluate the limit as ∆𝑥 → 0


𝒇(𝒙 + ∆𝒙) − 𝒇(𝒙)
𝐥𝐢𝐦 = 𝐥𝐢𝐦 (𝟐𝒙 + ∆𝒙 − 𝟓)
∆𝒙→𝟎 ∆𝒙 ∆𝒙→𝟎

= 𝟐𝒙 + 𝟎 − 𝟓
= 𝟐𝒙 − 𝟓

𝑑𝑦 ∆𝑦 𝑓(𝑥+∆𝑥)−𝑓(𝑥)
By definition: 𝑑𝑥
= lim = lim . Therefore:
∆𝑥→0 ∆𝑥 ∆𝑥→0 ∆𝑥

𝒅𝒚
= 𝟐𝒙 − 𝟓
𝒅𝒙
𝟏
Example 2. Differentiate the function 𝒚 = 𝒙−𝟑 using the “three-step rule”.

1
Solution: Given: 𝑦 = 𝑥−3

1 1
Step 1. 𝑓(𝑥 + ∆𝑥) − 𝑓(𝑥) = (𝑥+∆𝑥)−3
− 𝑥−3

(𝑥−3)−[(𝑥+∆𝑥)−3]
= (𝑥+∆𝑥−3)(𝑥−3)

𝑥−3−𝑥−∆𝑥+3
= (𝑥+∆𝑥−3)(𝑥−3)

−∆𝒙
= (𝒙+∆𝒙−𝟑)(𝒙−𝟑)

𝑓(𝑥+∆𝑥)−𝑓(𝑥) −∆𝑥
Step 2. ∆𝑥
= ∆𝑥(𝑥+∆𝑥−3)(𝑥−3)

−𝟏
= (𝒙+∆𝒙−𝟑)(𝒙−𝟑)

𝑓(𝑥+∆𝑥)−𝑓(𝑥) −1
Step 3. lim ∆𝑥
= lim (𝑥+∆𝑥−3)(𝑥−3)
∆𝑥→0 𝑥→0

−1
=
(𝑥+0−3)(𝑥−3)

−1
= (𝑥−3)(𝑥−3)

−𝟏
= (𝒙−𝟑)𝟐

𝒅𝒚 −𝟏
Therefore, 𝒅𝒙
= (𝒙−𝟑)𝟐

This document is the property of PHINMA EDUCATION


5
Mat 171: Differential Calculus
Student’s Activity Sheets # 4

Name: _______________________________________________________ Class number: ________


Section: ____________ Schedule: _________________________________ Date: _______________
𝑑𝑦
Example 3. If 𝑦 = √𝑥, find 𝑑𝑥
using the “three-step rule”.

Solution: Given: 𝑦 = √𝑥
Step 1: 𝑓(𝑥 + ∆𝑥) − 𝑓(𝑥) = √𝑥 + ∆𝑥 − √𝑥

√𝑥+∆𝑥+√𝑥
= [√𝑥 + ∆𝑥 − √𝑥] [ ] rationalize the numerator
√𝑥+∆𝑥+√𝑥

𝑥+∆𝑥−𝑥
= simplify
√𝑥+∆𝑥+√𝑥

∆𝒙
=
√𝒙+∆𝒙+√𝒙

𝑓(𝑥+∆𝑥)−𝑓(𝑥) ∆𝑥
Step 2. = divide by ∆𝑥
∆𝑥 ∆𝑥(√𝑥+∆𝑥+√𝑥)
𝟏
=
√𝒙+∆𝒙+√𝒙

𝑓(𝑥+∆𝑥)−𝑓(𝑥) 1
Step 3. lim ∆𝑥
= lim
∆𝑥→0 ∆𝑥→0 √𝑥+∆𝑥+√𝑥
1
=
√𝑥+√𝑥
𝟏
=𝟐
√𝒙
𝒅𝒚 𝟏
Therefore: 𝒅𝒙
= 𝟐√𝒙

Example 4. Differentiate the function 𝑦 = sin 𝑥

Solution: Given: 𝑦 = sin 𝑥


Step 1: 𝑓(𝑥 + ∆𝑥) − 𝑓(𝑥) = sin(𝑥 + ∆𝑥) − sin 𝑥
From the sum of two angles, = sin 𝑥 cos ∆𝑥 + cos 𝑥 sin ∆𝑥 − sin 𝑥
sin(𝐴 + 𝐵) = 𝑆𝑖𝑛 𝐴 cos 𝐵 + cos 𝐴 sin 𝐵
= cos 𝑥 sin ∆𝑥 − sin 𝑥 + sin 𝑥 cos ∆𝑥
= cos 𝑥 sin ∆𝑥 − sin 𝑥(1 − cos ∆𝑥)
1
From trigonometry: 𝑠𝑖𝑛2 𝐴 = 2 (1 − cos 2𝐴),
∆𝑥 1 ∆𝑥
hence, 𝑠𝑖𝑛2 2
= 2 (1 − cos 2 2 )
∆𝑥 1
𝑠𝑖𝑛2 2 = 2 (1 − cos ∆𝑥)
∆𝑥
2 𝑠𝑖𝑛2 2 = (1 − cos ∆𝑥)
∆𝑥
(1 − cos ∆𝑥) = 2 𝑠𝑖𝑛2 2

This document is the property of PHINMA EDUCATION


6
Mat 171: Differential Calculus
Student’s Activity Sheets # 4

Name: _______________________________________________________ Class number: ________


Section: ____________ Schedule: _________________________________ Date: _______________
∆𝑥
= cos 𝑥 sin ∆𝑥 − sin 𝑥 (2 𝑠𝑖𝑛2 2
)
∆𝑥
= cos 𝑥 sin ∆𝑥 − 2 sin 𝑥 ( 𝑠𝑖𝑛2 2 )

∆𝑥
𝑓(𝑥+∆𝑥)−𝑓(𝑥) cos 𝑥 sin ∆𝑥−2 sin 𝑥 ( 𝑠𝑖𝑛2 )
2
Step 2: =
∆𝑥 ∆𝑥

∆𝑥 ∆𝑥
𝑓(𝑥+∆𝑥)−𝑓(𝑥) cos 𝑥 sin ∆𝑥−2 sin 𝑥 [(𝑠𝑖𝑛 )(𝑠𝑖𝑛 )]
2 2
=
∆𝑥 ∆𝑥

∆𝑥 ∆𝑥
𝑓(𝑥+∆𝑥)−𝑓(𝑥) cos 𝑥 sin ∆𝑥 2 sin 𝑥(sin )(sin )
2 2
∆𝑥
= ∆𝑥
− ∆𝑥

∆𝑥 ∆𝑥
𝑓(𝑥+∆𝑥)−𝑓(𝑥) cos 𝑥 sin ∆𝑥 sin 𝑥(sin )(sin )
2 2
∆𝑥
= ∆𝑥
− ∆𝑥
2

∆𝑥
𝑓(𝑥+∆𝑥)−𝑓(𝑥) sin ∆𝑥 ∆𝑥 sin 2
= cos 𝑥 − sin 𝑥(sin ) ∆𝑥
∆𝑥 ∆𝑥 2
2

sin 𝛼
From Theorem: lim =1
𝛼→0 𝛼

sin ∆𝑥
Now lim =1
∆𝑥→0 ∆𝑥

1
𝑠𝑖𝑛 ∆𝑥
2
lim ∆𝑥 =1
∆𝑥→0
2

𝑓(𝑥+∆𝑥)−𝑓(𝑥) ∆𝑥
∆𝑥
= (cos 𝑥)(1) − sin 𝑥(sin 2
) (1)

𝑓(𝑥+∆𝑥)−𝑓(𝑥) 𝑑𝑦 0
Step 3: lim ∆𝑥
= 𝑑𝑥
= cos x −(sin 𝑥)(sin 2)
∆𝑥→0

𝑑𝑦
𝑑𝑥
= cos x −(sin 𝑥)(0)

𝒅𝒚
Therefore: 𝒅𝒙
= 𝐜𝐨𝐬 𝒙

This document is the property of PHINMA EDUCATION


7
Mat 171: Differential Calculus
Student’s Activity Sheets # 4

Name: _______________________________________________________ Class number: ________


Section: ____________ Schedule: _________________________________ Date: _______________

𝒅𝒚
GEOMETRIC SIGNIFICANCE OF 𝒅𝒙
.

 Consider the graph of 𝑦 = 𝑓(𝑥) shown in the figure. Let P (x, y) and Q (𝑥 + ∆𝑥, 𝑦 + ∆𝑦) be
any two points on this curve.

 Line S which intersects the curve of P and Q and having inclination 𝛼 is called the secant
line of the curve. Note that the slope S is,

∆𝒚 𝒇(𝒙+∆𝒙)−𝒇(𝒙)
𝒎 = 𝐭𝐚𝐧 𝜶 = ∆𝒙
= ∆𝒙

 The line T passing through point P and having inclination 𝜃 is the tangent line to the curve
at P. Let ∆𝑥 → 0, then 𝛼 → 𝜃 or 𝑄 → 𝑃 (P remains fixed) along the curve 𝑦 = 𝑓(𝑥).

 We note that the secant line S approaches the tangent line T as its limiting position. That is,
the slope of S approaches the slope of T or symbolically, tan 𝛼 → tan 𝜃. Hence,

∆𝒚 𝒇(𝒙+∆𝒙)−𝒇(𝒙)
𝐥𝐢𝐦 = 𝐥𝐢𝐦 = slope of T at P.
∆𝒙→𝟎 ∆𝒙 ∆𝒙→𝟎 ∆𝒙

 Thus we see that the derivative of 𝑦 = 𝑓(𝑥) at a point P on the curve is equal to the slope of
𝒅𝒚
the tangent line at P. This is the geometric significance of the derivative .
𝒅𝒙

This document is the property of PHINMA EDUCATION


8
Mat 171: Differential Calculus
Student’s Activity Sheets # 4

Name: _______________________________________________________ Class number: ________


Section: ____________ Schedule: _________________________________ Date: _______________

DERIVATIVE INTERPRETED AS SLOPE

“The derivative of a function is identical


with the slope of the graph of the function”.

Example: Find the slope of the curve 𝑦 = 𝑥 2 at the point (−1, 1)

Solution: Given: 𝑦 = 𝑥 2

𝑓(𝑥 + ∆𝑥) − 𝑓(𝑥) = (𝑥 + ∆𝑥)2 − 𝑥 2


= 𝑥 2 + 2𝑥∆𝑥 + (∆𝑥)2 − 𝑥 2
= 2𝑥∆𝑥 + (∆𝑥)2
= ∆𝒙(𝟐𝒙 + ∆𝒙)

𝑓(𝑥+∆𝑥)−𝑓(𝑥) ∆𝑥(2𝑥+∆𝑥)
=
∆𝑥 ∆𝑥
𝑓(𝑥+∆𝑥)−𝑓(𝑥)
∆𝑥
= 𝟐𝒙 + ∆𝒙

𝑓(𝑥+∆𝑥)−𝑓(𝑥)
lim ∆𝑥
= lim 2𝑥 + ∆𝑥
∆𝑥→0 ∆𝑥→0

= 𝟐𝒙
𝑑𝑦 ∆𝑦
= lim = 𝟐𝒙
𝑑𝑥 ∆𝑥→0 ∆𝑥
𝑑𝑦
𝑑𝑥
= 𝟐𝒙

hence, the slope of the tangent to the curve at (−1, 1) is,

𝒅𝒚
𝒅𝒙
= 2(−1) = −𝟐 as indicated in the graph.

Note that the slope of each tangent line is,


i) inclined upward to the left is always negative (−)
ii) inclines upward to the right is always positive ( + )

The two sets of lines are divided at the vertex, because the slope of the horizontal line at this
point is zero (0).

This document is the property of PHINMA EDUCATION


9
Mat 171: Differential Calculus
Student’s Activity Sheets # 4

Name: _______________________________________________________ Class number: ________


Section: ____________ Schedule: _________________________________ Date: _______________

RATE OF CHANGE

∆𝑦
 The ratio is called the average rate of change over the interval ∆𝒙.
∆𝑥

 This ratio approach a limiting value as ∆𝑥 approaches zero, called the rate of change in y
corresponding to the given value of x, that is,

𝑑𝑦 ∆𝑦
= lim = 𝑡ℎ𝑒 𝑟𝑎𝑡𝑒 𝑜𝑓 𝑐ℎ𝑎𝑛𝑔𝑒 𝑜𝑓 𝑦 𝑤𝑖𝑡ℎ 𝑟𝑒𝑠𝑝𝑒𝑐𝑡 𝑡𝑜 𝑥
𝑑𝑥 ∆𝑥→0 ∆𝑥

 Thus, the term derivative, rate of change and slope of the graph, may be used interchangeably.
Note that the slope of a curve is,
i) positive (𝑦 ′ > 0), the curve is increasing, or f(x) increases as x increases,
as in AB;
ii) negative (𝑦 < 0), the curve is decreasing, or f(x) decreases as x increases,
as in BC.
Also, the function f(x)
i) increases, when the rate of change is positive;
ii) decreases, when the rate of change is negative.

Example 1: Find the rate of change of the volume V with respect to the radius r of the base of a
right circular cylinder of height 10 cm.

The formula for the volume of a right circular cylinder is


𝑉 = 𝜋𝑟 2 ℎ.
Then with ℎ = 10, we have,
𝑉 = 10𝜋𝑟 2
Applying differentiation,
𝑉 = 𝑓(𝑟)
𝑓(𝑟 + ∆𝑟) − 𝑓(𝑟) = 10𝜋[𝑟 + ∆𝑟]2 − 10𝜋𝑟 2
= 10𝜋 (𝑟 2 + 2𝑟∆𝑟 + (∆𝑟)2 − 10𝜋𝑟 2
= 10𝜋𝑟 2 + 20 𝜋𝑟∆𝑟 + 10𝜋(∆𝑟)2 − 10𝜋𝑟 2
= ∆𝑟[20 𝜋𝑟 + 10𝜋(∆𝑟)]
𝑓(𝑟+∆𝑟)−𝑓(𝑟) ∆𝑟[20𝜋𝑟+10𝜋(∆𝑟)]
=
∆𝑟 ∆𝑟

lim 𝑓(𝑟+∆𝑟∆𝑟)−𝑓(𝑟) = lim [20 𝜋𝑟 + 10𝜋(∆𝑟)]


∆𝑟→0 ∆𝑟→0
𝒅𝑽
𝒅𝒓
= 𝟐𝟎 𝝅𝒓

This document is the property of PHINMA EDUCATION


10
Mat 171: Differential Calculus
Student’s Activity Sheets # 4

Name: _______________________________________________________ Class number: ________


Section: ____________ Schedule: _________________________________ Date: _______________

Activity 3: Skill-building Activities (with answer key)

A. Find the derivative in each using the “three-step rule”.

1. 𝑦 = 𝑥 2 − 4𝑥 + 5

2. 𝑦 = 𝑥 3 + 2𝑥

3. Differentiate the function 𝑦 = cos 𝑥

B. Find the slope of the curve at the given point.


1. 𝑦 = 3 + 2𝑥 − 𝑥 2 , 𝑎𝑡 (1, 4)

2. Find how fast is the area of the circle increases when the radius increases.

1) Activity 4: What I Know Chart, part 2

What I Learned
1. _________________________________________________________________________________
_________________________________________________________________________________
2. _________________________________________________________________________________
_________________________________________________________________________________
3. _________________________________________________________________________________
_________________________________________________________________________________

2) Activity 5: Check for Understanding

A. Find the derivative in each using the “three-step rule”.


5
1. 𝑦=
2𝑥+3

2. 𝑦 = √𝑥 + 5

B. Find the slope of the curve at the given point.


1. 𝑦 = 𝑥3, 𝑎𝑡 (2, 8)

2. Find how fast is the circumference of the circle


increases when the radius increases.

This document is the property of PHINMA EDUCATION


11
Mat 171: Differential Calculus
Student’s Activity Sheets # 4

Name: _______________________________________________________ Class number: ________


Section: ____________ Schedule: _________________________________ Date: _______________

C. LESSON WRAP-UP

1) Activity 6: Thinking about Learning

Think about your Learning

1. What motivated you to finish the lesson today?


________________________________________________________________________
________________________________________________________________________

2. What could you have done better to improve your learning today?
_________________________________________________________________________
_________________________________________________________________________

FAQs

1. How do you interpret the slope as a rate of change?


A linear function is increasing if the slope is positive and decreasing if the slope is
negative; so, the slope is interpreted as a rate of change.

2. Why is slope important in real life?


Slope is a measure of steepness. Some examples include: When constructing
wheelchair ramps, slope is a major consideration. When designing the stairs, consider its
steepness.

This document is the property of PHINMA EDUCATION


12
Mat 171: Differential Calculus
Student’s Activity Sheets # 4

Name: _______________________________________________________ Class number: ________


Section: ____________ Schedule: _________________________________ Date: _______________

KEY TO CORRECTIONS

ACTIVITY 3:
A. 1. 𝟐𝒙 − 𝟒
2. 𝟑𝒙𝟐 + 𝟐
3. −𝐬𝐢𝐧 𝒙

B. 1. 𝟎
2. 𝟐𝝅𝒓

ACTIVITY 5.
−𝟏𝟎
A. 1.
(𝟐𝒙+𝟑)𝟐
𝟏
2.
𝟐√𝒙+𝟓

B. 1. 12
2. 𝟐𝝅

This document is the property of PHINMA EDUCATION


13
MAT 171: Differential Calculus
Student’s Activity Sheets #5

Name: __________________________________________________________ Class number: ______


Section: ____________ Schedule:___________________________________ Date: ______________

Lesson Title: STANDARD DIFFERENTIATION Materials: Ballpoint, Notebook, Calculator


FORMULAS & THE CHAIN RULE OF
DIFFERENTIATION Textbook: Calculus by Ron Larson 11th Ed
Lesson Objectives: References:
At the end of the lesson, you should be able to: 1. The Calculus 7 by Louis Leithold
1. State and Illustrate the Standard Differentiation 2. Differential and Integral Calculus
Formulas by Clyde E. Love and Earl Rainville
1. 2. State the Chain Rule and related formulas
2. 3. Differentiate Algebraic Functions
3. 4. Find the derivative of functions using the
4. chain rule.

Productivity Tip
Obstacles are put in your way to help you determine if what you
want is really worth fighting for! If you really want to do something,
you’ll find a way. If you don’t, you’ll find an excuse.

A. LESSON PREVIEW/REVIEW

1) Introduction
he following topics shall be accomplished in this lesson:
1. State and Illustrate the Standard Differentiation Formulas; and
2. State the Chain Rule and related formulas
3. Differentiate Algebraic Functions
4. Find the derivative of functions using the chain rule

Activity 1: What I Know Chart, part 1

What I Know Questions: What I Learned (Activity 4)


1. Will you state the Standard
Differentiation Formulas? Can you
Illustrate them?
2. Do you know how to
differentiate algebraic functions?

3. Can you state the chain rule


and related formulas?

4. How do you find the derivative


of functions using the chain rule?

This document is the property of PHINMA EDUCATION


1
MAT 171: Differential Calculus
Student’s Activity Sheets #5

Name: __________________________________________________________ Class number: ______


Section: ____________ Schedule:___________________________________ Date: ______________

B.MAIN LESSON

THE DIFFERENTIATION FORMULAS OF ALGEBRAIC FUNCTIONS

1. CONSTANT RULE:

“The Derivative of a Constant is zero”

𝒅𝑪
=𝟎
𝒅𝒙

𝑑
Example 1.1 (2) =𝟎
𝑑𝑥

𝑑 1
1.2 ( ) =𝟎
𝑑𝑥 5

𝑑
1.4 (𝜋) =𝟎
𝑑𝑥

2. POWER RULE:

“ The Derivative of the Power of a Function is equal to


the power multiplied by the function raised to the power minus one”

𝒅 𝒏
𝒙 = 𝒏𝒙𝒏−𝟏
𝒅𝒙
𝒅
(𝒙) = 𝟏
𝒅𝒙

𝑑 5
Example 2.1 𝑑𝑥
𝑥 = 5𝑥 5−1
= 𝟓𝒙𝟒
𝑑 −4
2.3 𝑑𝑥
𝑥 = −4𝑥 −4−1
= −𝟒𝒙−𝟓

𝑑 3 3 3
2.4 𝑑𝑥
𝑥2 = 2 𝑥 2−1
𝟏
𝟑
= 𝟐 𝒙𝟐

This document is the property of PHINMA EDUCATION


2
MAT 171: Differential Calculus
Student’s Activity Sheets #5

Name: __________________________________________________________ Class number: ______


Section: ____________ Schedule:___________________________________ Date: ______________

3. CONSTANT MULTIPLE RULE

“The derivative of C multiplied by a power of x is equal to


C multiplied by the derivative of the power x”

𝒅 𝒅
(𝑪𝒙𝒏 ) = 𝑪 (𝒙𝒏 ) = 𝑪 (𝒏𝒙𝒏−𝟏 )
𝒅𝒙 𝒅𝒙

𝑑 𝑑
Example: 3.1 𝑑𝑥
5𝑥 2 =5 𝑑𝑥
𝑥2

= 5[2𝑥 2−1 ]
= 𝟏𝟎𝒙

𝑑 𝑑 −3
3.2 5𝑥 −3 =5 𝑥
𝑑𝑥 𝑑𝑥

= 5[−3𝑥 −3−1 ]
= −𝟏𝟓𝒙−𝟒

4. SUM OR DIFFERENCE RULE: “The Derivative of the Sum or Difference of


Two Functions is equal to the sum or difference of their derivatives”

If u and v are the functions x, the following formulas are true by the definition of
the derivative.
𝒅 𝒅𝒖 𝒅𝒗
(𝒖 ± 𝒗) = ±
𝒅𝒙 𝒅𝒙 𝒅𝒙

𝑑 𝑑 𝑑
Example: 4.1 (4𝑥 + 5) = 𝑑𝑥 (4𝑥) + 𝑑𝑥 (5)
𝑑𝑥
=4+0
=4

𝑑 𝑑 𝑑
4.2 (4𝑥 3 − 3) = 𝑑𝑥 (4𝑥 3 ) − 𝑑𝑥 (3)
𝑑𝑥
= 4(3𝑥 2 ) − 0
= 𝟏𝟐𝒙𝟐

𝑑 𝑑 𝑑 𝑑 𝑑
4.3 (7𝑥 3 − 5𝑥 2 + 3𝑥 − 5) = 𝑑𝑥 (7𝑥 3 ) − 𝑑𝑥 (5𝑥 2 ) + 𝑑𝑥 (3𝑥) − 𝑑𝑥 (5)
𝑑𝑥

= 𝟐𝟏𝒙𝟐 − 𝟏𝟎𝒙 + 𝟑

This document is the property of PHINMA EDUCATION


3
MAT 171: Differential Calculus
Student’s Activity Sheets #5

Name: __________________________________________________________ Class number: ______


Section: ____________ Schedule:___________________________________ Date: ______________

5. THE QUOTIENT RULE: “The Derivative of the Quotient of Two Functions is equal to
the denominator multiplied by the derivative of the numerator, minus
the numerator multiplied by the derivative of the denominator,
all divided by the square of the denominator”

𝒅𝒖 𝒅𝒗
𝒅 𝒖 𝒗 −𝒖
𝒅𝒙 𝒅𝒙
( )= 𝟐
𝒅𝒙 𝒗 𝒗

A very Common Mnemonic for the Quotient Rule is,

𝒉𝒊𝒈𝒉 𝒍𝒐𝒘 𝑫 (𝒉𝒊𝒈𝒉)−𝒉𝒊𝒈𝒉 𝑫 (𝒍𝒐𝒘)


𝒅𝒙 [ ]=
𝒍𝒐𝒘 𝒍𝒐𝒘 𝒔𝒒𝒖𝒂𝒓𝒆𝒅

𝑑 𝑑
𝑑 3+2𝑥 (5−3𝑥) (3+2𝑥)−(3+2𝑥) (5−3𝑥)
𝑑𝑥 𝑑𝑥
Example: 5.1 ( )=
𝑑𝑥 5−3𝑥 (5−3𝑥)2
(5−3𝑥)(2)−(3+2𝑥)(−3)
=
(5−3𝑥)2
10−6𝑥+9+6𝑥
=
(5−3𝑥)2
𝟏𝟗
=
(𝟓−𝟑𝒙)𝟐

𝑑 𝑑
𝑑 5𝑥+6 (3𝑥 3 −2) (5𝑥+6)−(5𝑥+6) (3𝑥 3 −2)
𝑑𝑥 𝑑𝑥
5.2 (
𝑑𝑥 3𝑥 3 −2
)= (3𝑥 3 −2)2

(3𝑥 3 −2)(5)−(5𝑥+6)(9𝑥 2 )
=
(3𝑥 3 −2)2

15𝑥 3 −10−45𝑥 3 −54𝑥 2


=
(3𝑥 3 −2)2

−30𝑥 3 −54𝑥 2 −10


=
(3𝑥 3 −2)2

−𝟐(𝟏𝟓𝒙𝟑 +𝟐𝟕𝒙𝟐 +𝟓)


=
(𝟑𝒙𝟑 −𝟐)𝟐

This document is the property of PHINMA EDUCATION


4
MAT 171: Differential Calculus
Student’s Activity Sheets #5

Name: __________________________________________________________ Class number: ______


Section: ____________ Schedule:___________________________________ Date: ______________

6. PRODUCT RULE “The Derivative of the Product of Two Functions is equal to the
first function multiplied by the derivative of the second function plus the
second function multiplied by the derivative of the first function”

𝒅 𝒅𝒗 𝒅𝒖
(𝒖𝒗) = 𝒖 + 𝒗
𝒅𝒙 𝒅𝒙 𝒅𝒙
𝑑
Example: 6.1 (3𝑥 + 1)(2𝑥 − 5) let: 𝑢 = 3𝑥 + 1, 𝑣 = 2𝑥 − 5
𝑑𝑥

𝑑 𝑑 𝑑
(3𝑥 + 1)(2𝑥 − 5) = (3𝑥 + 1) 𝑑𝑥 (2𝑥 − 5) + (2𝑥 − 5) 𝑑𝑥 (3𝑥 + 1)
𝑑𝑥

= (3𝑥 + 1)(2) + (2𝑥 − 5)(3)


= 6𝑥 + 2 + 6𝑥 − 15
= 𝟏𝟐𝒙 − 𝟏𝟑
𝑑
6.2 [(𝑥 3 − 6𝑥)(𝑥 2 + 4)] let: 𝑢 = 𝑥 3 − 6𝑥, 𝑣 = 𝑥 2 + 4
𝑑𝑥
𝑑 𝑑 𝑑
[(𝑥 3 − 6𝑥)(𝑥 2 + 4)] = (𝑥 3 − 6𝑥) (𝑥 2 + 4) + (𝑥 2 + 4) (𝑥 3 − 6𝑥)
𝑑𝑥 𝑑𝑥 𝑑𝑥

= (𝑥 3 − 6𝑥)(2𝑥) + (𝑥 2 + 4)(3𝑥 2 − 6)
= 24 − 12𝑥 2 + 3𝑥 4 + 12𝑥 2 − 6𝑥 2 − 24
= 𝟓𝒙𝟒 − 𝟔𝒙𝟐 − 𝟐𝟒
6.a Formulas 5 and 6 can be extended to the case where n functions are
involved.
For three functions ( say, u, v, w ), formula 6 becomes,

𝒅 𝒅𝒖 𝒅𝒗 𝒅𝒘
𝒖𝒗𝒘 = 𝒗𝒘 + 𝒖𝒘 + 𝒖𝒗
𝒅𝒙 𝒅𝒙 𝒅𝒙 𝒅𝒙

In the special case when u=C, if C is a constant; 5 and 6 become,

𝒅 𝒅𝒗
6.b 𝒅𝒙
𝑪𝒗 = 𝑪 𝒅𝒙

𝑑 𝑑
Example 6.b.1 5𝑥 4 = 5 𝑥4
𝑑𝑥 𝑑𝑥

= 5(4𝑥 4−1 )
= 𝟐𝟎𝒙𝟑

This document is the property of PHINMA EDUCATION


5
MAT 171: Differential Calculus
Student’s Activity Sheets #5

Name: __________________________________________________________ Class number: ______


Section: ____________ Schedule:___________________________________ Date: ______________

𝒅𝒗
𝒅 𝑪 −𝑪
𝒅𝒙
6.c ( )=
𝒅𝒙 𝑽 𝒗𝟐

𝑑
𝑑 5 −5 (3𝑥−2)
𝑑𝑥
Example: 6.c.1 ( )=
𝑑𝑥 3𝑥−2 (3𝑥−2)2
−5(3)
=
(3𝑥−2)2
−𝟏𝟓
=
(𝟑𝒙−𝟐)𝟐

DERIVATIVE OF A FUNCTION

Derivative of 𝒙𝒏 , (The Power Rule)

𝒅𝒚
If 𝒚 = 𝒙𝒏 , then = 𝒏𝒙𝒏−𝟏
𝒅𝒙

This rule is very basic and should be mastered!


𝑑𝑦
In particular, if n=1, that is, if y=x, then = 1.
𝑑𝑥

𝑑𝑦
Example 1. If 𝑦 = 𝑥 5 , then, 𝑑𝑥
= 5𝑥 5−1 = 𝟓𝒙𝟒

𝑑𝑦
2. If 𝑦 = 𝑥 100 , = 𝟏𝟎𝟎𝒙𝟗𝟗
𝑑𝑥

1 1
3. If 𝑦 = 𝑥 3 𝑦 = 𝑥 3 = 𝑥 −3

In some cases, the laws of exponents must be used to rewrite an expression


before applying the power rule.
𝑑𝑦
Then, 𝑦 = 𝑥 −3 , 𝑑𝑥
= −3𝑥 −3−1

= −3𝑥 −4
𝑑𝑦 −𝟑
Or, 𝑑𝑥
= 𝒙𝟒

This document is the property of PHINMA EDUCATION


6
MAT 171: Differential Calculus
Student’s Activity Sheets #5

Name: __________________________________________________________ Class number: ______


Section: ____________ Schedule:___________________________________ Date: ______________

MORE EXAMPLES ON DIFFERENTIATING FUNCTIONS:

Example 1. Differentiating Sum of Functions

Differentiate 𝑦 = 5𝑥 2 + 3𝑥 + 10

Given: 𝑦 = 5𝑥 2 + 3𝑥 + 10
𝑑𝑦 𝑑 𝑑 𝑑
Solution: 𝑑𝑥
= 𝑑𝑥 5𝑥 2 + 𝑑𝑥 3𝑥 + 𝑑𝑥 10 using formula 3 and 5

𝑑𝑦
𝑑𝑥
= 10𝑥 + 3 + 0
𝑑𝑦
𝑑𝑥
= 𝟏𝟎𝒙 + 𝟑

Example 2. Differentiating Sum and Difference of Functions

1 3
1
Differentiate 𝑦 = 3𝑥 3 − 8𝑥 −4 + 3 𝑥
1 3
1
Given: 𝑦 = 3𝑥 3 − 8𝑥 −4 + 3 𝑥
1 3
𝑑𝑦 1 3 1
Solution: = 3 ( ) 𝑥 3−1 − 8(− )𝑥 −4−1 + using formula 3 and 5, then simplify
𝑑𝑥 3 4 3

𝟐 𝟕
𝟏
= 𝒙−𝟑 + 𝟔𝒙−𝟒 +
𝟑

Example 3. Differentiating Quotient of Two Functions

7𝑥−1
Differentiate 𝑦=
5𝑥 2 +2
7𝑥−1
Given: 𝑦=
5𝑥 2 +2
𝑑 𝑑
𝑑𝑦 (5𝑥 2 +2) (7𝑥−1)−(7𝑥−1) (5𝑥 2 +2)
𝑑𝑥 𝑑𝑥
= (5𝑥 2 +2)2
using formula 5
𝑑𝑥

(5𝑥 2 +2)(7)−(7𝑥−1)(10𝑥)
= (5𝑥 2 +2)2

35𝑥 2 +14−70𝑥 2 +10𝑥


= (5𝑥 2 +2)2

𝟏𝟒+𝟏𝟎𝒙−𝟑𝟓𝒙𝟐
= (𝟓𝒙𝟐 +𝟐)𝟐

This document is the property of PHINMA EDUCATION


7
MAT 171: Differential Calculus
Student’s Activity Sheets #5

Name: __________________________________________________________ Class number: ______


Section: ____________ Schedule:___________________________________ Date: ______________

Example 4. Differentiating Product of Functions


𝑑𝑦
Find 𝑑𝑥
if 𝑦 = (7𝑥 − 1)(5𝑥 2 + 2)

Given: 𝑦 = (7𝑥 − 1)(5𝑥 2 + 2)


𝑑𝑦 𝑑 𝑑
𝑑𝑥
= (7𝑥 − 1) 𝑑𝑥 (5𝑥 2 + 2) + (5𝑥 2 + 2) 𝑑𝑥
(7𝑥 − 1) using formula 6

= (7𝑥 − 1)(10𝑥 + 0) + (5𝑥 2 + 2)(7 − 0)


= 70𝑥 2 − 10𝑥 + 35𝑥 2 + 14
= 𝟏𝟎𝟓𝒙𝟐 − 𝟏𝟎𝒙 + 𝟏𝟒
Second Solution:
We can also multiply the two factors and get,
𝑦 = (7𝑥 − 1)(5𝑥 2 + 2) = 35𝑥 3 + 14𝑥 − 5𝑥 2 − 2
Using rules 3 and 4, the derivative of y is,
𝑑𝑦
𝑑𝑥
= 105𝑥 2 + 14 − 10𝑥
𝑑𝑦
𝑑𝑥
= 𝟏𝟎𝟓𝒙𝟐 − 𝟏𝟎𝒙 + 𝟏𝟒

Note that the answer is consistent with the solution using the product rule.

This document is the property of PHINMA EDUCATION


8
MAT 171: Differential Calculus
Student’s Activity Sheets #5

Name: __________________________________________________________ Class number: ______


Section: ____________ Schedule:___________________________________ Date: ______________

THE CHAIN RULE


There are functions written in complicated form, where differentiation formula will not
immediately apply. Here is the role of the Chain Rule to put the complicated expression to a
particular form where differentiation formula will easily apply. Chain Rule is also known as the
composite function rule.

If y is a differentiable function of u given by y = f(u) and if u is a differentiable function of x


given by u = g (x), then y is a differentiable function of x and we have,

𝒚 = 𝒇(𝒖)
𝒅𝒚 𝒅𝒚 𝒅𝒖
= [ ]
𝒅𝒙 𝒅𝒖 𝒅𝒙

The General Power Formula


By means of chain rule, we are able to differentiate any power of any function of x. To do this,
𝑑𝑦
let y = un where u =g (x). Then by the Power Rule, 𝑑𝑢 = n un-1. We have by the chain rule,

𝒚 = 𝒖𝒏
𝒅𝒚 𝒅𝒖
= 𝒏 𝒖𝒏−𝟏 [𝒅𝒙 ]
𝒅𝒙

In words: “The derivative of the power of a function is equal to the power multiplied by
the function raised to the power minus one, multiplied by the derivative of the function.”
An important special case of this formula is the case ,
1 −1
𝟏 𝑑 1 𝑑𝑢
If n=𝟐 then, 𝑑𝑥
𝑢2 = 2 𝑢 2 𝑑𝑥

𝒅𝒖
𝒅 𝒅𝒙
Thus, 𝒅𝒙
√𝒖 = 𝟐√𝒖

This document is the property of PHINMA EDUCATION


9
MAT 171: Differential Calculus
Student’s Activity Sheets #5

Name: __________________________________________________________ Class number: ______


Section: ____________ Schedule:___________________________________ Date: ______________

Example 1. Find the derivative of y = ( 𝟒𝒙𝟐 + 𝟑 )𝟏𝟎

Given: y = (4𝑥 2 + 3 )10 Write the original function


𝑛 2
This function is of the form 𝑢 , with u = 4𝑥 + 3, n= 10.
𝑑 𝑑𝑢
Hence , from 𝑑𝑥
un = n un-1 𝑑𝑥
,
𝑑𝑦 𝑑
we have 𝑑𝑥
= 𝑑𝑥
(4𝑥 2 + 3) 10

𝑑
= 10 ( 4𝑥 2 + 3)10−1 𝑑𝑥 (4𝑥 2 + 3)
= 10 ( 4𝑥 2 + 3)9 (8x) simplify
= 80x ( 4𝒙𝟐 + 3)𝟗

Note: In using the chain rule


we work from the outside to inside.
We differentiate the outer function
and then multiply by the derivative
of the inner function.

Example 2. Differentiate y = (3𝒙𝟐 + 𝟓 )𝟏𝟎𝟎

Given: y = (3𝑥 2 + 5 )100 write the original function


𝑑 n 𝑑𝑢
From u = n un-1
𝑑𝑥 𝑑𝑥
Solution: Taking u = (3𝑥 2 + 5), and n = 100,
𝑑𝑦 𝑑
By chain rule, = 100(3𝑥 2 + 5 )100−1 (3𝑥 2 + 5 )
𝑑𝑥 𝑑𝑥

= 100 (3𝑥 2 + 5 )99(6x)


= 600x (3𝒙𝟐 + 𝟓 )𝟗𝟗

𝟏
Example 3. Find the first derivative of y = 𝟖 (𝟓 − 𝟐𝒙)𝟔

1
Given: y = 8 (5 − 2𝑥)6 write the original function
𝑑𝑦 1 𝑑 𝑑 𝑑𝑢
𝑑𝑥
= 8
(6) (5 − 2𝑥)6−1 𝑑𝑥 (5 − 2𝑥) Apply Chain rule: 𝑑𝑥
un = n un-1 𝑑𝑥
𝑑𝑦 1 𝑑
𝑑𝑥
= 8 (6) (5 − 2𝑥)5 𝑑𝑥
(−2) simplify
𝟑
=- 𝟐
(𝟓 − 𝟐𝒙)𝟓

This document is the property of PHINMA EDUCATION


10
MAT 171: Differential Calculus
Student’s Activity Sheets #5

Name: __________________________________________________________ Class number: ______


Section: ____________ Schedule:___________________________________ Date: ______________
𝒅𝒚
Example 4a. If y = 6 √𝟒 + 𝒙 , find 𝒅𝒙
Given: y = 6 √4 + 𝑥
1
y = 6(4 + x)2 write the equation in exponential form
1
𝑑𝑦 𝑑 𝑑 𝑑𝑢
𝑑𝑥
= 𝑑𝑥
6(4 + x) 2 apply chain rule 𝑑𝑥
un = n un-1 𝑑𝑥
𝑑𝑥
And, note: 𝑑𝑥 = 1
1
𝑑𝑦 1 𝑑
𝑑𝑥
= 6 (2) (4 + x)2−1 𝑑𝑥
( 4 + x)
1
𝑑𝑦
= 3 (4 + x)− 2 (1) write with positive exponent
𝑑𝑥
3
= 1
(4+𝑥) 2
𝑑𝑦 𝟑
= write in radical form
𝑑𝑥 √𝟒+𝒙

𝒅𝒖
𝒅 𝒅𝒙
Another solution : By using the formula
𝒅𝒙
√𝒖 = 𝟐√𝒖

𝒅𝒚
Example 4b. If y = 6 √𝟒 + 𝒙 , find
𝒅𝒙
Given: y = 6 √4 + 𝑥
𝒅𝒖
𝑑𝑢 𝒅 𝒅𝒙
Let u = 4 + x ,
𝑑𝑥
=1 By using the formula
𝒅𝒙
√𝒖 = 𝟐√𝒖
𝑑
𝑑𝑦 (4+𝑥)
𝑑𝑥
Hence, =6
𝑑𝑥 2√4+𝑥
𝑑𝑦 3(1)
=
𝑑𝑥 √4+𝑥

𝒅𝒚 𝟑
Therefore: =
𝒅𝒙 √𝟒+𝒙

𝟑 𝒅𝒚
Example 5. If y = √(𝟏 − 𝒙𝟒 ) , Find 𝒅𝒙
3
Given : y = √(1 − 𝑥 4 ) write the original function
1
Solution: y = ( 1 − 𝑥 4 ) 3 write in exponential form
1
𝑑𝑦 1 −1 𝑑 4
= (1–x ) 4 3 (1− 𝑥 ) apply general formula, chain rule
𝑑𝑥 3 𝑑𝑥
𝑑 𝑑𝑢
𝑑𝑥
un = n un-1 𝑑𝑥
2
1
= (1 – x4 )− 3 (−4𝑥 3 ) simplify
3
2
−4
= 𝑥 3 (1 – x4 )− 3
3
−4 𝑥 3
= 2 write with positive exponent
3(1 – 𝑥 4 ) 3
𝑑𝑦 −𝟒 𝒙𝟑
𝑑𝑥
= 𝟑 write in radical form
𝟑 √(𝟏−𝒙𝟒 )𝟐

This document is the property of PHINMA EDUCATION


11
MAT 171: Differential Calculus
Student’s Activity Sheets #5

Name: __________________________________________________________ Class number: ______


Section: ____________ Schedule:___________________________________ Date: ______________
𝒅𝒚 𝟑
Example 6. Find 𝒅𝒙
, if y = .
√𝟏𝟔+ 𝒙𝟒

3
Given: y= write the original function
√16+ 𝑥 4
1

y = 3 ( 16 + x4) 2 write in exponential form
1
𝑑𝑦 1 4 (− −1) 𝑑 4
= (3) (− ) (16 + x ) 2 (16 + 𝑥 ) apply constant multiple rule and chain rule
𝑑𝑥 2 𝑑𝑥
3
𝑑𝑦 3 4 − 3
𝑑𝑥
= - 2
( 16 +x) 2 (4x ) simplify
−6𝑥 3
= 3 write with positive exponent
(16 + 𝑥 4 )2
𝒅𝒚 −𝟔𝒙𝟑
= write in radical form
𝒅𝒙 √(𝟏𝟔+ 𝒙𝟒 )𝟑

Example 7. Differentiate y = ( 1 – x)3 ( 2 + x)2


Given: y = ( 1 – x)3 ( 2 + x)2
𝑑 𝑑𝑣 𝑑𝑢
Solution: apply product rule 𝑑𝑥
(uv) = u𝑑𝑥 + v𝑑𝑥
𝑑𝑦 𝑑 𝑑
𝑑𝑥
= ( 1 – x)3 𝑑𝑥( 2 + x)2 + (2 + x)2 𝑑𝑥 ( 1 – x)3 apply product rule , chain rule
𝑑𝑦
𝑑𝑥
= ( 1- x)3 [(2) (2 + x)(1) ]+ ( 2 + x )2 [3(1 − x)2 (−1)]
𝑑𝑦
𝑑𝑥
= 2 ( 1 –x)3 ( 2 + x) + (-3) (2 + x)2(1 – x )2 arrange
Note “ multiplication is commutative”
= (1 – x)2 (2 + x) [2(1 − 𝑥) − 3 (2 + 𝑥 )] factor
= (1 – x)2 (2 + x) ( 2 -2x -6 -3x) simplify
= (1 – x)2 (2 + x) ( -5x -4)
𝒅𝒚
𝒅𝒙
= - (1 – x)2 (2 + x) (5x + 4)

𝒙𝟐
Example 8. Differentiate y =
√𝟏𝟔− 𝒙𝟐
𝑥2
Given: y= write the original function
√16− 𝑥 2
1
2 −2
Solution: y = x2 (16-x ) write in exponential form
𝑑 𝑑𝑣 𝑑𝑢
Apply product rule 𝑑𝑥
(uv) = u𝑑𝑥 + v𝑑𝑥 , and chain rule
We have,
1 1
𝑑𝑦 𝑑 𝑑 2
𝑑𝑥
= x2 𝑑𝑥
(16 – x2)− 2 + (16 – x2)− 2
𝑑𝑥
x

This document is the property of PHINMA EDUCATION


12
MAT 171: Differential Calculus
Student’s Activity Sheets #5

Name: __________________________________________________________ Class number: ______


Section: ____________ Schedule:___________________________________ Date: ______________
1 1
𝑑𝑦 1 𝑑
𝑑𝑥
= x2 (- 2 ) (16 – x2)(− 2
−1)
𝑑𝑥
(16 – x2) + (16 – x2)− 2 ( 2x)
3 1
𝑑𝑦 1
𝑑𝑥
= x2 (- 2 ) (16 – x2)(− 2
)
(- 2x) + (16 – x2)− 2 (2x)
3 1
𝑑𝑦
𝑑𝑥
= x3 (16 – x2)(− 2
)
+ 2x (16 – x2)− 2 factor
3
𝑑𝑦
𝑑𝑥
= x (16 – x2)− 2 [𝑥 2 + 2( 16 − 𝑥 2 )] simplify
3
𝑑𝑦
𝑑𝑥
= x (16 – x2)− 2 (x2 + 32 - 2x2 )
3
𝑑𝑦
𝑑𝑥
= x (16 – x2)− 2 (32 - x2 ) write with positive exponent
𝑑𝑦 x (32−𝑥 2 )
𝑑𝑥
= 3
(16− 𝑥 2 )2
𝒅𝒚 𝐱 (𝟑𝟐−𝒙𝟐 )
Thus, = write in radical form
𝒅𝒙 √(𝟏𝟔−𝒙𝟐 )𝟑

( 𝟑𝒙+𝟐)𝟒 𝒅𝒚
Example 9. If y = , find
(𝟏−𝒙)𝟔 𝒅𝒙

( 3𝑥+2)4
Given: y=
(1−𝑥)6
𝑑𝑢 𝑑𝑣
𝑑 𝑢 𝑣 −𝑢
𝑑𝑥 𝑑𝑥
Solution: apply quotient rule: ( )= and chain rule
𝑑𝑥 𝑣 𝑣2

𝑑 𝑑
𝑑𝑦 (1−𝑥)6 (3𝑥+2)4 − (3𝑥+2)4 ( 1−𝑥)6
𝑑𝑥 𝑑𝑥
𝑑𝑥
= [(1−𝑥)6 ]2
𝑑𝑦 (1−𝑥)6 [4(3𝑥+2)3 (3)] − (3𝑥+2)4 [6(1−𝑥)5 (−1)]
𝑑𝑥
= (1−𝑥)12
(1−𝑥)6 [12(3𝑥+2)3 ] − (3𝑥+2)4 [−6(1−𝑥)5 ]
=
(1−𝑥)12

12(1−𝑥)6 [(3𝑥+2)3 ] + 6(3𝑥+2)4 [(1−𝑥)5 ]


=
(1−𝑥)12

6(1−𝑥)5 (3𝑥+2)3 [2(1−𝑥)+(3𝑥+2)]


= (1−𝑥)12

6(1−𝑥)5 (3𝑥+2)3 (2−2𝑥+3𝑥+2)


=
(1−𝑥)12

6(1−𝑥)5 (3𝑥+2)3 (4+𝑥)


= (1−𝑥)12

𝒅𝒚 𝟔(𝟒+𝒙)(𝟑𝒙+𝟐)𝟑
Therefore, 𝒅𝒙
= (𝟏−𝒙)𝟕

This document is the property of PHINMA EDUCATION


13
MAT 171: Differential Calculus
Student’s Activity Sheets #5

Name: __________________________________________________________ Class number: ______


Section: ____________ Schedule:___________________________________ Date: ______________

Example10. Differentiate y = (7x - 1)𝟑 (5x + 2)𝟐

Given: y = (7x - 1)3 ( 5x + 2)2


𝑑 𝑑𝑣 𝑑𝑢
Apply product rule (uv) = u +v and chain rule
𝑑𝑥 𝑑𝑥 𝑑𝑥

Solution:
𝑑𝑦 𝑑 𝑑
𝑑𝑥
= (7x - 1)3 𝑑𝑥
(5x + 2)2 + (5x + 2)2 𝑑𝑥
(7x -1)3

𝑑𝑦
= (7x - 1)3 [2(5𝑥 + 2)(5)] + (5x + 2)2 [3(7𝑥 − 1)2 (7)]
𝑑𝑥

= 10 (7x - 1)3 (5x + 2) + 21 (5x + 2)2 (7x - 1)2


= (7x - 1)2 (5x +2) [10 (7𝑥 − 1) + 21 (5𝑥 + 2)]
= (7x - 1)2 (5x +2) (70x -10+ 105x + 42)
𝑑𝑦
𝑑𝑥
= (7x - 1)𝟐 (5x +2) (175x +32)

Activity 3: Skill-building Activities (with answer key)


Differentiate
the Functions
5 4
1. 𝑦 = 𝑥 3 − 𝑥 2 + 10
𝑥(2𝑥+1)
2. 𝑦 = 3𝑥−5

3. 𝑦 = (𝑥)(2𝑥 + 1)(3𝑥 − 5)

4. Find the derivative of y = ( 4𝑥 3 − 3𝑥 2 − 𝑥 − 5)3


𝑑𝑦
5. If y = √ 𝑥 3 − 4𝑥 + 1 ; Find 𝑑𝑥

( 𝑥−2)3
6. Find the first derivative of y = 𝑥2
.
Use three methods and check your answers.
Hint: Consider y as a product, as a quotient,
or with binomial expansion.

7. Differentiate y = (3x + 1)3 ( 2𝑥 − 3)4

This document is the property of PHINMA EDUCATION


14
MAT 171: Differential Calculus
Student’s Activity Sheets #5

Name: __________________________________________________________ Class number: ______


Section: ____________ Schedule:___________________________________ Date: ______________

Activity 4: What I Know Chart, part 2

What I Learned
1. ______________________________________________________________________________
______________________________________________________________________________

2. ______________________________________________________________________________
______________________________________________________________________________

3. ______________________________________________________________________________
______________________________________________________________________________

2) Activity 5: Check for Understanding

Differentiate
The Functions

1. Find the derivative of y = ( 𝑥 2 - 3x + 1 )4


𝑑𝑦
2. If y = √5 − 3𝑥 , Find 𝑑𝑥

C. LESSON WRAP-UP
1) Activity 6: Thinking about Learning
You are done with this session! Let’s track your progress. Shade the session number you just
completed.

A. Think about your Learning


1. What motivated you to finish the lesson today?
________________________________________________________________________
________________________________________________________________________

2. What could you have done better to improve your learning today?
_________________________________________________________________________
_________________________________________________________________________

This document is the property of PHINMA EDUCATION


15
MAT 171: Differential Calculus
Student’s Activity Sheets #5

Name: __________________________________________________________ Class number: ______


Section: ____________ Schedule:___________________________________ Date: ______________

FAQs
1. What is the first principle of differentiation?
The Slope of a Curve as a Derivative. This is called differentiation from the first principle (or the
Delta Method). It gives the instantaneous rate of change of y with respect to x.

2. What is the purpose of differentiation?


Differentiation helps to find instantaneous rate of change of a function with respect to an
independent variable. It is used when a quantity shows non-linear variable. You can find the
velocity of a particle at a particular time by knowing the distance as a function of time.

3. How does chain rule works?


This rule is called the chain rule because we use it to take derivatives of composition of
functions by chaining together their derivatives. The chain rule can be thought of as taking the
derivative of the outer function (applied to the inner function) and multiplying it times the
derivative of the inner function.

4. Who invented the chain rule?


The chain rule has been known since Isaac Newton and Leibniz first discovered the calculus at
the end of the 17th century. The rule facilitates calculations that involve finding the derivatives of
complex expressions, such as those found in many physics applications.

KEY TO CORRECTIONS
Activity 3:
𝑑𝑦 −15 8
1. = 4 +
𝑑𝑥 𝑥 𝑥3

6𝑥 2 −20𝑥−5
2. 𝑦 ′ = (3𝑥−5)2

3. 𝑦 = 18𝑥 2 − 14𝑥 − 5

𝑑𝑦
4. 𝑑𝑥
= 3 ( 4x3 - 3x2 – x – 5)2 (12x2 -6x -1)

𝑑𝑦 3 𝑥2− 4
5. =
𝑑𝑥 2√𝑥 3 −4𝑥+1

𝑑𝑦 (𝑥+4)(𝑥−2)2
6. 𝑑𝑥
= 𝑥3

𝑑𝑦
7. 𝑑𝑥
= (2x - 3)3 (3x + 1)2 (42x – 19)

Activity 5:
𝑑𝑦
5.1 𝑑𝑥
= 4 (2x – 3) ( 𝑥 2 – 3x + 1)3
𝑑𝑦 −3
5.2 𝑑𝑥 = 2√5−3𝑥

This document is the property of PHINMA EDUCATION


16
MAT 171: Differential Calculus
Student’s Activity Sheets #6

Name: ___________________________________________________________ Class number: ______


Section: ____________ Schedule: ____________________________________ Date: _____________

Lesson Title: HIGHER DERIVATIVES Materials: Ballpoint, Notebook, Calculator

Lesson Objectives: Textbook: Calculus by Ron Larson , 11th


At the end of the lesson, you should be able to: Ed.
1. Find the higher derivative of functions References:
1. 2. Derive other forms of the second derivative 1. The Calculus 7 by Louis Leithold
2. Differential and Integral Calculus
by Clyde E. Love and Earl Rainville

NOTHING WORTH HAVING COMES EASY.


WE DON’T GROW WHEN THINGS ARE EASY,
WE GROW WHEN WE FACE CHALLENGES.
LESSON PREVIEW/REVIEW

1) Introduction

GOOD DAY! The following topics shall be accomplished


in this lesson:
1. Find the higher derivative of functions
2. Derive other forms of the second derivative

2) Activity 1: What I Know Chart, part 1

What I Know Questions: What I Learned (Activity 4)


1 Do you know how to find the
higher derivative of a function?
2 Can you derive other forms of
the second derivative?

B. MAIN LESSON
1) Activity 2: Content Notes

THE HIGHER DERIVATIVE

The derivative of y with respect to x is itself of function of x, and may in the turn be
differentiated. The derivative of the first derivative is called the second derivative and is
𝒅𝟐 𝒚 𝒅𝟑 𝒚
written 𝒅𝒙𝟐
. The derivative of the second derivative is the third derivative and written as, 𝒅𝒙𝟑
,
𝒅𝟒 𝒚 𝒅𝟓 𝒚
etc. Other higher derivatives are 𝒅𝒙𝟒
, 𝒅𝒙𝟓
, etc.

This document is the property of PHINMA EDUCATION


1
MAT 171: Differential Calculus
Student’s Activity Sheets #6

Name: ___________________________________________________________ Class number: ______


Section: ____________ Schedule: ____________________________________ Date: _____________

Further differentiation give us the derivative of order higher than 2. These derivatives
are defined and denoted as follows.

𝑑3 𝑦
= 𝑓 ′′′ (𝑥) = 𝒚′′′ third derivative
𝑑𝑥 3
𝑑4 𝑦
𝑑𝑥 4
= 𝑓 (4) (𝑥) = 𝒚(𝟒) fourth derivative
𝑑5 𝑦
𝑑𝑥 5
= 𝑓 (5) (𝑥) = 𝒚(𝟓) fifth derivative
𝑑𝑛𝑦
= 𝑓 (𝑛) (𝑥) = 𝒚(𝒏) nth derivative
𝑑𝑥 𝑛

Note that parentheses are used in y(n) and f (n)(x). The symbol y(n) is used to
distinguish it from the symbol yn. Recall that yn indicates the nth power of y = f(x)
while the present notation y(n) indicates the nth derivative of y = f(x).

Example 1. If y = x4 – 2x3 + 5x2 + 10 ,


𝑑𝑦
Then, = y' = 4x3 – 6x2 + 10x
𝑑𝑥
𝑑2 𝑦
𝑑𝑥 2
= y'' = 12x2 – 12x + 10
𝑑3 𝑦
𝑑𝑥 3
= y''' = 24x – 12
𝑑4 𝑦
𝑑𝑥 4
= y(4) = 24
𝑑5 𝑦
𝑑𝑥 5
= y(5) = 0

𝟓
Example 2. Find the second derivative and the third derivative of y = 𝒙
5
Given : y=
𝑥
𝑑𝑦 −5
Solution: 𝑑𝑥
= 𝑥2
first derivative , quotient rule
𝑑2 𝑦 +5(2𝑥)
𝑑𝑥 2
= 𝑥4
+𝟏𝟎
= 𝒙𝟑
second derivative,
𝑑3 𝑦 −10 (3𝑥 2 )
𝑑𝑥 3
= 𝑥6
−30𝑥 2
= 6
𝑥
−𝟑𝟎
= 𝟒 third derivative
𝒙

This document is the property of PHINMA EDUCATION


2
MAT 171: Differential Calculus
Student’s Activity Sheets #6

Name: ___________________________________________________________ Class number: ______


Section: ____________ Schedule: ____________________________________ Date: _____________

Example 3. Find the second derivative of y = ( x2 + x + 1 ) 2


Given : y = ( x2 + x + 1 ) 2
𝑑𝑦
Solution: 𝑑𝑥 = 2( 𝑥 2 + 𝑥 + 1 )( 2𝑥 + 1 ) power rule, chain rule
𝑑2 𝑦
𝑑𝑥 2
= 2 [ (𝑥 2 + 𝑥 + 1 )(2) + ( 2𝑥 + 1)( 2𝑥 + 1 )] Product rule

= 2 ( 2𝑥 2 + 2𝑥 + 2 + 4𝑥 2 + 4𝑥 + 1 ) Distributive law
= 2 ( 6𝑥 2 + 6𝑥 + 3 ) simplify
= 𝟔 ( 𝟐𝒙𝟐 + 𝟐𝒙 + 𝟏) factor out by 3

Example 4. Find the third derivative of y = √𝟒 − 𝒙𝟐


Given : y = √4 − 𝑥 2
𝑑𝑢
𝑑𝑦 −2𝑥 𝑑
Solution: = from (√𝑢 ) = 𝑑𝑥
𝑑𝑥 2√4−𝑥 2 𝑑𝑥 2 𝑢

𝑑𝑦 −𝑥
=
𝑑𝑥 √4−𝑥 2
−2𝑥
√4−𝑥 2 (−1) − (−𝑥)( )
𝑑2𝑦 2√4−𝑥2
= apply quotient rule
𝑑𝑥 2 (4−𝑥 2 )
𝑥2
−√4−𝑥 2 −
√4−𝑥2
= simplify
4−𝑥 2
−( 4−𝑥 2 )−𝑥 2
=
(√4−𝑥 2 )( 4−𝑥 2 )

− 4+𝑥 2 −𝑥 2
= ( 4−𝑥 2 )3/2

𝑑2 𝑦 −4 1 3
𝑑𝑥 2
= ( 4−𝑥2 )3/2 2
+1= 2

𝑑2 𝑦
= - 4 (4 – x2) -3/2 law of exponent
𝑑𝑥 2
3
𝑑3 𝑦 12
= ( 4 − 𝑥 2 )− 2 −1 (-2x) apply power rule & chain rule
𝑑𝑥 3 2

= -12x( 4- x2 )-5/2 simplify


−12𝑥
= 5
( 4−𝑥 2 ) 2

𝒅𝟑 𝒚 −𝟏𝟐𝒙
therefore : 𝒅𝒙𝟑
= 𝟓
(√𝟒−𝒙𝟐 )

This document is the property of PHINMA EDUCATION


3
MAT 171: Differential Calculus
Student’s Activity Sheets #6

Name: ___________________________________________________________ Class number: ______


Section: ____________ Schedule: ____________________________________ Date: _____________

𝒅𝟐 𝒚
Example 5. If y = x3- x , Find and interpret 𝒅𝒙𝟐
Given: 𝑦 = 𝑥3 − 𝑥
𝑑𝑦 𝑑𝑦
Solution: 𝑦′ = 𝑑𝑥
= 3𝑥 2 − 1 We found that the first derivative is 𝑑𝑥 = 3𝑥 2 − 1.
𝑑2 𝑦 𝒅𝟐 𝒚
So the second derivative is 𝑑𝑥 2
= 6𝑥 . 𝒚′′ = 𝒅𝒙𝟐 = 𝟔𝒙

The graph of y, y', y'' are shown in the figure.

Since y'' is the derivative of y', we see that the second derivative is the rate of change of
slope of the graph of the function. In general, we can interpret a second derivative as a
rate of change of a rate of change. The most familiar example of this is acceleration.

𝒅𝒗 𝒅𝟐 𝒔
In Leibniz notation. 𝒂= = 𝒅𝒕𝟐
𝒅𝒕

This document is the property of PHINMA EDUCATION


4
MAT 171: Differential Calculus
Student’s Activity Sheets #6

Name: ___________________________________________________________ Class number: ______


Section: ____________ Schedule: ____________________________________ Date: _____________

2) Activity 3: Skill-building Activities (with answer key)

Evaluate the following.

1. 𝑦 = ( 𝑥 3 − 2𝑥)2 , find y'''


𝑥
2. 𝑦= find y''
𝑥+1

3
3. 𝑦 = √(1 + 2𝑥)2 , find y''
𝑥3
4. 𝑦 = √(1−𝑥) find y' and y''
𝑑𝑦 𝑑2 𝑦
5. Find and of 𝑦 = (𝑥 + 2)2 (2 − 1)3
𝑑𝑥 𝑑𝑥 2

3) Activity 4: What I Know Chart, part 2

What I Learned
1. ______________________________________________________________________________
______________________________________________________________________________
2. ______________________________________________________________________________
______________________________________________________________________________
3. ______________________________________________________________________________
______________________________________________________________________________

4) Activity 5: Check for Understanding


𝟏
Evaluate: 1. 𝒚= , find y(4)
√𝒙

C. LESSON WRAP-UP
1) Activity 6: Thinking about Learning

A. Work Tracker
You are done with this session! Let’s track your progress. Shade the session number you just
completed.

This document is the property of PHINMA EDUCATION


5
MAT 171: Differential Calculus
Student’s Activity Sheets #6

Name: ___________________________________________________________ Class number: ______


Section: ____________ Schedule: ____________________________________ Date: _____________

B. Think about your Learning

1. What motivated you to finish the lesson today?


_____________________________________________________________________________
_____________________________________________________________________________

2. What could you have done better to improve your learning today?
_____________________________________________________________________________
_____________________________________________________________________________

FAQs

1. What does derivative mean in real life?


The derivative is often called as the “instantaneous” rate of change. The derivative of a
function represents an infinitely small change of the function with respect to one of its variation.

2. Can a function have multiple derivatives?


No. A function cannot have more than one derivative. Recall that we can define a
derivative as: And a limit of a real-valued function cannot approach more than one value ………
if two different equations describe the same set of points, then they describe the same function.

KEY TO CORRECTIONS

Evaluate the following.


1. 𝑦 ′′′ = 24x (5x2 – 4)
−𝟐
2. 𝒚′′ = (𝒙+𝟏)𝟑
𝟖
3. 𝒚′′ = − 𝟑
𝟗 √(𝟏+𝟐𝒙)𝟒
3 1 3
1
4. 𝑦′ = 2 (1 − 𝑥)− 2 (3𝑥 2 − 2𝑥 2 )
𝟑 −𝟏 𝟓
𝒚′′ = 𝟒
𝒙 𝟐 (𝟏 − 𝒙)𝟐
𝑑𝑦
5. 𝑑𝑥
= 10[(2𝑥 − 1)2 (𝑥 + 2)(𝑥 + 1)]
𝒅𝟐 𝒚
𝒅𝒙𝟐
= 𝟏𝟎(𝟐𝒙 − 𝟏)(𝟖𝒙𝟐 + 𝟏𝟔𝒙 + 𝟓)

Activity 5
𝟏𝟎𝟓
1. y(4) =
𝟏𝟔√𝒙𝟗

This document is the property of PHINMA EDUCATION


6
MAT 171: Differential Calculus
Student’s Activity Sheets #7

Name: ________________________________________________________ Class number:______


Section: ____________ Schedule:__________________________________ Date: _____________

Lesson Title: IMPLICIT DIFFERENTIATION Materials: Ballpoint, Notebook, Calculator


AND THE DIFFERENTIAL
Textbook: Calculus by Ron Larson, 11th Ed.
Lesson Objectives: References:
At the end of the lesson, you should be able to: 1. The Calculus 7 by Louis Leithold
1. State and illustrate the rule on implicit 2. Differential and Integral Calculus
differentiation. by Clyde E. Love and Earl Rainville
2. Find the first derivative, second derivative,
and slope of implicit functions.
3. Define and illustrate the differential of a
function.
4. Find the differential of a function

Don’t limit your challenges, challenge


your limit. Make the most every day!

A. LESSON PREVIEW/REVIEW
1) Introduction

GOOD DAY! The following topics shall be accomplished in this


lesson:
1. State and illustrate the rule on implicit differentiation
2. Find the first derivative, second derivative, and slope
of implicit functions
3. Define and illustrate the differential of a function
4. Find the differential of a function

1) Activity 1: What I Know Chart, part 1

What I Know Questions: What I Learned (Activity 4)


1 Can you state and illustrate the
rule on implicit differentiation?
2 Do you know how to find the
first derivative, second derivative,
and slope of implicit functions?

3. Can you define and illustrate


the differential of the function?
4. Do you know how to find the
differential of the function?

This document is the property of PHINMA EDUCATION


1
MAT 171: Differential Calculus
Student’s Activity Sheets #7

Name: ________________________________________________________ Class number:______


Section: ____________ Schedule:__________________________________ Date: _____________

B.MAIN LESSON
1) Activity 2: Content Notes

IMPLICIT DIFFERENTIATION
In the preceding lessons, we have been concerned mainly with functions defined by the
equation y = f(x). In this form, y is said to be an explicit function of x.

For example, in the equation 𝑦 = 𝑥 2 + 4𝑥 + 10, y is an explicit function of x.

If y is a function of x but is not expressed explicitly in terms of x, then y is said to be an


implicit function of x. In each of the equations below, y is an implicit function of x.
1. 𝑥 2 + 4𝑥𝑦 + 4𝑦 2 = 0
2. 𝑥 2 + 𝑦 2 − 6𝑥 + 2𝑦 = 0
3. 2 − (1 − 𝑥) ln 𝑦 = 0
In general, an implicit function may be represented by the equation f (x, y) = 0.
𝑑𝑦
To find 𝑑𝑥
or y' of an implicit function, we differentiate both sides of the equation with respect to
𝑑𝑦
x and then solve for 𝑑𝑥
or y'. The process involved is called implicit differentiation.

Guidelines for implicit differentiation

1. Differentiate both sides of the equation with respect to x, bearing in


𝑑𝑦
mind that y is a function of x, that is, 𝑑𝑥 or y' follows when the

variable y is differentiated.
𝑑𝑦
2. Collect all terms involving 𝑑𝑥
on the left side of the equation and

move all other terms to the right side of the equation.


𝑑𝑦
3. Factor 𝑑𝑥
on the left side of the equation.

𝑑𝑦
4. Solve for 𝑑𝑥
.

This document is the property of PHINMA EDUCATION


2
MAT 171: Differential Calculus
Student’s Activity Sheets #7

Name: ________________________________________________________ Class number:______


Section: ____________ Schedule:__________________________________ Date: _____________

𝒅𝒚
Example1. Determine 𝒅𝒙
of the given implicit function 𝒙𝟐 + 𝒚𝟐 = 𝟑𝟔

Given: 𝑥 2 + 𝑦 2 = 36
Solution:
Differentiate both sides of the equation 𝑥 2 + 𝑦 2 = 36
𝑑 𝑑
(𝑥 2 + 𝑦 2 ) = (36)
𝑑𝑥 𝑑𝑥
𝑑 𝑑 𝑑
(𝑥 2 ) + (𝑦 2 ) = 𝑑𝑥 (36)
𝑑𝑥 𝑑𝑥
𝑑 𝑑
(𝑥 2 ) + (𝑦 2 ) =0
𝑑𝑥 𝑑𝑥

Remember that y is a function of x. Using the Chain Rule, we have,


𝑑𝑦
2𝑥 + 2𝑦 =0
𝑑𝑥
𝑑𝑦
Collect all terms involving 𝑑𝑥 on the left side of the equation and move all other
terms to right side of the equation.
𝑑𝑦
2𝑦 𝑑𝑥 = −2𝑥
𝑑𝑦
Now, we solve for ;
𝑑𝑥
𝑑𝑦 −2𝑥
=
𝑑𝑥 2𝑦
𝒅𝒚 𝒙
Therefore, 𝒅𝒙
= −𝒚

Example 2. Alternative solution in solving the equation 𝒙𝟐 + 𝒚𝟐 = 𝟑𝟔


We get, 𝑦 = ±√36 − 𝑥 2 .
Differentiating: 𝑦 = √36 − 𝑥 2
= (36 − 𝑥 2 )1/2
Using chain rule , we have,
1
𝑑𝑦 1 𝑑
𝑑𝑥
= 2 (36 − 𝑥 2 )2−1 𝑑𝑥 (36 − 𝑥 2 )
1
1
= (36 − 𝑥 2 )− 2 (−2𝑥)
2
𝑑𝑦 𝑥
= −
𝑑𝑥 √36−𝑥 2
𝑑𝑦 𝑥
Note: The expression 𝑑𝑥
= − 𝑦 in solution 1 gives the derivative, both in terms
of x and y. It is correct no matter which function y determines the given
equation. For instance, for y = f(x) = √36 − 𝑥 2 ,
𝒅𝒚 𝒙 −𝒙
we have, 𝒅𝒙
= −𝒚 =
√𝟑𝟔−𝒙𝟐

This document is the property of PHINMA EDUCATION


3
MAT 171: Differential Calculus
Student’s Activity Sheets #7

Name: ________________________________________________________ Class number:______


Section: ____________ Schedule:__________________________________ Date: _____________

𝒅𝒚
Example 3. If 𝟑𝒙𝟐 + 𝟒𝒙𝒚 + 𝟒𝒚𝟐 = 𝟎 , find 𝒅𝒙
Solution:
3𝑥 2 + 4𝑥𝑦 + 4𝑦 2 = 0
𝑑
𝑑𝑥
(3𝑥 2 + 4𝑥𝑦 + 4𝑦 2 ) = 0
Differentiating both sides with respect to x, regarding y as a function of x, and
using the Chain Rule on the term 4y2 and the Product Rule on the term 4xy, we
get,
𝑑
6𝑥 + 4 𝑑𝑥 (𝑥𝑦) + 8𝑦𝑦’ = 0
6𝑥 + 4[𝑥𝑦′ + 𝑦(1)] + 8𝑦𝑦′ = 0
6𝑥 + 4𝑥𝑦′ + 4𝑦 + 8𝑦𝑦′ = 0
Collect all terms involving y' on the left side of the equation
4𝑥𝑦’ + 8𝑦𝑦’ = −6𝑥 − 4𝑦
factor out y’
𝑦’(4𝑥 + 8𝑦) = −6𝑥 − 4𝑦
we now solve for y'
−6𝑥−4𝑦
𝑦′ = 4𝑥+8𝑦
−2(3𝑥+2𝑦)
𝑦′ =
4(𝑥+2𝑦)
𝒅𝒚 −(𝟑𝒙+𝟐𝒚)
Therefore, = 𝒚′ =
𝒅𝒙 𝟐(𝒙+𝟐𝒚)

Example 4. Find the second derivative of 𝒙𝟐 + 𝒚𝟐 = 𝟒


Given: 𝑥2 + 𝑦2 = 4
Solution: Differentiating the equation implicitly with respect to x, we have,
2𝑥 + 2𝑦𝑦′ = 0
2𝑦𝑦′ = −2𝑥
Solving for y' gives,
2𝑥
𝑦′ = − 2𝑦
𝑥
𝑦′ = − 𝑦

Differentiating further with respect to x,


𝑦(1)−𝑥𝑦 ′
𝑦′′ = −[ ]
𝑦2

Using quotient rule and remembering that y is a function of x.

This document is the property of PHINMA EDUCATION


4
MAT 171: Differential Calculus
Student’s Activity Sheets #7

Name: ________________________________________________________ Class number:______


Section: ____________ Schedule:__________________________________ Date: _____________

(𝑦−𝑥𝑦 ′ ) 𝑥
=− 𝑦2
but y' = − 𝑦
𝑥
[𝑦−𝑥(− )]
𝑦
=− 𝑦2
𝑥2
[𝑦+ ]
𝑦
=− 𝑦 2

𝑦2 +𝑥2
( )
𝑦
=− 2 simplify
𝑦

(𝑦 2 +𝑥 2 )
=−
𝑦3
𝟒
therefore 𝒚′′ = − 𝒚𝟑 by substitution: 𝑥 2 + 𝑦 2 = 4

This is the second derivative.


Note that y'' can also be obtained without solving for y' in terms of x and y.
That is, starting with,
𝑥2 + 𝑦2 = 4
2𝑥 + 2𝑦𝑦’ = 0
𝑥 + 𝑦𝑦′ = 0
we can differentiate implicitly again to obtain,
1 + 𝑦𝑦’’ + 𝑦’𝑦’ = 0
1 + 𝑦𝑦’’ + (𝑦’)2 = 0
Solving for y'', we get
2
−(𝑦 ′ ) −1 𝑥
𝑦′′ = 𝑦
substituting 𝑦′ = − 𝑦
𝑥 2
−(− ) −1
𝑦
𝑦′′ = 𝑦
𝑥2
−( 2 )−1
𝑦
= simplify
𝑦

−(𝑥 2 +𝑦 2 )
= 𝑦3

since 𝑥 2 + 𝑦 2 = 4
−𝟒
Therefore, 𝒚′′ = 𝒚𝟑

This document is the property of PHINMA EDUCATION


5
MAT 171: Differential Calculus
Student’s Activity Sheets #7

Name: ________________________________________________________ Class number:______


Section: ____________ Schedule:__________________________________ Date: _____________

THE DIFFERENTIAL
𝑑𝑦
We have been using 𝑑𝑥
or y' to denote the derivative of y with respect to x. We define two
quantities 𝑑𝑥 𝑎𝑛𝑑 𝑑𝑦, which are the differential of the function f(x).

Consider two points P and Q on y = f(x).The figure shows the diagram of the differentials 𝑑𝑥
and 𝑑𝑦 with the given two points 𝑃(𝑥, 𝑦) and Q (𝑥 + ∆𝑥, 𝑦 + ∆𝑦) on the graph of 𝑦 = 𝑓(𝑥),
where ∆𝑦 = 𝑓(𝑥 + ∆𝑥) − 𝑓(𝑥).

The value of the derivative at P is equal to the slope of the tangent line PT.
𝑑𝑦
We have, 𝑑𝑥 = 𝑓 ′(𝑥) , 𝑜𝑟 𝑑𝑦 = 𝑓 ′ 𝑑𝑥. This will lead us to the following definition:

For the function 𝑦 = 𝑓(𝑥), we define the differential of the independent variable x is equal to the
increment of that variable. In mathematical symbols,

𝒅𝒙 = ∆𝒙

The differential 𝑑𝑦 is equal to its derivative multiplied by the differential of the independent
variable.
𝒅𝒚 = 𝒚′∆𝒙, 𝒐𝒓 𝒅𝒚 = 𝒇′(𝒙)𝒅𝒙

Definition:
Note: The differential of any function is equal to its derivative multiplied by the differential
of the independent variable.

This document is the property of PHINMA EDUCATION


6
MAT 171: Differential Calculus
Student’s Activity Sheets #7

Name: ________________________________________________________ Class number:______


Section: ____________ Schedule:__________________________________ Date: _____________

Example 1.Find the differential of the given function 𝒚 = 𝟑𝒙𝟒 − 𝟓𝒙𝟑 + 𝟐𝒙 + 𝟓.

Given: 𝑦 = 3𝑥 4 − 5𝑥 3 + 2𝑥 + 5
𝑑𝑦 = 12𝑥 3 𝑑𝑥 − 15𝑥 2 𝑑𝑥 + 2𝑑𝑥 Differentiate by the power rule, sum and difference rule
𝒅𝒚 = (𝟏𝟐𝒙𝟑 − 𝟏𝟓𝒙𝟐 + 𝟐)𝒅𝒙 From the definition “ the differential of y is equal to its
derivative multiplied by the differential of the
independent variable.”

Example 2. Find the differential of the function 𝒚 = √𝟐𝒙 − 𝒙𝟐 .

𝑦 = √2𝑥 − 𝑥 2
𝑑𝑦
𝑑√𝑢 𝑑𝑥
From 𝑑𝑥
= 2√𝑢

2𝑑𝑥−2𝑥𝑑𝑥
𝑑𝑦 = Factor out by 2
2√2𝑥−𝑥 2

2(1−𝑥)𝑑𝑥
dy = Simplify
2√2𝑥−𝑥 2

(𝟏−𝒙)𝒅𝒙
dy =
√𝟐𝒙−𝒙𝟐

Example 3. If 𝒚 = 𝒙𝟐 √𝒙𝟒 + 𝟏 , find dy

Given: 𝑦 = 𝑥 2 √𝑥 4 + 1
1
y = 𝑥 2 (𝑥 4 + 1)2 Write in exponential form
1 1
𝑦 = 𝑥 2 𝑑(𝑥 4 + 1) + (𝑥 4 + 1) 𝑑(𝑥 2 )
2 2 Differentiate, apply the chain rule
1 1
1 −
= 𝑥 2 [ (𝑥 4 + 1) 2 (4𝑥 3 𝑑𝑥)] + (𝑥 4 + 1) (2𝑥𝑑𝑥)
2
2
1 1
= [2𝑥 5 (𝑥 4 + 1)− 2 + 2𝑥(𝑥 4 + 1)2 ] 𝑑𝑥 Factor
1
= 2𝑥(𝑥 4 + 1)− 2 (2𝑥 4 + 1)𝑑𝑥 Write in a positive exponent
2𝑥(2𝑥 4 +1) 𝑑𝑥
dy = 1
(𝑥 4 +1)2
𝟐𝒙(𝟐𝒙𝟒 +𝟏)𝒅𝒙
𝒅𝒚 = Write in radical form
√𝒙𝟒 +𝟏

This document is the property of PHINMA EDUCATION


7
MAT 171: Differential Calculus
Student’s Activity Sheets #7

Name: ________________________________________________________ Class number:______


Section: ____________ Schedule:__________________________________ Date: _____________

(𝟏−𝒙)𝟐
Example 4. If 𝒚 = (𝟏−𝟐𝒙)𝟐 ; find 𝒅𝒚

(1−𝑥)2
𝑦 = (1−2𝑥)2
(1−2𝑥)2 𝒅(1−𝑥)2 − (1−𝑥)2 𝒅(1−2𝑥)2
𝑑𝑦 = ((1−2𝑥)2 )2
Apply the quotient rule
(1−2𝑥)2 2(1−𝑥)(−𝑑𝑥) − (1−𝑥)2 [2(1−2𝑥)(−2𝑑𝑥)]
= ((1−2𝑥)2 )2

−2(1−2𝑥)2 (1−𝑥)𝑑𝑥 + 4(1−𝑥)2 (1−2𝑥)𝑑𝑥


= ((1−2𝑥)2 )2

−2(1−𝑥)(1−2𝑥) [(1−2𝑥)−2(1−𝑥)] 𝑑𝑥
= ((1−2𝑥)2 )2
−2(1−𝑥)(1−2𝑥)(1−2𝑥−2+2𝑥)𝑑𝑥
= (1−2𝑥)4
−2(1−𝑥)(1−2𝑥)(−𝑑𝑥)
= (1−2𝑥)4
𝟐(𝟏−𝒙)𝒅𝒙
𝒅𝒚 = (𝟏−𝟐𝒙)𝟑

Example 5. From the implicit function 𝒙𝟐 + 𝒙𝒚 + 𝒚𝟐 = 𝟏𝟎 , Find 𝒅𝒚.

Given: 𝑥 2 + 𝑥𝑦 + 𝑦 2 = 10 Apply implicit differentiation


2𝑥𝑑𝑥 + (𝑥𝑑𝑦 + 𝑦𝑑𝑥) + 2𝑦𝑑𝑦 = 0
2𝑥𝑑𝑥 + 𝑥𝑑𝑦 + 𝑦𝑑𝑥 + 2𝑦𝑑𝑦 = 0
𝑑𝑦(𝑥 + 2𝑦) = (−2𝑥 − 𝑦)𝑑𝑥
(−2𝑥−𝑦)𝑑𝑥
𝑑𝑦 = 𝑥+2𝑦
;

− (𝟐𝒙 + 𝒚)𝒅𝒙
𝒅𝒚 =
𝒙 + 𝟐𝒚
Example 6. Find 𝒅𝒚 if the implicit function is 𝟑𝒚𝟐 + 𝟐𝒚 − 𝟔𝒙𝒚 + 𝟑𝒙𝟐 − 𝟕𝒙 − 𝟏 = 𝟎

Given: 3𝑦 2 + 2𝑦 − 6𝑥𝑦 + 3𝑥 2 − 7𝑥 − 1 = 0 .
6𝑦𝑑𝑦 + 2𝑑𝑦 − 6(𝑥𝑑𝑦 + 𝑦𝑑𝑥) + 6𝑥𝑑𝑥 − 7𝑑𝑥 = 0 Apply implicit differentiation
6𝑦𝑑𝑦 + 2𝑑𝑦 − 6𝑥𝑑𝑦 − 6𝑦𝑑𝑥 + 6𝑥𝑑𝑥 − 7𝑑𝑥 = 0
6𝑦𝑑𝑦 + 2𝑑𝑦 − 6𝑥𝑑𝑦 = +6𝑦𝑑𝑥 − 6𝑥𝑑𝑥 + 7𝑑𝑥
𝑑𝑦(6𝑦 + 2 − 6𝑥) = (6𝑦 − 6𝑥 + 7)𝑑𝑥
(𝟔𝒚−𝟔𝒙+𝟕)𝒅𝒙
𝒅𝒚 = (𝟔𝒚+𝟐−𝟔𝒙)

This document is the property of PHINMA EDUCATION


8
MAT 171: Differential Calculus
Student’s Activity Sheets #7

Name: ________________________________________________________ Class number:______


Section: ____________ Schedule:__________________________________ Date: _____________

2) Activity 3: Skill-building Activities


Solve the
following problems
𝑑𝑦
1. Find , of the implicit function
𝑑𝑥
𝑥 2 + 4𝑥𝑦 + 𝑦 2 = 5
2. From the implicit function of
𝑑𝑦
𝑥 2 + 𝑦 2 = 2𝑥𝑦, find 𝑑𝑥
3. From the implicit function of
𝑑2 𝑦
𝑥 2 + 𝑦 2 = 25, find
𝑑𝑥 2

Find the differential of the given function.


4. 𝑦 = 5𝑥 3 + 62 + 10𝑥 + 5
5. 𝑦 = √4 − 5𝑥
2𝑥 5𝑦
6. 𝑦
− 𝑥
=6

3) Activity 5: Check for Understanding

1. Find the second derivative of the


implicit function √𝑥 + √𝑦 = 25

Find the differential of the given function

2. y = 𝑥 3 - √𝑥
3. 𝑦 = √ (2 − 𝑥)3

C. LESSON WRAP-UP

1) Activity 6: Thinking about Learning


A. Work Tracker
You are done with this session! Let’s track your progress. Shade the session number you just
completed.

This document is the property of PHINMA EDUCATION


9
MAT 171: Differential Calculus
Student’s Activity Sheets #7

Name: ________________________________________________________ Class number:______


Section: ____________ Schedule:__________________________________ Date: _____________

B. Think about your Learning

1. What motivated you to finish the lesson today?


_____________________________________________________________________________
_____________________________________________________________________________

2. What could you have done better to improve your learning today?
_____________________________________________________________________________
_____________________________________________________________________________

FAQs
1. What is implicit differentiation?
Implicit differentiation is the special case of related rates where one of the variables is time. Implicit
differentiation has an important application: it allows to compute the derivatives of inverse functions. It is
good that we review this, because we can use these derivatives to find anti-derivatives.

2. Why is implicit differentiation important?


Implicit differentiation is important because it allows you to find the derivative of y with respect to x without
having to solve the given equation for y. The chain rule must be used whenever the function y is being
differentiated because of our assumption that y may be expressed as a function of x.

3. What is the difference between a derivative and a differential?


A derivative is a change in a function, a differential is the change in a variable. A function is the
relationship between two variables, so the derivative is always a ratio of differential.

4. What is the differential formula?


dy = f' (x) dx

KEY TO CORRECTIONS

−(𝒙+𝟐𝒚) Activity 5.
Activity 3. 1. 𝒚′ = (𝟐𝒙+𝒚) 𝟐𝟓
1. 𝒚′′ =
𝟐(𝒚−𝒙) 𝟐 √ 𝒙𝟑
2. 𝒚′ = 𝟐(𝒚−𝒙) = 𝟏
𝟏
−𝟐𝟓
2. dy = (𝟑𝒙𝟐 − 𝟐 𝒙) 𝒅𝒙

3. 𝒚′′ = 𝒚𝟑

4. 𝒅𝒚 = (𝟏𝟓𝟐 + 𝟏𝟐𝒙 + 𝟏𝟎)𝒅𝒙 𝟑


3. 𝒅𝒚 = − 𝟐 √𝟐 − 𝒙𝒅𝒙
−𝟓𝒅𝒙
5. 𝒅𝒚 =
𝟐√𝟒−𝟓𝒙
(𝟐𝒙𝟐 𝒚+𝟓𝒚𝟑 )𝒅𝒙
6. 𝒅𝒚 = 𝟐𝒙𝟑 +𝟓𝒚𝟐 𝒙

This document is the property of PHINMA EDUCATION


10
MAT 171: Differential Calculus
Students Activity Sheets #8

Name: __________________________________________________________ Class number: _____


Section: ____________ Schedule: ___________________________________ Date: ____________

Lesson Title: Applications of Differentiation Materials: Ballpoint, Notebook,


Calculator
Lesson Objectives:
At the end of the lesson, you should be able to: Textbook: Calculus by Ron Larson
1. Recall and state the differentiation formulas 11th Ed.
1. 2. Apply differentiation formulas in various computation References:
of (a) slope, (b) vertex of parabola, (c) points where the 1. The Calculus 7 by Louis Leithold
tangent is parallel to the x-axis 2. Differential and Integral Calculus
by Clyde E. Love and Earl Rainville

PRACTICE SOLVING REGULARLY WILL HELP YOU TO REMEMBER THE FORMULAS.

A. LESSON PREVIEW/REVIEW
1. Introduction
GOOD DAY! The following topics shall be
accomplished in this lesson:
1. Recall and state the differentiation formulas
2. Apply the differentiation formulas in various
computation of,
2.a slope
2.b vertex of parabola
2.c points where the tangent is
parallel to the x-axis

1) Activity 1: What I Know Chart, part 1

What I Know Questions: What I Learned (Activity 4)


1. Can you state the
differentiation
formulas?

2. Will you be able to


solve differentiation
problems on slope,
vertex of parabola,
and points where the
tangent is parallel to
the x-axis?

This document is the property of PHINMA EDUCATION


1
MAT 171: Differential Calculus
Students Activity Sheets #8

Name: __________________________________________________________ Class number: _____


Section: ____________ Schedule: ___________________________________ Date: ____________

B.MAIN LESSON

In our previous lesson, we learned about the differentiation formulas:

1. CONSTANT RULE: “The Derivative of a Constant is zero”

𝒅𝑪
=𝟎
𝒅𝒙
2. POWER RULE:“ The Derivative of the Power of a Function is equal to the power multiplied by
the function raised to the power minus one”

𝒅 𝒅
𝒙𝒏 = 𝒏𝒙𝒏−𝟏 ; (𝒙) = 𝟏
𝒅𝒙 𝒅𝒙
3. THE CONSTANT MULTIPLE RULE: “The derivative of C multiplied by a power of x is equal
to C multiplied by the derivative of the power x”

𝒅 𝒅
(𝑪𝒙𝒏 ) = 𝑪 (𝒙𝒏 ) = 𝑪 (𝒏𝒙𝒏−𝟏 )
𝒅𝒙 𝒅𝒙
4. THE SUM OR DIFFERENCE RULE: “The Derivative of the Sum or Difference of Two
Functions is equal to the sum or difference of their derivatives”.

𝒅 𝒅𝒖 𝒅𝒗
(𝒖 ± 𝒗) = ±
𝒅𝒙 𝒅𝒙 𝒅𝒙

5. THE QUOTIENT RULE: “The Derivative of the Quotient of Two Functions is equal to the
denominator multiplied by the derivative of the numerator, minus the numerator multiplied by
the derivative of the denominator, all divided by the square of the denominator”

𝒅𝒖 𝒅𝒗
𝒅 𝒖 𝒗 −𝒖
𝒅𝒙 𝒅𝒙
( )= 𝟐
𝒅𝒙 𝒗 𝒗
A very Common Mnemonic for the Quotient Rule is,

𝒉𝒊𝒈𝒉 𝒍𝒐𝒘 𝑫 (𝒉𝒊𝒈𝒉) − 𝒉𝒊𝒈𝒉 𝑫 (𝒍𝒐𝒘)


𝒅𝒙 [ ]=
𝒍𝒐𝒘 𝒍𝒐𝒘 𝒔𝒒𝒖𝒂𝒓𝒆𝒅

This document is the property of PHINMA EDUCATION


2
MAT 171: Differential Calculus
Students Activity Sheets #8

Name: __________________________________________________________ Class number: _____


Section: ____________ Schedule: ___________________________________ Date: ____________

6. PRODUCT RULE “The Derivative of the Product of Two Functions is equal to the first
function multiplied by the derivative of the second function plus the second function multiplied
by the derivative of the first function”

𝒅 𝒅 𝒅
(𝒖𝒗) = 𝒖 +𝒗
𝒅𝒙 𝒅𝒙 𝒅𝒙
Formulas 5 and 6 can be extended to the case where n functions are involved. For three
functions ( say, u, v, w ),
𝒅 𝒅𝒖 𝒅𝒗 𝒅𝒘
𝒖𝒗𝒘 = 𝒗𝒘 + 𝒖𝒘 + 𝒖𝒗
𝒅𝒙 𝒅𝒙 𝒅𝒙 𝒅𝒙

In the special case when u=C, if C is a constant; formulas 5 and 6 become,

𝒅𝒗
𝒅 𝑪 −𝑪𝒅𝒙
𝒅 𝒅𝒗
6.b 𝑪𝒗 = 𝑪 6.c ( )=
𝒅𝒙 𝒅𝒙 𝒅𝒙 𝑽 𝒗𝟐

The Power Rule


Derivative of 𝒙𝒏 , (The Power Rule)

𝒅𝒚
If 𝒚 = 𝒙𝒏 , then = 𝒏𝒙𝒏−𝟏
𝒅𝒙

The Chain Rule


If y is a differentiable function of u given by y = f(u) and if u is a differentiable
function of x given by u = g (x), the y is a differentiable function of x and we have,

𝒅𝒚 𝒅𝒚 𝒅𝒖
=( )( )
𝒅𝒙 𝒅𝒖 𝒅𝒙

The General Power Formula


𝒅 𝒅𝒖
un = n un-1
𝒅𝒙 𝒅𝒙

This document is the property of PHINMA EDUCATION


3
MAT 171: Differential Calculus
Students Activity Sheets #8

Name: __________________________________________________________ Class number: _____


Section: ____________ Schedule: ___________________________________ Date: ____________

FINDING THE SLOPE OF THE CURVE


Let us continue our lesson on how to apply differentiation formulas in various computation.
Examples are given for easy comprehension and understanding.

Example 1. Find the slope of the curve 𝒚 = 𝟑 + 𝟐𝒙 − 𝒙𝟐 at (𝟏, 𝟒)

Solution: Recall the derivative of a function is identical with the slope of the graph of the
function.
𝑦
2
𝑦 = 3 + 2𝑥 − 𝑥
(𝟏, 𝟒) y’ = 𝟎
Find the derivative

𝑑𝑦
𝑑𝑥
= 2 − 2𝑥

at (𝟏, 𝟒) substitute 𝒙 = 𝟏
= 2 − 2(1) 𝒚 = 𝟑 + 𝟐𝒙 − 𝒙𝟐

𝒅𝒚
𝒅𝒙
=𝟎
𝒙

Therefore, the slope of the curve


at (𝟏, 𝟒) is 0.

Example 2. Find the slope of the curve 𝒚 = 𝒙𝟑 at (2, 8)

Given: 𝑦 = 𝑥 3 at (2, 8) 𝒚
(2, 8)

Solution: 𝑦 = 𝑥3

find the derivative


𝑑𝑦
𝑑𝑥
= 3𝑥 2
y’= 𝟏𝟐

at (2, 8), let 𝒙 = 𝟐


= 3(2)2 𝒚 = 𝒙𝟑

𝒅𝒚
𝒅𝒙
= 𝟏𝟐
𝒙
Therefore, the slope of the curve at (2, 8) is 12.

This document is the property of PHINMA EDUCATION


4
MAT 171: Differential Calculus
Students Activity Sheets #8

Name: __________________________________________________________ Class number: _____


Section: ____________ Schedule: ___________________________________ Date: ____________

Example 3. Find the slope of the tangent to the curve


𝒚 = 𝟕 − 𝒙𝟐 + 𝟒𝒙𝟑 at (−𝟏, 𝟐) 𝒚

Solution: 𝑦 = 7 − 𝑥 2 + 4𝑥 3 y’= 𝟏𝟒

Differentiate
𝒚 = 𝟕 − 𝒙𝟐 + 𝟒𝒙𝟑
𝑑𝑦
𝑑𝑥
= −2𝑥 + 12𝑥 2

at (−𝟏, 𝟐), substitute 𝒙 = −𝟏


= −2(−1) + 12(−1)2
(−𝟏, 𝟐)
= 2 + 12

𝒅𝒚
𝒅𝒙
= 𝟏𝟒 𝒙

Thus, the slope of the curve at (−𝟏, 𝟐) is 14.

Example 4. Find the slope of the tangent line to the curve of 𝒙𝟐 𝒚𝟐 − 𝒙𝒚 + 𝒙 = 𝟏 at (1, 1)
Given: 𝑥 2 𝑦 2 − 𝑥𝑦 + 𝑥 = 1
𝑑 𝑑
(𝑥 2 𝑦 2 − 𝑥𝑦 + 𝑥) = (1)
𝑑𝑥 𝑑𝑥
[𝑥 2 (2𝑦𝑦’) + 𝑦 2 (2𝑥)] − [𝑥𝑦’ + 𝑦(1)] + 1 = 0
2𝑥 2 𝑦𝑦’ + 2𝑥𝑦 2 − 𝑥𝑦’ − 𝑦 + 1 = 0
2𝑥 2 𝑦𝑦’ + 𝑥𝑦’ = 𝑦 − 2𝑥𝑦 2 − 1
𝑦−2𝑥𝑦 2 −1
𝑦′ = 2𝑥 2 𝑦−𝑥
at (1, 1), substitute x = 1,y = 1
1−2(1)(1)−1 −2
𝒚′ = 2(1)(1)−1
= 1
= −𝟐
Therefore, the slope of the tangent line is -2.

Example 5. Find the slope of the curve 𝒚𝟑 = 𝒙𝟐 − 𝟏 , at the point (3, 2)


𝑦3 = 𝑥2 − 1
3𝑦 2 𝑦’ = 2𝑥
2𝑥
𝑦′ = 3𝑦2 at (3, 2), x = 3, y = 2
2(3)
𝑦 ′ = 3(4)
𝟏
𝒚′ = 𝟐

This document is the property of PHINMA EDUCATION


5
MAT 171: Differential Calculus
Students Activity Sheets #8

Name: __________________________________________________________ Class number: _____


Section: ____________ Schedule: ___________________________________ Date: ____________

FINDING THE VERTEX OF A PARABOLA

Example 1 Find the vertex of the parabola 𝒚 = 𝒙𝟐 + 𝟔𝒙 + 𝟕

By means of derivative, check the equation of the parabola by completing the square and
transform to the standard form.
𝒅𝒚
Note: If the axis of the parabola is vertical, its slope at the vertex 𝒅𝒙 = 𝟎; if the axis is
𝒅𝒚
horizontal, the slope of the vertex is 𝒅𝒙 = ∞.

Given: 𝑦 = 𝑥 2 + 6𝑥 + 7

The vertex of the parabola (refer to the graph) lies at a point on the horizontal tangent line of the
curve, that is, the slope 𝑦 ′ = 0.

Solution: 𝑦 = 𝑥 2 + 6𝑥 + 7
differentiate
𝑑𝑦
𝑑𝑥
= 2𝑥 + 6 𝒚

𝑑𝑦
But, = 0,
𝑑𝑥
0 = 2𝑥 + 6 𝒚 = 𝒙𝟐 + 𝟔𝒙 + 𝟕
2𝑥 = −6
𝒙 = −𝟑
From, 𝑦 = 𝑥 2 + 6𝑥 + 7
Substitute 𝒙 = −𝟑 and solve for y
𝑦 = (−3)2 + 6(−3) + 7
𝑦 = 9 − 18 + 7 𝒙
𝒚 = −𝟐
Thus, the vertex of the parabola is 𝑽(−𝟑, −𝟐)

Check by Completing the square: 𝒚′ = 𝟎


𝑦 = 𝑥 2 + 6𝑥 + 7 V (-3, -2)
2
𝑥 + 6𝑥 = 𝑦 − 7
arrange in the form of completing the square
𝑥 2 + 6𝑥 + 9 = 𝑦 − 7 + 9
From the standard equation of the parabola, (𝒙 − 𝒉)𝟐 = 𝟒𝒂(𝒚 − 𝒌), where the
vertex (V) is (h, k)
(𝑥 + 3)2 = (𝑦 + 2)
Therefore, the vertex of the parabola is 𝑽(−𝟑, −𝟐)

This document is the property of PHINMA EDUCATION


6
MAT 171: Differential Calculus
Students Activity Sheets #8

Name: __________________________________________________________ Class number: _____


Section: ____________ Schedule: ___________________________________ Date: ____________

Example 3.2 Find the vertex of the parabola


𝒚 = 𝒙𝟐 − 𝟐𝒙

Given: 𝑦 = 𝑥 2 − 2𝑥
𝑑𝑦
Solution: 𝑑𝑥
= 2𝑥 − 2
𝑑𝑦
But, 𝑑𝑥
= 0,
0 = 2𝑥 − 2
2𝑥 = 2
𝒙=𝟏

From, 𝑦 = 𝑥 2 − 2𝑥
Substitute 𝒙 = 𝟏 and solve for y
𝑦 = (1)2 + 6(1)
𝑦 = 1−2
𝒚 = −𝟏
Thus, the vertex of the parabola is 𝑽(𝟏, −𝟏)

Check by Completing the square: 𝑦 = 𝑥 2 − 2𝑥

𝑥 2 − 2𝑥 = 𝑦 arrange in the form of completing the square


𝑥 2 − 2𝑥 + 1 = 𝑦 + 1

From the standard equation of the parabola, (𝒙 − 𝒉)𝟐 = 𝟒𝒂(𝒚 − 𝒌), where the
vertex (V) is (h, k)
(𝑥 − 1)2 = (𝑦 + 1)

Therefore, the vertex of the parabola is 𝑽(𝟏, −𝟏)

This document is the property of PHINMA EDUCATION


7
MAT 171: Differential Calculus
Students Activity Sheets #8

Name: __________________________________________________________ Class number: _____


Section: ____________ Schedule: ___________________________________ Date: ____________

EXAMPLE 4. Finding the points where the tangent is


parallel to the x-axis

Example 4.1 𝑦 = 𝑥 3 + 6𝑥 2 + 9𝑥 + 7

𝑑𝑦
Differentiate, solve for 𝑑𝑥

𝑑𝑦
𝑑𝑥
= 3𝑥 2 + 12𝑥 + 9
Note: when the tangent is parallel
𝑑𝑦
to Ox, 𝑑𝑥
=0

0 = 3𝑥 2 + 12𝑥 + 9

Divide both sides by 3

𝑥 2 + 4𝑥 + 3 = 0

Factor

(𝑥 + 3)(𝑥 + 1) = 0

Solve for x

𝒙 = −𝟑, 𝒙 = −𝟏

When 𝑥 = −3 When 𝑥 = −1
From: 𝑦 = 𝑥 3 + 6𝑥 2 + 9𝑥 + 7 From: 𝑦 = 𝑥 3 + 6𝑥 2 + 9𝑥 + 7

Substitute 𝑥 = −3 Substitute 𝑥 = −1
y= (−3)3 + 6(−3)2 + 9(−3) + 7 y= (−1)3 + 6(−1)2 + 9(−1) + 7

= −27 + 54 − 27 + 7 = −1 + 6 − 9 + 7
𝒚=𝟕 𝒚=𝟑

Thus the points where the tangent is parallel to x-axis are, (-3, 7), (−𝟏, 𝟑)

This document is the property of PHINMA EDUCATION


8
MAT 171: Differential Calculus
Students Activity Sheets #8

Name: __________________________________________________________ Class number: _____


Section: ____________ Schedule: ___________________________________ Date: ____________

1) Activity 3: Skill-building Activities (with answer key)

Find the SLOPE, the VERTEX of the parabola, and the


POINT where the tangent is parallel to the x-axis

1. Find the slope of 𝑦 = 2 − 𝑥 2 at the point (3, −7)

2. Find the slope of 𝑦 = 3𝑥 2 − 2𝑥 at the point (1, 1)


1
3. Find the vertex of the parabola 𝑦 = (𝑥 2 + 8)
8

4. Find the vertex of the parabola 𝑦 = 2𝑥 2 + 4𝑥 + 5

5. Find the vertex of the parabola 𝑦 = 4𝑥 2 − 4𝑥 + 3

6. Find the point where the tangent is parallel to x-axis, 𝑦 = 2𝑥 3 − 3𝑥 2 − 12𝑥 + 6

2) Activity 4: What I Know Chart, part 2


What I Learned
1. ______________________________________________________________________________
______________________________________________________________________________

2. ______________________________________________________________________________
______________________________________________________________________________

3. ______________________________________________________________________________
______________________________________________________________________________

3) Activity 5: Check for Understanding

𝟏
1. Find the slope of 𝒚 = 𝒙−𝟏 at (𝟐, 𝟏)

This document is the property of PHINMA EDUCATION


9
MAT 171: Differential Calculus
Students Activity Sheets #8

Name: __________________________________________________________ Class number: _____


Section: ____________ Schedule: ___________________________________ Date: ____________

C. LESSON WRAP-UP
1) Activity 6: Thinking about Learning

Work Tracker
You are done with this session! Let’s track your progress. Shade the session number you just
completed.

A. T
Think about your Learning

1. What motivated you to finish the lesson today?


_____________________________________________________________________________
_____________________________________________________________________________

2. What could you have done better to improve your learning today?
_____________________________________________________________________________
_____________________________________________________________________________

FAQs
1. Why do we differentiate a function?
Differentiation allows us to find rates of change. For example, it allows us to find the rate
of change of velocity with respect to time (which is acceleration). It also allows us to find the rate
of change of x with respect to y, which on a graph of y against x is the gradient of the curve.

2. What is differentiation in simple words


Differentiation is a process of finding a function that outputs the rate of change of one
variable with respect to another variable.

KEY TO CORRECTIONS
Activity 3: 1. -6
2. 4
3. (0, 1)
4. (-1, 3)
𝟏
5 . ( 𝟐, 2)
6. (2, −𝟏𝟒) and (−𝟏, 13)

Activity 5: 1. −𝟏

This document is the property of PHINMA EDUCATION


10
MAT 171: Differential Calculus
Students Activity Sheets #9

Name: _______________________________________________________ Class number: _______


Section: ____________ Schedule: ________________________________ Date: ______________

Lesson Title: Polynomial Curves: Materials: Ballpoint, Notebook, Calculator


Tangent and Normal lines
First Derivative Test Textbook: Calculus by Ron Larson, 11th Ed.
Lesson Objectives: References:
At the end of the lesson, you should be able to: 1. The Calculus 7 by Louis Leithold
1. Relate the significance of the slope with the 2. Differential and Integral Calculus
equation of the tangent and normal lines by Clyde E. Love and Earl Rainville
1. 2. Find the tangent and normal lines to a curve at a 3. Calculus by James Stewart, 7th Ed.
given point.
3. Define and illustrate geometrically increasing
and decreasing functions
2. 4. Apply the first derivative test to find relative
extremes of a function

REMEMBER KEY TERMS AND CONCEPTS!


FOCUS AND MANAGE TASKS
A. LESSON PREVIEW/REVIEW

1) Introduction
GOOD DAY! The following topics shall be accomplished
in this lesson:
1) Find the tangent and normal lines to a curve at a given point
2) Define and illustrate geometrically increasing
and decreasing functions
3) Apply the first derivative test to find relative
extremes of a function

2) Activity 1: What I Know Chart, part 1

What I Know Questions: What I Learned (Activity 4)


1 Can you find the tangent to a
curve at a given point?

2 How do you find the normal line


to the curve at a given point?

1. Can you define and illustrate


increasing and decreasing
functions?
2. Do you know how to apply the
first derivative test to find the
relative extremes of a function?

This document is the property of PHINMA EDUCATION


1
MAT 171: Differential Calculus
Students Activity Sheets #9

Name: _______________________________________________________ Class number: _______


Section: ____________ Schedule: ________________________________ Date: ______________

B.MAIN LESSON
1) Activity 2: Content Notes

POLYNOMIAL CURVES: TANGENT AND NORMAL LINE


Equations of Tangent and Normal to Plane Curves

 We have seen that the derivative of a function can be interpreted as the slope of the
tangent to the graph of the function.
 In the figure below, the line T is the tangent to the curve y = f(x) at P1(x1, y1,).
 The other line N perpendicular to T at P1(x1, y1) is called the normal to the curve.

 The tangent to the curve y = f(x) at P1(x1, y1) is the line through P1 with slope y’ = f ‘(x) = m.

 The normal to the curve y =f(x) at P1(x1, y1) is the line through P1 and perpendicular to the
tangent at P1.

 The equation of the tangent line is given by the point-slope form of the equation of the
straight line in Analytic Geometry,

y – y1 = m(x – x1)

 Where m = value of y' at x = x1 or m = f '(x). Since the normal is perpendicular to the


tangent, then its slope is negative reciprocal of the slope of the tangent. Hence, the
equation of the normal is,

𝟏
y – y1 = - (𝒙 − 𝒙𝟏 )
𝒎

This document is the property of PHINMA EDUCATION


2
MAT 171: Differential Calculus
Students Activity Sheets #9

Name: _______________________________________________________ Class number: _______


Section: ____________ Schedule: ________________________________ Date: ______________

Example 1. Find the equation of the (a) tangent and (b) normal lines to the parabola y = x2 – 1
at the point (2, 3)
Solution:
Compute the slope of the tangent at line (2, 3) with 𝑦 = 𝑥 2 − 1. 𝑦′ = 2𝑥

a) Solve for the tangent line


The point of tangency is (2, 3). Hence x1 = 2 and y1= 3. Since y' = 2x, then y' = 2(2) = 4
Thus, the tangent line at ( 2, 3) has slope 4. From the point-slope form of the equation of a line,
y – y1 = m(x – x1), we have,

𝑦 − 3 = 4(𝑥 − 2) substitute
𝑦 − 3 = 4𝑥 − 8 simplify
𝟒𝒙 − 𝒚 − 𝟓 = 𝟎 The equation of the tangent line.

b) Solve for the normal line


The normal line to the graph at (2, 3) is perpendicular to the tangent line at that point. Because
1
the slope of the tangent line at ( 2, 3) is 4, then the slope of the normal line at ( 2, 3) is − .
4
From the point-slope form of the equation of a line, y-y1 = m( x-x1)
1
The equation of the normal line is, 𝑦 − 3 = − 4 (𝑥 − 2)
4𝑦 − 12 = −𝑥 + 2
𝒙 + 𝟒𝒚 − 𝟏𝟒 = 𝟎 The equation of the normal line

This document is the property of PHINMA EDUCATION


3
MAT 171: Differential Calculus
Students Activity Sheets #9

Name: _______________________________________________________ Class number: _______


Section: ____________ Schedule: ________________________________ Date: ______________

Example 2. Find the equation of the tangent and normal lines to the ellipse 4𝑥 2 + 9𝑦 2 = 25
at (−2, −1).
Differentiate both members of the equation of the ellipse
4𝑥 2 + 9𝑦 2 = 25 implicit function
8𝑥 + 18𝑦𝑦′ = 0 apply implicit differentiation
−8𝑥
𝑦′ = 18𝑦
−4𝑥
𝑦′ = 9𝑦
at the point of tangency (-2, -1), x has the value -2, y has the value -1, and we shall
denote the slope by m.

−4(−2) 8 8
𝑦′ = 9(−1)
= − 9, 𝑚 = −9

a) Thus the equation of the tangent line is, b. The equation of the normal line at (−2, −1)
8
𝑦 − 𝑦1 = 𝑚(𝑥 − 𝑥1 ) point-slope form from mTL = - 9
8 9
𝑦 + 1 = − 9 (𝑥 + 2) substitute therefore, mNL =
8
9𝑦 + 9 = −8𝑥 − 16 simplify
𝑦 − 𝑦 ′ = 𝑚𝑁𝐿 (𝑥 − 𝑥1 )
8𝑥 + 9𝑦 + 25 = 0 the equation of tangent line 9
𝑦 + 1 = (𝑥 + 2)
8
1 8𝑦 + 8 = 9𝑥 + 18
𝑇𝐿 ⊥ 𝑁𝐿 , 𝑚 𝑇𝐿 = −
𝑚𝑁𝐿
𝟗𝒙 − 𝟖𝒚 + 𝟏𝟎 = 𝟎 equation of the normal line.

4𝑥 2 + 9𝑦 2 = 25

This document is the property of PHINMA EDUCATION


4
MAT 171: Differential Calculus
Students Activity Sheets #9

Name: _______________________________________________________ Class number: _______


Section: ____________ Schedule: ________________________________ Date: ______________

Example 3. Find the tangent of the slope 2 to the circle 𝒙𝟐 + 𝒚𝟐 = 𝟓 .

From the equation of the circle


𝑥 2 + 𝑦 2 = 5 →1
Differentiate both members of the equation
2𝑥 + 2𝑦𝑦′ = 0
𝑥 + 𝑦𝑦′ = 0 The slope of the tangent line is 2. Let y' = 2
𝑥 + 2𝑦 = 0 →2
Therefore, the coordinates of the point of contact must satisfy the equation 𝑥 + 2𝑦 = 0 , must
also satisfy the equation of the original curve, 𝑥 2 + 𝑦 2 = 5 . By solving the simultaneous
equation 1 & 2, we determine the points of contact or call this as points of tangency.
𝑥 2 + 𝑦 2 = 5 →1 𝑥 + 2𝑦 = 0 →2
𝑥 = −2𝑦
equate 1 & 2
(−2𝑦)2 + 𝑦 2 = 5
4𝑦 2 + 𝑦 2 = 5
5𝑦 2 = 5
𝑦2 = 1
𝑦 = ±1
from 2
𝑥 = −2𝑦 when y = 1
𝑥 = −2(1)
𝑥 = −2
Hence, the point of tangency is (-2, 1)
from 2, when y = -1
𝑥 = −2𝑦
𝑥 = −2(−1)
𝑥=2
Another point of tangency is (2,-1)
at (-2, 1) m = 2
𝑦 − 𝑦1 = 𝑚(𝑥 − 𝑥1 ) from point-slope form
𝑦 − 1 = 2(𝑥 + 2)
𝑦 − 1 = 2𝑥 + 4
𝟐𝒙 − 𝒚 = −𝟓 equation of tangent line
at (2,-1) m = 2
𝑦 − 𝑦1 = 𝑚(𝑥 − 𝑥1 ) from point – slope form
𝑦 + 1 = 2(𝑥 − 2)
𝑦 + 1 = 2𝑥 − 4
𝟐𝒙 − 𝒚 = 𝟓 The equation of the tangent line

This document is the property of PHINMA EDUCATION


2
MAT 171: Differential Calculus
Students Activity Sheets #9

Name: _______________________________________________________ Class number: _______


Section: ____________ Schedule: ________________________________ Date: ______________

Example 4. Find the equation of tangent lines to the ellipse 𝒙𝟐 + 𝟒𝒚𝟐 = 𝟖 parallel to the
line 𝒙 + 𝟐𝒚 = 𝟔

From the equation of the ellipse: 𝑥 2 + 4𝑦 2 = 8 ,

Differentiate 𝑥 2 + 4𝑦 2 = 8
we have, 2𝑥 + 8𝑦𝑦′ = 0
−2𝑥
𝑦′ =
8𝑦
𝑥
or 𝑦′ = − 4𝑦

From the equation of the given line 𝑥 + 2𝑦 = 6.

Differentiate 𝑥 + 2𝑦 = 6
1 + 2𝑦’ = 0
1
𝑦′ = − 2

1
Thus, the slope of the given line is - 2 , or we can also use the “ slope-intercept form” of
the equation of the line 𝑦 = 𝑚𝑥 + 𝑏.

Where the coefficient of x is the slope of the line.

𝑥 + 2𝑦 = 6
2𝑦 = −𝑥 + 6 Theorem of analytic geometry
1
𝑦 = −2𝑥 + 3 L1 and L2 are parallel if and only if m1=m2
1
𝑚 = −2

Since the slope of the tangent lines at the point of tangency must be equal to that of the
given line for these lines to be parallel, then
𝑥 1
− =−
4𝑦 2
𝑥 = 2𝑦

Substituting 𝑥 = 2𝑦 in the equation of the ellipse 𝑥 2 + 4𝑦 2 = 8, 𝑤𝑒 𝑔𝑒𝑡

4𝑦 2 + 4𝑦 2 = 8,
8𝑦 2 = 8
𝑦2 = 1
𝑦 = ±1

This document is the property of PHINMA EDUCATION


3
MAT 171: Differential Calculus
Students Activity Sheets #9

Name: _______________________________________________________ Class number: _______


Section: ____________ Schedule: ________________________________ Date: ______________

when y = 1, solve for x, x = 2y


x=2(1) = 2 at ( 2, 1 )

when y = -1, solve for x, x = 2y


x=2(-1) = -2 at ( -2, -1 )

and so, the points of tangency are ( 2, 1) and ( -2, -1). By the point-slope form of the equation
of a line 𝑦 − 𝑦1 = 𝑚(𝑥 − 𝑥1 )

The equation of tangent lines are:


1
when m = - at ( 2, 1 )
2
1
𝑦 − 1 = − (𝑥 − 2)
2
2𝑦 − 2 = −𝑥 + 2
𝑥 + 2𝑦 = 4

at (-2, -1)
1
𝑦 + 1 = − 2 (𝑥 + 2)
2𝑦 + 2 = −𝑥 − 2
𝑥 + 2𝑦 = −4

This document is the property of PHINMA EDUCATION


4
MAT 171: Differential Calculus
Students Activity Sheets #9

Name: _______________________________________________________ Class number: _______


Section: ____________ Schedule: ________________________________ Date: ______________

Example 5. Find the point where the tangent to the curve 𝒙𝟑 + 𝟔𝒙𝟐 + 𝟗𝒙 + 𝟕
is parallel to the x – axis.
𝑦 = 𝑥 3 + 6𝑥 2 + 9𝑥 + 7
𝑦’ = 3𝑥 2 + 12𝑥 + 9 = 0 Set y' = 0
𝑥 2 + 4𝑥 + 3 = 0
(𝑥 + 3)(𝑥 + 1) = 0 By factoring
𝑥 = −3 𝑥 = −1 By Factor Theorem
when x= -3,
Solve for y from the given equation 𝑦 = 𝑥 3 + 6𝑥 2 + 9𝑥 + 7
𝑦 = (−3)3 + 6(−3)2 + 9(−3) + 7
𝑦 =7, (−3, 7)

when x = -1 𝑦 = (−1)3 + 6(−1)2 + 9(−1) + 7


𝑦 = −1 + 6 − 9 + 7
𝑦 = 3, (−1, 3)

Thus the points where the horizontal tangent line to the curve
are (-1, 3) and ( -3, 7)

← 𝑦 = 𝑥 3 + 6𝑥 2 + 9𝑥 + 7

This document is the property of PHINMA EDUCATION


5
MAT 171: Differential Calculus
Students Activity Sheets #9

Name: _______________________________________________________ Class number: _______


Section: ____________ Schedule: ________________________________ Date: ______________

POLYNOMIAL CURVES: MAXIMA AND MINIMA, & FIRST DERIVATIVE TEST


This section attempts to give stronger method for obtaining graphs by using the first and
second derivatives. Specifically, most of the graphs are confined with polynomials and rational
fractions.

In sketching the graph of a function, we usually rely on plotting of points. Although this process
is useful, the graph fails to exhibit distinct properties of a function.

Increasing and Decreasing Functions.

When a point moves along a curve of a function, say from left to right, the curve either rises or
falls. Consider the curve y=f(x) in figure below. In the interval [a, b], the value of y increases as
the value of x increases and consequently the curve rises along the arc AB.

In the interval [b, c], y decreases as x increases and hence the curve falls along the arc BC.
The function y=f(x) is said to be increasing in the interval [a, b] and likewise said to be
decreasing in the interval [b, c].

Figure 1:

𝒚 = 𝒇(𝒙) →

If the curve rises on certain interval, the slope y' is positive as in the arc AB. If it falls, the slope y'
is negative as in arc BC. That is,

If y' > 0, f(x) increases as x increases


If y' < 0, f(x) decreases as x increases

This document is the property of PHINMA EDUCATION


6
MAT 171: Differential Calculus
Students Activity Sheets #9

Name: _______________________________________________________ Class number: _______


Section: ____________ Schedule: ________________________________ Date: ______________

Maxima and Minima

The point where the curve is


higher than some neighboring
point is called the maximum
point and the function is said
to have a maximum value.

We have the same definition


for the minimum point and the
minimum value of the function.
Point B is a maximum point,
while the point C is the
minimum point as indicated
in the graph, both are obtained
at the horizontal tangent line
(tangent is parallel to OX).

Note: We have the horizontal tangent line y' = 0 at the maximum or minimum points B and C.

The vanishing of the derivative


does not mean that the function
attains maxima or minima.

For instance, the function f(x) =


x3 is strictly monotonic on the
entire real number line because
it is increasing on the entire real
line, as shown in figure 3.

A function is strictly
monotonic on an interval when
it is either increasing on the
entire interval or decreasing on
the entire interval.

This document is the property of PHINMA EDUCATION


7
MAT 171: Differential Calculus
Students Activity Sheets #9

Name: _______________________________________________________ Class number: _______


Section: ____________ Schedule: ________________________________ Date: ______________

The First Derivative Test for the Extremes of the function

At the point where y' = 0, if y' changes from positive to negative (as x increases), y is a maximum. If y'
changes negative to positive (as x increases), y is a minimum; if y' does not change sign, y is neither a
maximum nor a minimum.

𝒇ꞌ(𝒙) > 𝟎 𝒇ꞌ(𝒙) < 𝟎 𝒇ꞌ(𝒙) < 𝟎 𝒇ꞌ(𝒙) > 𝟎

Figure 4 Figure 5

The point at which y' = 0 are called critical points, and the corresponding values of x are the critical
values of x. In figure 2: B,C,D are the critical points. Maxima and minima collectively are called
extremes. In figure 2: B and D are extremes. An extreme is merely greater or less than any
neighboring values.

The following steps can be used to find the maximum or minimum


values of a function f(x).

1. Solve for f '(x) = 0 for the critical values.


2. Locate the critical values on the x axis, thereby establishing a
number of intervals.
3. Determine the sign of f '(x) on each interval.
4. Let x increase through each critical values, then,
(a) f(x) has a maximum value if f '(x) changes from (+) to (–) .
(b) f(x) has a minimum values if f '(x) changes from (–) to (+).

This document is the property of PHINMA EDUCATION


8
MAT 171: Differential Calculus
Students Activity Sheets #9

Name: _______________________________________________________ Class number: _______


Section: ____________ Schedule: ________________________________ Date: ______________

Example 1. Locate and classify the critical points by First Derivative Test.

𝟑
Given: 𝒚 = 𝒙𝟑 + 𝒙𝟐 − 𝟔𝒙 − 𝟒
𝟐
Solution: Write the original function
3
𝑦 = 𝑥 3 + 𝑥 2 − 6𝑥 − 4
2
𝑑𝑦 𝑑 3
Differentiate 𝑑𝑥
= 𝑑𝑥
(𝑥 + 2 𝑥 2 − 6𝑥
3
−4)
′ 𝑑𝑦
y = 𝑑𝑥
= 3𝑥 2 + 3𝑥 − 6
Factor out 3 y ′ = 3(𝑥 + 𝑥 − 2)
2
𝑑𝑦
Set 𝑑𝑥
= y′ = 0 y ′ = 3(𝑥 + 2)(𝑥 − 1)
Solve for x 3(𝑥 + 2)(𝑥 − 1) = 0
x = -2, x = 1
Thus, x = -2 and x =1 are the critical values of x
Solve y
When x = - 2
3
Write the original function 𝑦 = 𝑥 3 + 𝑥 2 − 6𝑥 − 4
2
3
Substitute -2 in x y = (-2)3 +2 (-2)2 – 6(-2) – 4
Simplify y=6
Therefore, (-2, 6) is a critical point

When x = 1 Write original function


3 2
𝑦 = 𝑥3 + 𝑥 − 6𝑥 − 4
2
3
Substitute x = 1 y = (1)3 +2 (1)2 – 6(1) - 4
15
y=-
2
𝟏𝟓
Therefore (1, - 𝟐
) is another critical point

To determine the relative extremes of the curve, test one value to the left of (near but slightly
less than) one value to the right of (near but slightly greater than) the critical number. Using the
neighboring points, we have,

For x = -2 , the neighboring points are : x = -3, x = -1


Let x = -3 Substitute this in the factored form of the derivative
y ′ = 3(x+2)(x-1)
= 3(-3+2)(-3-1)
= +12
Let x = -1
y ' = 3(-1+2)(-1 - 1)
y ' = -6

This document is the property of PHINMA EDUCATION


9
MAT 171: Differential Calculus
Students Activity Sheets #9

Name: _______________________________________________________ Class number: _______


Section: ____________ Schedule: ________________________________ Date: ______________

Since y ' changes sign from positive to negative, then (-2, 6) is maximum point

For x = 1, the neighbourhood points are x = 0, x=2


Let x = 0 y ' = 3(x+2)(x-1)
= 3(2)(-1)
= -6

Let x = 2 y ' = 3(2+2)(2-1)


=+12
𝟏𝟓
Since y ' changes sign from negative to positive, then (1, - 𝟐
) is a minimum point

This document is the property of PHINMA EDUCATION


10
MAT 171: Differential Calculus
Students Activity Sheets #9

Name: _______________________________________________________ Class number: _______


Section: ____________ Schedule: ________________________________ Date: ______________

2) Activity 3: Skill-building Activities (with answer key)

Find the equations of the tangent and


the normal at the point indicated

1. 𝑦 = 𝑥 3 − 3𝑥 2 − 2 𝑎𝑡 ( 1, −4)

2. 𝑦 = 𝑥 3 + 𝑥 𝑎𝑡 (−1, −2)

3. 𝑦 = 2 + 4𝑥 − 𝑥 3 𝑎𝑡 𝑥 = −1

4. 𝑥 2 + 𝑦 2 − 6𝑥 + 2𝑦 = 0 𝑎𝑡 ( 0, 0)

5. Find the equation of the tangent line


to the ellipse 𝑥 2 + 2𝑦 2 = 9
perpendicular to the line 4𝑥 − 𝑦 = 6 .
1 1
6. 1. If y = 3 x3 + 2 x2 - 6x + 8

Find a) the critical points


b) the maximum and minimum
values of y by First Derivative Test

7. Locate and classify the critical points


by the First Derivative Test.
Given y = x3 – 3x2 – 9x + 20

3) Activity 4: What I Know Chart, part 2


What I learned
1. ______________________________________________________________________________
______________________________________________________________________________
2. ______________________________________________________________________________
______________________________________________________________________________
3. ______________________________________________________________________________
______________________________________________________________________________

4) Activity 5: Check for Understanding.


1. Find the equation of the tangent and normal at the point indicated.
𝑦 = 𝑥 − 𝑥 3 𝑎𝑡 (1,0)

2. Locate the critical points and determine the maxima and minima by first derivative test.
Given: y = 4 – 6x + x2

This document is the property of PHINMA EDUCATION


11
MAT 171: Differential Calculus
Students Activity Sheets #9

Name: _______________________________________________________ Class number: _______


Section: ____________ Schedule: ________________________________ Date: ______________

C. LESSON WRAP-UP
1) Activity 6: Thinking about Learning
Work Tracker
You are done with this session! Let’s track your progress. Shade the session number you just
completed.

B. Think about your Learning


1. What motivated you to finish the lesson today?
_____________________________________________________________________________
_____________________________________________________________________________
2. What could you have done better to improve your learning today?
_____________________________________________________________________________
_____________________________________________________________________________
FAQs
1 Is the slope of a tangent line the derivative of a function? The derivative of a function is
a slope. ... This is the same as saying that the derivative is the slope of the tangent line to the graph of
the function at the given point. The slope of a secant line (line connecting two points on a graph)
approaches the derivative when the interval between the points shrinks down to zero.

2. What is a tangent in a circle? A tangent to a circle is a straight line which touches


the circle at only one point. This point is called the point of tangency. The tangent to a circle is
perpendicular to the radius at the point of tangency.

3. What are relative maxima and minima?

Finding relative maxima and minima of a function can be done by looking at a


graph of the function. A relative maximum is a point that is higher than the points
directly beside it on both sides, and a relative minimum is a point that is lower than the
points directly beside it on both sides.

4. Can there be two relative maximums?

A relative max/min point is a point higher or lower than the points on both of its
sides while a global max/min point is a point that is highest or lowest point in the graph.
In other words, there can be multiple relative max/min points while there can only be
one global/absolute max/min point.

This document is the property of PHINMA EDUCATION


12
MAT 171: Differential Calculus
Students Activity Sheets #9

Name: _______________________________________________________ Class number: _______


Section: ____________ Schedule: ________________________________ Date: ______________

KEY TO CORRECTIONS
Answers to activity 3.
1. 𝟑𝒙 + 𝒚 = −𝟏 T.L.
𝒙 − 𝟑𝒚 = 𝟏𝟑 N.L.
2. x+𝟒𝒚 = −𝟗 N.L.
𝟒𝒙 − 𝒚 = −𝟐 T.L.
3 𝒙 − 𝒚 = 𝟎 T.L.
𝒙 + 𝒚 = −𝟐 N.L.
4. 𝟑𝒙 − 𝒚 = 𝟎 TL
𝒙 + 𝟑𝒚 = 𝟎 N.L.
5. 𝒙 + 𝟒𝒚 = 𝟗 T.L.
𝒙 + 𝟒𝒚 = −𝟗 T.L.
𝟒𝟑 𝟐
6. The critical points are (-3, 𝟐
) and (2, 𝟑 ).
𝟒𝟑
(-3, 𝟐
) is a maximum.
𝟐
(2, 𝟑
) is a minimum.
7. (3, -7) CP
(-1, 25) CP
(-1, 25) maximum
( 3, -7) minimum

Activity 5.
1. 𝟐𝒙 + 𝒚 = 𝟐 T.L.
𝒙 − 𝟐𝒚 = 𝟏 N.L.

2. Thus C.P is ( 3, -5)


(3, -5) is the minimum

This document is the property of PHINMA EDUCATION


13
MAT 171: Differential Calculus
Student’s Activity Sheets #10

Name: ______________________________________________________ Class number: ________


Section: ____________ Schedule: _______________________________ Date: _______________

Lesson Title: Polynomial Curves: Second Materials: Ballpoint, Notebook, Calculator


Derivative Test & Point of Inflection
Textbook: Calculus by Ron Larson, 11th Ed.
Lesson Objectives: References:
At the end of the lesson, you should be able to: 1. The Calculus 7 by Louis Leithold
1. Discuss the properties of Concavity of Curves 2. Differential and Integral Calculus
1. 2. Apply the second derivative test to find by Clyde E. Love and Earl Rainville
maxima and minima of function 3. Calculus by James Stewart, 7th Ed
2. 3. Use the critical points to determine a curve
3. 4. Define the point of inflection
4. 5. Solve for the point of inflection of the curve
5.

DON’T BE BUSY, BE PRODUCTIVE


A. LESSON PREVIEW/REVIEW
1) Introduction

GOOD DAY! The following topics shall be


accomplished in this lesson:
1. Discuss the properties of Concavity of Curves.
2. Apply the second derivative test to find maxima
and minima of functions.
3. Use the critical points to determine a curve.
4. Define the point of inflection
5. Solve for the point of inflection of the curve

2) Activity 1: What I Know Chart, part 1

What I Know Questions: What I Learned (Activity 4)


1. Are you familiar with the
properties of Concavity of
Curves?
2. Do you know how to apply
the second derivative test in
finding the maxima and minima
of functions?
3. How do you use the critical
points in determining a curve?
4. Will you define the point of
inflection?
5. Can you solve for the point of
inflection of the curve?

This document is the property of PHINMA EDUCATION


1
MAT 171: Differential Calculus
Student’s Activity Sheets #10

Name: ______________________________________________________ Class number: ________


Section: ____________ Schedule: _______________________________ Date: _______________

B.MAIN LESSON

1) Activity 2: Content Notes

Concavity and the Second Derivative Test for Maxima and Minima
The second derivative, y'', is the rate of change of the first derivative. This means that y'' is
positive, while y' is increasing (as x increases) and the tangent turns in a counterclockwise
direction, the curve is concave upward. At the minimum point, the curve is concave upward.

When y'' is negative, y' decreases, and the tangent turns in clockwise direction, the curve is
concave downward. At the maximum point, the curve is concave downward.

Concavity test

a) If f''(x) > 0 for all x in interval ( I ), then the graph of f is concave upward on ( I ).

b) If f''(x) < 0 for all x in interval ( I ), then the graph of f is concave downward on ( I ).

Second Derivative Test for Maximum or Minimum

At a point y' = 0, if y'' < 0, y is a maximum;

If y'' > 0, y is minimum;

if y'' = 0, the test fails.

This document is the property of PHINMA EDUCATION


2
MAT 171: Differential Calculus
Student’s Activity Sheets #10

Name: ______________________________________________________ Class number: ________


Section: ____________ Schedule: _______________________________ Date: _______________
𝟑
Example 1. Find the maximum and minimum points of the function 𝒚 = 𝒙𝟑 + 𝟐
𝒙𝟐 − 𝟔𝒙 − 𝟒
by the second Derivative test.

𝟑
Given: 𝒚 = 𝒙𝟑 + 𝟐
𝒙𝟐 − 𝟔𝒙 − 𝟒

Solution: Write the original function


3
𝑦 = 𝑥 3 + 2 𝑥 2 − 6𝑥 − 4 (-2, 6)
𝑑𝑦 𝑑 3
Differentiate = (𝑥 3 + 𝑥 2 − 6𝑥 −4)
𝑑𝑥 𝑑𝑥 2
′ 𝑑𝑦 2 𝟑
y = 𝑑𝑥
= 3𝑥 + 3𝑥 − 6 𝒚 = 𝒙𝟑 +
𝟐
𝒙𝟐 − 𝟔𝒙 − 𝟒

Factor out 3 y ′ = 3(𝑥 + 𝑥 − 2)


2
𝑑𝑦
Set 𝑑𝑥
= y′ = 0 y ′ = 3(𝑥 + 2)(𝑥 − 1)
Solve for x 3(𝑥 + 2)(𝑥 − 1) = 0
x = -2, x = 1
Thus, x = -2 and x =1 are the critical values of x
Solve y
When x = - 2
3
Write the original function 𝑦 = 𝑥 3 + 2 𝑥 2 − 6𝑥 − 4
3
Substitute -2 in x y = (-2)3 + (-2)2 – 6(-2) – 4
2
Simplify y=6
Therefore, (-2, 6) is a critical point

When x = 1 Write original function


3
𝑦 = 𝑥 3 + 2 𝑥 2 − 6𝑥 − 4
3 𝟏𝟓
Substitute x = 1 y = (1)3 +2 (1)2 – 6(1) - 4 (1, - 𝟐
)
15
y=- 2
𝟏𝟓
Therefore (1, - 𝟐
) is another critical point
𝑑2 𝑦 𝑑 𝑑𝑦
Solve now the second derivative [ 𝑑𝑥 2 = (
𝑑𝑥 𝑑𝑥
)]
𝑑𝑦
𝑑𝑥
= 3𝑥 2 + 3𝑥 − 6
𝑑 𝑑𝑦 𝑑
( )
𝑑𝑥 𝑑𝑥
= 𝑑𝑥
(3𝑥 2 + 3𝑥 − 6)
𝑑2 𝑦
𝑑𝑥 2
= 6𝑥 + 3

Or y'' = 6x +3
Test the critical points (by using the second derivative test)
At CP ( -2, 6 )

This document is the property of PHINMA EDUCATION


3
MAT 171: Differential Calculus
Student’s Activity Sheets #10

Name: ______________________________________________________ Class number: ________


Section: ____________ Schedule: _______________________________ Date: _______________

y'' = 6x + 3
y'' = 6(-2) + 3
y'' = -9
Since y'' < 0, therefore ( -2, 6) is a maximum point, concave downward.
15
At CP ( 1, − 2
)
y'' = 6x + 3
y'' = 6(1) + 3
y'' = +9
𝟏𝟓
Since y'' > 0, therefore (1, − 𝟐
) is minimum point, concave upward.

Example 2. Determine the coefficient a, b, c etc. so that the curve will satisfy the stipulated
condition:

2a. Make the curve y = ax3 + bx2 + cx + d have a critical points at


( 0, 4 ) and ( 2, 0 ).
Solution:
At ( 0, 4), let x = 0, y = 4

y = ax3 + bx2 + cx + d
4=0+0+0+d
d=4

At (2, 0), let x = 2, y = 0

0 = a(2)3 + b(2)2 + c(2) + d


0 = 8a + 4b + 2c + 4
0 = 4a + 2b + c + 2 → 1

Solve for y'

y = ax3 + bx2 + cx + d at the critical points, solve for y' , then equate y' to 0
y' = 3ax2 + 2bx + c = 0

At CP (0, 4)

0=0+0+c
c=0

This document is the property of PHINMA EDUCATION


4
MAT 171: Differential Calculus
Student’s Activity Sheets #10

Name: ______________________________________________________ Class number: ________


Section: ____________ Schedule: _______________________________ Date: _______________

At CP ( 2, 0)

3a(2)2 + 2b(2) = 0
12a + 4b = 0
3a + b = 0 →2

From 1 4a + 2b + c + 2 = 0
c=0
4a + 2b + 0 + 2 = 0
2a + b + 1 = 0
2a + b = -1 →3

Subtract 2 & 3
3a + b = 0
2a + b = -1
a =1
when a = 1
3a + b = 0 from 1
3+b=0
b = -3

Therefore: a =1, b = -3, c = 0, d = 4


The required equation is 𝒚 = 𝒙𝟑 − 𝟑𝒙𝟐 + 4.

Point of Inflection
A point of inflection is a point at which a curve changes from concave upward to concave
downward, or from concave downward to concave upward.

Point of inflection test

(1) y = f(x) has point of inflection at a if the second derivative is zero at x = a and the
second derivative changes sign as the value of x increases through a.

(2) If the second derivative is zero but the third derivative is not equal to zero at x = a,
then y = f(x) has a point of inflection at x = a.

This document is the property of PHINMA EDUCATION


5
MAT 171: Differential Calculus
Student’s Activity Sheets #10

Name: ______________________________________________________ Class number: ________


Section: ____________ Schedule: _______________________________ Date: _______________
𝟑
Example 1. Find the point of inflection of the function y = x3 + 𝟐 x2 – 6x – 4
Solution:
3
y = x3 + x2 – 6x – 4 write the original equation
2
y' = 3x2 + 3x – 6 Differentiate (Solve for the first derivative)
y'' = 6x + 3 Solve for the second derivative

𝒅𝟐 𝒚
For point of inflection, we set 𝒅𝒙𝟐
=0

That is, 6x + 3 = 0 ,
Solve for x
3
x =-
6
1
x =-
2
1
When x = - 2
3
From y = x3 + x2 – 6x – 4
2
1 3 1 1
Solve for y, y = (- 2 )3 + 2
(- 2 )2 – 6 (- 2 ) – 4
1 3
=-8+ 8
+3–4
3
y=-
4
1 3
Therefore: (- , - 4 ) is a critical point for the test for point of inflection.
2

1 3
Test (- 2 , - 4 ) for point of inflection.
To test the point of inflection by statement (1).
1
(1) Select two values near - as follows:
2

1
Note – 1 < - <0
2
1 1
For x = - 2
when x < - 2 , y'' < 0
y'' = 6x + 3
1
y'' = 6(-1) + 3 x>-2, y'' > 0
y'' = -3
y'' = 6(0) + 3
y'' = 3

This document is the property of PHINMA EDUCATION


6
MAT 171: Differential Calculus
Student’s Activity Sheets #10

Name: ______________________________________________________ Class number: ________


Section: ____________ Schedule: _______________________________ Date: _______________

Since the sign of y'' changes from negative to positive as x increases, then the curve of
1 3
the function has point of inflection at (- 2 , - 4 )
To test the point of inflection by statement (2)
3
y = x3 + 2 x2 – 6x – 4
y' = 3x2 + 3x – 6
y'' = 6x + 3
y''' = 6
1 3
Since y''' ≠ 0, then the curve of the function has point of inflection at (- 2 , - 4 ).

SOLVE FOR THE


Activity 3: Skill-building Activities (with answer key) FOLLOWING
1. Locate and classify the critical points by Second PROBLEMS
Derivative Test.
1 1
1a. y = 3 x3 + 2 x2 – 6x + 8
1b. Locate and classify the critical points by
second derivative test. y = 4 – 6x + x2

2. Determine the coefficients a, b, c, and d so that the


curve will satisfy the stipulated condition. Make
the curve y = ax3 + bx2 + cx + d have critical points
(1, 0 ) and ( -2 , 27)

3. Find the points of inflection of the curve y = x4 – 4x3

Activity 4: What I Know Chart, part 2


What I Learned
1. ______________________________________________________________________________
______________________________________________________________________________
2. ______________________________________________________________________________
______________________________________________________________________________
3. ______________________________________________________________________________
______________________________________________________________________________

Activity 5: Check for Understanding

Locate and classify the critical points by second derivative test.


1. y = x3 – 3x2 – 9x + 20

2. If y = x3 – 12x + 8, Solve for the point of inflection.

This document is the property of PHINMA EDUCATION


7
MAT 171: Differential Calculus
Student’s Activity Sheets #10

Name: ______________________________________________________ Class number: ________


Section: ____________ Schedule: _______________________________ Date: _______________

C. LESSON WRAP-UP
1) Activity 6: Thinking about Learning
Work Tracker
You are done with this session! Let’s track your progress. Shade the session number you just
completed.

A. Think about your Learning


1. What motivated you to finish the lesson today?
_____________________________________________________________________________
_____________________________________________________________________________
2. What could you have done better to improve your learning today?
_____________________________________________________________________________
_____________________________________________________________________________

FAQs
1. What is concave up and down?
The derivative of a function gives the slope. When the slope continually increases, the
function is concave upward. When the slope continually decreases, the function
is concave downward.

2. What if the second derivative test is 0?


The second derivative is zero (f (x) = 0): When the second derivative is zero, it
corresponds to a possible inflection point. If the second derivative changes sign
around the zero (from positive to negative, or negative to positive), then the point is an
inflection point.

3. What is a point of inflection in calculus?

Inflection points are where the function changes concavity. Since concave up
corresponds to a positive second derivative and concave down corresponds to a
negative second derivative, then when the function changes from concave up to
concave down (or vice versa) the second derivative must equal zero at that point.

This document is the property of PHINMA EDUCATION


8
MAT 171: Differential Calculus
Student’s Activity Sheets #10

Name: ______________________________________________________ Class number: ________


Section: ____________ Schedule: _______________________________ Date: _______________

4. What happens if the second derivative is 0?

Since the second derivative is zero, the function is neither concave up nor concave
down at x = 0. It could be still be a local maximum or a local minimum and it even could
be an inflection point. Let's test to see if it is an inflection point. We need to verify that
the concavity is different on either side of x = 0.

KEY TO CORRECTIONS
𝟒𝟑
1a. ( -3 , 𝟐
) maximum
𝟐
(2 ,𝟑
) minimum
1b. ( 3, -7) minimum

2. b = 3
a=2
c = -12
d=7

3. (0, 0 )
(2,-16)

Activity 5.

1. At ( 3, -7 ) min
At ( -1, 25) max

2. 0, 8 )

This document is the property of PHINMA EDUCATION


9
Course Code: MAT 171
Course Title: Differential Calculus
Teachers’ Guide Module #11
Name: ______________________________________________________ Class number: ________
Section: ____________ Schedule: _______________________________ Date: _______________

Lesson Title: Application of the Derivative – Materials: Ballpoint, Notebook, Scientific


Optimization Problems Calculator.
Textbook: Calculus 11th Ed. by Ron Larson
Lesson Objectives: References:
At the end of the lesson, you should be able to: 1. The Calculus 7, by Louis Leithold
1. Recall and state the basic formulas of Solid 2. Differential and Integral Calculus
Mensuration by Clyde E. Love and Earl Rainville
1. 2. Solve problems dealing with maximum and 3.Calculus 7th Ed, by James Stewart
2. minimum values

Your mind is a powerful thing. When you fill it with positive thoughts, your life will start to change.

A. LESSON PREVIEW/REVIEW

Introduction

GOOD DAY! The following topics shall be


accomplished in this lesson:
1. The basic formulas of Solid Mensuration
2. Solve problems dealing with maximum and
minimum values

Activity 1: What I Know Chart, part 1

What I Know Questions: What I Learned (Activity 4)


1. Can you recall and state the
basic formulas of Solid
Mensuration?
2. Do you know how to solve
problems dealing with maximum
and minimum values?

B.MAIN LESSON

Activity 2: Content Notes

Application of Maxima and Minima

The problems in maxima and minima involve maximizing or minimizing a certain quantity with other
quantities with determinable or given values. The problems of this type are easily recognizable because
of a key word like any of the words: maximum, biggest, longest, highest, most, minimum, smallest,
lowest, least and the like. There is a need for you to recall some basic formulas of solid mensuration
that are very useful in this lesson.

This document is the property of PHINMA EDUCATION


1
Course Code: MAT 171
Course Title: Differential Calculus
Teachers’ Guide Module #11
Name: ______________________________________________________ Class number: ________
Section: ____________ Schedule: _______________________________ Date: _______________

1. Triangle
Perimeter = a + b + c
1
Area = base x altitude
2
1 1 1
Area = 2
ab Sin C, Area = 2 bc SinA, Area = 2 ac Sin B
Cosine Law: a2 = b2 + c2 – 2bccosA
b2 = a2 + c2 – 2accosB
c2 = a2 + b2 – 2abcosC
𝑎 𝑏 𝑐
Law of Sine: = =
sin 𝐴 sin 𝐵 sin 𝐶

2. Right triangle
Pythagorean Theorem: c2 = a2 + b2
Trigonometric Functions: SOH – CAH – TOA

3. Equilateral Triangle
√3
h= 2
S, h – height, S – side
√3
Area = S2
4

4. Rectangle
Perimeter = 2 (length + width)
=2(L+W)
Area = LW

5. Circle
Circumference = 2 𝜋 (radius) = 2 𝜋 r
Area = A = 𝜋 r2
𝜋𝑑 2
Area =
4

6. Trapezoid
1
Area = 2 (b + B)(h)
where: h – altitude
B – longer base
b = shorter base

7. Circular Sector
1
Area = r2 𝜃 where r = radius
2
S = 𝜃𝑟 𝜃 = central angle in radian
S = length of the arc

This document is the property of PHINMA EDUCATION


2
Course Code: MAT 171
Course Title: Differential Calculus
Teachers’ Guide Module #11
Name: ______________________________________________________ Class number: ________
Section: ____________ Schedule: _______________________________ Date: _______________

8. Right Circular Cylinder


S = 2 𝜋 rh where S= lateral surface area
V = 𝜋 r2h r = radius of base
h = height or altitude

9. Right Circular Cone


S = 𝜋 rL where: r = radius
1 2
V= 3
𝜋rh L = slant height
L = √𝑟 2 + ℎ2
h = height

10. Sphere
S = 4 𝜋 r2
4
V = 𝜋 r3
3

Steps in Solving Max – Min Problems


1. Read the problem carefully until it is clearly understood. Ask yourself:
What is the unknown? What are the given quantities?
What are the given condition?

2. Draw a figure whenever necessary and denote the variable quantities by, x, y, z etc.

3. Identify the quantity to be maximized or minimized and express it in other variable


quantities. If possible, express this quantity in terms of one independent variable.

4. Find the first derivative of the function and equate the derivative to zero.

5. Solve for values of the variable and check whether they maximize or minimize the
given quantity.

This document is the property of PHINMA EDUCATION


3
Course Code: MAT 171
Course Title: Differential Calculus
Teachers’ Guide Module #11
Name: ______________________________________________________ Class number: ________
Section: ____________ Schedule: _______________________________ Date: _______________

Example 1. Express the diameter of a spherical ball as a function of its volume.


𝜋𝐷 3 6𝑉 3 6𝑉
Since the volume is given by: 𝑉 = then, 𝐷 3 = Thus, 𝐷 = √
6 𝜋 𝜋

6
Example 2. The area of a rectangle is 6 sq.in, then the length of the other side is 𝑥
and the
perimeter is ____.

From: 𝑃 = 2𝑥 + 2𝑦
𝑃 = 2(𝑥 + 𝑦) And: 𝐴 = 𝑥𝑦 y A= 6 sq.in.
6 = 𝑥𝑦
6
𝑦=𝑥 x
6
Thus, 𝑃𝑒𝑟𝑖𝑚𝑒𝑡𝑒𝑟 = 𝑃 = 2( 𝑥 + )
𝑥

Example 3. The sum of two numbers is 100. Find the minimum value of the sum of their cubes.

Solution:
Let x = first number
y = second number
Given condition:
x + y = 100
x = 100 – y →1
Sum of the cubes
S = x3 + y3 →2 Substitute x = 100 – y in equation 2
S = (100 – y ) + y3
3
Apply power rule, chain rule
Differentiate
𝑑𝑆
= 3( 100 - )2 (-1) + 3y2
𝑑𝑦
𝑑𝑆 𝑑𝑠
𝑑𝑦
= -3(100 –y)2 + 3y2 = 0 equate 𝑑𝑦
=0
- (100 –y)2 + 3y2 = 0
-(10000 - 200y + y2 ) + y2 = 0 simplify
-10000 + 200y – y2 + y2 = 0
10000
y= = 50
200
From x = 100 – y, but y = 50
x = 100 – 50
x = 50
From 2 S = x3 + y3
= (50)3 + (50)3
S = 250,000
Therefore the minimum value of the sum of their cubes is, S = 250,000.

This document is the property of PHINMA EDUCATION


4
Course Code: MAT 171
Course Title: Differential Calculus
Teachers’ Guide Module #11
Name: ______________________________________________________ Class number: ________
Section: ____________ Schedule: _______________________________ Date: _______________

Example 4. Find the dimensions of a rectangle with perimeter 100m and whose area is as large
as possible.

Let: L - length of rectangle


W - width of rectangle
P - perimeter = 100 m
A – area
Solution:
From the given condition
P = 2( L + W)
100 = 2( L + W)
50 = L + W
L = 50 – W →1
Area ( A ) = LW →2
A = ( 50 – W) (W) But L = 50 – W
A = 50W – W 2
𝑑𝐴
𝑑𝑊
= 50 – 2W = 0 Equate to zero
2W = 50 Solve for the W
W = 25
From 1 L = 50 – W but W= 25
L = 50 – 25
L = 25
Therefore, L = 25m, W = 25m

Example 5. Find the area of the largest rectangle that can be inscribed in a given circle.

Solution:
Let: A = WL, L – Length of the rectangle
W–Width of the rectangle
By Pythagorean Theorem
(2r)2 = L2 + W2
4r2 = L2 + W2
L = √4𝑟 2 − 𝑊 2 →1
Since the area is to be maximized,
then expressing A = WL in terms of single variable,
we have,
A = W√4𝑟 2 − 𝑊 2
From 1 L = √4𝑟 2 − 𝑊 2 substitute

This document is the property of PHINMA EDUCATION


5
Course Code: MAT 171
Course Title: Differential Calculus
Teachers’ Guide Module #11
Name: ______________________________________________________ Class number: ________
Section: ____________ Schedule: _______________________________ Date: _______________

𝑑𝐴 −2𝑊
= 𝑊( ) + √4𝑟 2 − 𝑊 2 = 0
𝑑𝑊 2√4𝑟 2 −𝑊 2

𝑊2
= √4𝑟 2 − 𝑊 2
√4𝑟 2 −𝑊 2
W 2 = 4r2 – W 2
2W 2 = 4r2
W 2 = 2r2
W = √2 r

From 1 L = √4𝑟 2 − 2𝑟 2
= √2𝑟 2
L = √2 r
Area = LW
From A = LW substitute L = √2 r
= (√2 r ) (√2 r )
Therefore, Area = 2r2

Example 6. A box is to be made of cardboard 9 in. square by cutting equal squares out of the
corners and turning up the sides. Find the Volume of the largest box that can be
made in this way.

Solution:
Let x – be the length of the side of the tiny square cut out of each corner of the given square of
side 9 in. as indicated in the figure.

Volume ( V ) = ( Base ) ( Height)


V = ( 9 – 2x ) 2 (x)
V = ( 81 – 36x + 4x2 ) (x)
= 81x – 36x2 + 4x3
For maximum Volume:
𝑑𝑉 𝑑𝑉
= 81 − 72𝑥 + 12𝑥 2 = 0 Set =0
𝑑𝑥 𝑑𝑥
4x – 24x + 27 = 0
2

( 2x – 3) ( 2x – 9 ) = 0 Factor
3 9
x = 2 = 1.5, x = 2 = 4.5

Then, Vmax = [ 9- 2x]2 [x]


= ( 9 – 2(1.5))2(1.5)
= (36)(1.5)

This document is the property of PHINMA EDUCATION


6
Course Code: MAT 171
Course Title: Differential Calculus
Teachers’ Guide Module #11
Name: ______________________________________________________ Class number: ________
Section: ____________ Schedule: _______________________________ Date: _______________

Vmax = 54 cu. inches


Test for
𝑑2 𝑉
𝑑𝑥 2
= -72 + 24x At x = 1.5
= -72 + 24(1.5)
= -36
𝒅𝟐 𝑽
𝒅𝒙𝟐
< 𝟎, Therefore maximum

Example 7. A rectangular field of given area is to be fenced off along the bank of the river. If no
fence is needed along the river, what is the shape of the rectangle requiring the
least amount of fencing?
Solution:
Length of fencing ( F ) = 2y + x →1
Area = xy
A = xy
𝐴
x=𝑦 →2
𝐴
F = 2y + x but x =
𝑦
𝐴
F = 2y + 𝑦
For minimum
𝑑𝐹 𝐴
𝑑𝑦
=2− 𝑦2
=0
𝐴
𝑦2
=2
A = 2y2
Equating
xy = 2y2
𝒙
x = 2y Or, y = 𝟐

Therefore, x is twice y
Length is twice the width
The width of the field should be half its length

This document is the property of PHINMA EDUCATION


7
Course Code: MAT 171
Course Title: Differential Calculus
Teachers’ Guide Module #11
Name: ______________________________________________________ Class number: ________
Section: ____________ Schedule: _______________________________ Date: _______________

Example 8. A closed cylindrical tank is to be made with a fixed volume. Find the relative
dimensions of the tank that will require the least amount of material in making it.

Solution:

Let A = total surface area of the tank


Ab = area of the bottom = 𝜋 r2
At = area of the top = 𝜋 r2
As = area of the side = 2 𝜋 rh
V = Volume of the tank (constant ) = 𝜋 r2h

The quantity to be minimized is A.


Thus, A = Ab + At + As
= 𝜋 r2 + 𝜋 r2 + 2 𝜋 rh
From, V = 𝜋 r2h
𝑉
h= →1
𝜋𝑟 2
𝑉
= 2 𝜋 r2 + 2 𝜋 r( ) Substitute
𝜋𝑟 2
2𝑉
A = 2 𝜋 r2 + 𝑟

Differentiating A with respect to r and keeping in mind that V is a constant.


𝑑𝐴 2𝑉 𝑑𝐴
= 4𝜋 r- ; = 0
𝑑𝑟 𝑟2 𝑑𝑟
2𝑉
4𝜋 r- =0
𝑟2
2𝑉
4𝜋 r = 𝑟2
2𝑉 𝑉
r3 = 4𝜋 =
2𝜋

V = 2πr3 →2

Substitute, V = 2 𝜋 r3 but V = 𝜋 r2h


𝜋 r2h = 2 𝜋 r3
h = 2r

The result tells us that the proportion which requires the best amount of material in making
the tank with a fixed volume is that the height shall be twice the radius of the base.

This document is the property of PHINMA EDUCATION


8
Course Code: MAT 171
Course Title: Differential Calculus
Teachers’ Guide Module #11
Name: ______________________________________________________ Class number: ________
Section: ____________ Schedule: _______________________________ Date: _______________

Activity 3: Skill-building Activities (with answer key)

1. A rectangular page is to contain 24 square inches of print. The margins at the top and the
bottom of the page are 1.5 inches, and the margins on the left and the right are 1 inch ( see
figures). What should be the measure of the page for the least amount of paper?

2. Find two positive numbers whose sum is 8 and the


product is minimum.

3. Find two positive number whose product is 196 and


the sum is a minimum.

4. Find the length and width of a rectangle that has the


given area 49 and a minimum perimeter.

5. Find two positive numbers that satisfy the given


requirements.
. a. The product is 147 and the sum of the first number
plus times the second number is a maximum.

b. The sum of the first number and twice the second


number is 108, and the product is a maximum.

Activity 4: What I Know Chart, part 2

What I Learned
1. ______________________________________________________________________________
______________________________________________________________________________

2. ______________________________________________________________________________
______________________________________________________________________________

3. ______________________________________________________________________________
______________________________________________________________________________

Solve the
Activity 5: Check for Understanding Problem!

1. Find the length and width of a rectangle having a


perimeter equal to 80m and a maximum area.

This document is the property of PHINMA EDUCATION


9
Course Code: MAT 171
Course Title: Differential Calculus
Teachers’ Guide Module #11
Name: ______________________________________________________ Class number: ________
Section: ____________ Schedule: _______________________________ Date: _______________

C. LESSON WRAP-UP

Activity 6: Thinking about Learning

Work Tracker
You are done with this session! Let’s track your progress. Shade the session number you just
completed.

Think about your Learning

1. What motivated you to finish the lesson today?


________________________________________________________________________________________
________________________________________________________________________________________

2. What could you have done better to improve your learning today?
________________________________________________________________________________________
________________________________________________________________________________________

FAQs

1.How do you solve maxima and minima word problems easily?

You should follow the three steps. First step is find the derivative of the function. Next
set the derivative equal to 0 and solve for x. This gives you the x-values of the maximum and
minimum points. Then plug those x-values back into the function to find the corresponding y-
values. This will give you your maximum and minimum points of the function.

2. Is the maximum area of a rectangle always a square?

The area of a rectangle is the length times the width. With the given amount of fencing,
you get the maximum area if the length and width are equal, that is, the shape of the garden is a
square.
KEY TO CORRECTIONS
Activity 3: b. two positive numbers are 54 and 27.
1. 9 in. x 6 in. Activity 5.
2. Two positive numbers are 4 and 4.
3. Two positive numbers are 14 and 14. 1. W = 20, L = 20
4. length = 7 and the width = 7.
5. a. Two positive numbers are 21 and 7

This document is the property of PHINMA EDUCATION


10
MAT 171: Differential Calculus
Student’s Activity Sheets #12
Name: ______________________________________________________ Class number: ________
Section: ____________ Schedule: _______________________________ Date: _______________

Lesson Title: Application of Derivative: Materials: Ballpoint, Notebook, Scientific


TIME RATES Calculator.
Textbook: Calculus 11th Ed. by Ron Larson
Lesson Objectives: References:
At the end of the lesson, you should be able to: 1. The Calculus 7, by Louis Leithold
1. Illustrate the derivative of functions with 2. Differential and Integral Calculus by
respect to time Clyde E. Love and Earl Rainville
1. 2. Solve problems in time-rates 3.Calculus 7th Ed, by James Stewart
2.

THE BEST PREPARATION FOR TOMORROW


IS DOING YOUR BEST TODAY
A. LESSON PREVIEW/REVIEW

Introduction

GOOD DAY! The following topics shall be accomplished


in this lesson:
1. Illustrate the derivative of functions with respect to time
2. Solve problems in time-rates

Activity 1: What I Know Chart, part 1

What I Know Questions: What I Learned (Activity 4)


1. Will you illustrate the derivative
of functions with respect to time?
2. Can you solve problems
involving time-rates?

B.MAIN LESSON

Activity 2: Content Notes

TIME –RATES
The derivative of a function is identical with the rate of change. This application tackles another
rate of change, but this time with respect to time.

When two or more variables, all functions of t are related by an equation, the relation between
their rates of change may be obtained by differentiating the quantity with respect to t.

This document is the property of PHINMA EDUCATION


1
MAT 171: Differential Calculus
Student’s Activity Sheets #12
Name: ______________________________________________________ Class number: ________
Section: ____________ Schedule: _______________________________ Date: _______________

𝒅𝒔
Velocity ………….. v= , where: s is the distance, v is velocity.
𝒅𝒕
𝒅𝒗 𝒅𝟐 𝒔
Acceleration ……. a= =
𝒅𝒕 𝒅𝒕𝟐
𝒅𝑽
Discharge ……….. 𝜽= , V is the volume at anytime
𝒅𝒕
𝒅𝜽
Angular Speed….. 𝝎= , 𝜃 𝑖𝑠 𝑡ℎ𝑒 𝑎𝑛𝑔𝑙𝑒 at any time.
𝒅𝒕

Illustrations of derivative with respect to time, t.


𝑑𝑦 𝑑𝑥
1. y = x2, 𝑑𝑡
= 2x 𝑑𝑡

𝑑𝑥 𝑑𝑦
2. x2 + y2 = 10, 2x + 2y =0
𝑑𝑡 𝑑𝑡

4 𝑑𝑣 𝑑𝑟
3. V = 3 𝜋 r3, 𝑑𝑡
= 4 𝜋 r2 𝑑𝑡

𝑑𝐷
4. D2 = ( 5 – 2t)2 + (2t)2, 2D 𝑑𝑡
= -2( 5- 2t) ( -2) + 2(2t)(2)
𝑑𝐷
2D 𝑑𝑡
= -4( 5- 2t) + 8t

𝑑𝑥 𝑑𝑥
𝑑𝑆 20𝑥 10𝑥
𝑑𝑡 𝑑𝑡
5. S = √4 + 10𝑥 2 , = =
𝑑𝑡 2√4+10𝑥 2 √4+10𝑥 2
𝑑𝑉 𝑑ℎ
6. V = 𝜋 r2h, 𝑑𝑡
= 𝜋 r2 𝑑𝑡
, r is constant in cylinder

Guidelines in Solving Time–Rate Problems.

1. Identify all given quantities and quantities to be determined, make a sketch and label the
quantities.

2. Write an equation involving the variables whose rates of change either are given or are to be
determined.

This document is the property of PHINMA EDUCATION


2
MAT 171: Differential Calculus
Student’s Activity Sheets #12
Name: ______________________________________________________ Class number: ________
Section: ____________ Schedule: _______________________________ Date: _______________

3. Using the chain rule, implicitly differentiate both sides of the equation with respect to time t.

4. After completing step 3, substitute into the resulting equation all known values for the
variables and their rates of change. Then solve for the required rate of change.

Note: In solving “time-rate” problems, it is important to observe that all quantities which
change with respect to time must be denoted by letters.

“Do not substitute the numerical values of such variable until after differentiation with
respect to time t is done”.

The following problems give the application of rate of change with respect to time.

Example 1. Water flows into a vertical cylindrical tank at the rate of 24 cu.ft. per minute. If the
radius of the tank is 4 feet, how fast is the surface rising?

𝑑𝑉
Given rate: = 24 cu. ft./min.
𝑑𝑡
𝑑ℎ
Find when r = 4
𝑑𝑡

Solution: Volume of the water (V) = 𝜋 r2h


V = 𝜋 r2h
differentiate both sides of the equation with
respect to t
𝑑𝑉 𝑑ℎ 𝑑ℎ
𝑑𝑡
= 𝜋 r2 𝑑𝑡 ; solve for 𝑑𝑡
𝑑𝑉
𝑑ℎ 𝑑𝑡
=
𝑑𝑡 𝜋𝑟 2

𝑑ℎ
Finally, when r = 4 , the rate of change 𝑑𝑡
is,
𝑑ℎ 24
𝑑𝑡
= 𝜋(4)2
𝒇𝒕
= 0.477
𝒎𝒊𝒏

This document is the property of PHINMA EDUCATION


3
MAT 171: Differential Calculus
Student’s Activity Sheets #12
Name: ______________________________________________________ Class number: ________
Section: ____________ Schedule: _______________________________ Date: _______________

Example 2. A ladder 20 ft. long leans against a vertical wall. If the top slides downward at the
rate of 2 feet/sec. Find how fast the lower end is moving when it is 16 feet from the
wall.

Given : Length of the ladder 20 ft.


𝑑ℎ
𝑑𝑡
= -2 feet/sec. (the minus sign denotes that the ladder is sliding down)

By Pythagorean theorem h2 + x2 = 202 →1

Differentiate both sides with respect to time ( t )


𝑑 𝑑
𝑑𝑡
(h2 + x2) = 𝑑𝑡
202

𝑑ℎ 𝑑𝑥
2h 𝑑𝑡
+ 2x 𝑑𝑡
=0

𝑑𝑥 𝑑ℎ 𝑑𝑥
x 𝑑𝑡
=-h 𝑑𝑡
; solve for 𝑑𝑡

When x = 16 ft., From h2 + x2 = 400


h = √400 − 162
h = 12 ft.
𝑑ℎ
𝑑𝑥 −ℎ
𝑑𝑡
𝑑𝑡
= 𝑥

𝑑𝑥 −(12)(−2)
𝑑𝑡
= 16

𝒅𝒙
𝒅𝒕
= 1.5 ft/s

This document is the property of PHINMA EDUCATION


4
MAT 171: Differential Calculus
Student’s Activity Sheets #12
Name: ______________________________________________________ Class number: ________
Section: ____________ Schedule: _______________________________ Date: _______________

Example 3. A water tank in the form of an inverted cone is being implied at the rate of 2 cubic
feet/second. The height of the cone is 8 feet and the radius is 4 feet (Figure 3a, 3b).
Find the rate of change of the water level when the depth is 6 feet.
Solution:
𝑑𝑉 𝑓𝑡 3
Given : 𝑑𝑡
=2 𝑠

Required:
𝑑ℎ
- the rate change of the water
𝑑𝑡
level when the depth is 6 feet

Let: h- height of the water


. r – radius of the water
1
The Volume of the water ( in the form of right circular cone ) is given by V = 3 𝜋 r2h
We can use similar triangles to get a relationship between r and h.
𝑟 4
=8

1
r=2h
1 1 1
From the formula V = 𝜋 r2h, Substitute r = h in V = 𝜋 r2h
3 2 3
1 1
V= 3
𝜋 (2 h) 2 h
1
V= 𝜋 h3
12
Differentiate this formula with respect to time t
𝑑𝑉 3 𝑑ℎ
𝑑𝑡
= 12 𝜋 h2 𝑑𝑡
1 𝑑ℎ
= 4 𝜋 h2 𝑑𝑡

𝑑ℎ
Solve the last equation for
𝑑𝑡
𝑑𝑉
𝑑ℎ 4
𝑑𝑡
=
𝑑𝑡 πℎ2
𝑑𝑉
Substitute = 2 cu.ft./sec. and h = 6 feet.
𝑑𝑡
𝑑ℎ 4(2) 8 2
= 𝜋(6)2 = =
𝑑𝑡 36𝜋 9𝜋

𝑑ℎ 𝑓𝑡
Solving for 𝑑𝑡
= 0.0707 𝑠𝑒𝑐

𝒅𝒉 𝒇𝒕
Therefore, 𝒅𝒕
= 0.0707 𝒔𝒆𝒄

This document is the property of PHINMA EDUCATION


5
MAT 171: Differential Calculus
Student’s Activity Sheets #12
Name: ______________________________________________________ Class number: ________
Section: ____________ Schedule: _______________________________ Date: _______________

Example 4. A rectangular trough is 7 feet long, 3 feet across the top, and 4 feet deep. If water
𝒇𝒕𝟑
flows in at the rate of 3 𝒎𝒊𝒏. , how fast is the surface rising, when the water is 1 ft.
deep.

Solution:
𝑑𝑉 𝑓𝑡 3
Given 𝑑𝑡
= 3 𝑚𝑖𝑛. ,

𝑑ℎ
Required: =? when h = 1 ft.
𝑑𝑡

Let: h - be the depth of the water


Volume of the rectangular trough is V = Bh,
but the base is constant, that is B = 7(3) = 21
V = (7) (3) h
V = 21h
𝑑𝑉 𝑑ℎ
𝑑𝑡
= 21 𝑑𝑡
𝑑ℎ
Solve for 𝑑𝑡

𝑑𝑉
𝑑ℎ 𝑑𝑡
𝑑𝑡
= 21

𝑑ℎ 3
=
𝑑𝑡 21
𝒇𝒕
= 0.1428 𝒎𝒊𝒏

This document is the property of PHINMA EDUCATION


6
MAT 171: Differential Calculus
Student’s Activity Sheets #12
Name: ______________________________________________________ Class number: ________
Section: ____________ Schedule: _______________________________ Date: _______________

Example 5. A rectangular trough is 10 feet long, 4 feet across the top, and 4 feet deep. If water
𝒇𝒕𝟑
flows in at the rate of 3 𝒎𝒊𝒏. , find how fast is the surface rising, when the water is 0.5
ft. deep.
Solution:
𝑑𝑉 𝑓𝑡 3
Given : 𝑑𝑡
= 3 𝑚𝑖𝑛

1
Volume of the water (V)= 2 (x) (y) L
1
V = (x) (y) (10)
2

= 5(x)(y) →1
𝑥 4
By similar triangle: 𝑦
=4

x=y
From 1 V = 5xy but x = y
V = 5(y)(y)
= 5 y2

𝑑𝑉 𝑑𝑦
Differentiate both sides = 10y
𝑑𝑡 𝑑𝑡
𝑑𝑉
𝑑𝑦 𝑑𝑡 𝑑𝑉 𝑓𝑡 3
𝑑𝑡
= 10𝑦
, but 𝑑𝑡
= 3 𝑚𝑖𝑛 ; and y = 0.5
𝒅𝒚 3 𝒇𝒕
𝒅𝒕
= 10(0.5) = 0.6
𝒎𝒊𝒏

This document is the property of PHINMA EDUCATION


7
MAT 171: Differential Calculus
Student’s Activity Sheets #12
Name: ______________________________________________________ Class number: ________
Section: ____________ Schedule: _______________________________ Date: _______________

Activity 3: Skill-building Activities (with answer key)


𝑚3
1. Water is poured into a conical tank 6 m across the top and 8m deep at the rate of 10 𝑚𝑖𝑛
How fast is the water level rising when the water in the tank is 5m deep?

Activity 4: What I Know Chart, part 2


What I Learned
1. ______________________________________________________________________________
______________________________________________________________________________
2. ______________________________________________________________________________
______________________________________________________________________________
3. ______________________________________________________________________________
______________________________________________________________________________

Activity 5: Check for Understanding SOLVE!

1. A Meralco worker left a ladder 5 m long leaning against


a concrete vertical post. If the bottom of the ladder is
𝑚
pulled horizontally away from the post at 3 𝑠𝑒𝑐 , how fast
is the top of the ladder sliding down when the bottom is
4m from the base of the post.

C. LESSON WRAP-UP

Activity 6: Thinking about Learning

Work Tracker
You are done with this session! Let’s track your progress. Shade the session number you just
completed.

This document is the property of PHINMA EDUCATION


8
MAT 171: Differential Calculus
Student’s Activity Sheets #12
Name: ______________________________________________________ Class number: ________
Section: ____________ Schedule: _______________________________ Date: _______________

Think about your Learning

1. What motivated you to finish the lesson today?


________________________________________________________________________________________
________________________________________________________________________________________

2. What could you have done better to improve your learning today?
________________________________________________________________________________________
________________________________________________________________________________________

FAQs

1.What is the rate of change of the area of a circle with respect to the radius?

The rate of change of the area of a circle with respect to its radius is 2𝜋r.

2.What are related rates used for?

In differential Calculus, related rates problems involve finding a rate at which a quantity
changes by relating that quantity to other quantities whose rates of change are known. The rate
of change is usually with respect to time.

KEY TO CORRECTIONS

Activity 3:
𝒅𝒉 𝒎
1. 𝒅𝒕 = 0.905 𝒎𝒊𝒏

Activity 5:
𝒅𝒉 −(4)(3) 𝒎
1. = = -4 𝒔𝒆𝒄
𝒅𝒕 3

This document is the property of PHINMA EDUCATION


9
MAT 171: Differential Calculus
Student’s Activity Sheets #13
Name: ______________________________________________________ Class number: ________
Section: ____________ Schedule: _______________________________ Date: _______________

Lesson title: Differentiation of Transcendental Materials: Ballpoint, Notebook, Scientific


Functions: Trigonometric Functions & Calculator
Application of Maxima and Minima
Involving Trigonometric Functions Textbook: Calculus 11th Edition by Ron Larson
References:
Lesson Objectives: 1.The Calculus 7 by Louis Leithold
At the end of the lesson, you should be able to: 2. Calculus 7th Edition by James Stewart
1. Recall and state the trigonometric identities 3. Differential and Integral Calculus by Clyde
2. State and derive the derivative of trigonometric Love and Earl Rainville
functions
3. Find the derivative of trigonometric functions

The ladder of success is best


climbed by stepping on the ladder
of opportunity.
A. LESSON PREVIEW/REVIEW

Introduction

GOOD DAY. The following topics shall be discussed


in this lesson:

1. The Trigonometric Identities


2. The Derivative of Trigonometric Functions
3. The application of maxima and minima involving trigonometric functions.

Activity 1: What I Know Chart, part 1

What I Know Questions: What I Learned (Activity 4)


1. Are you familiar with the
trigonometric identities?
2. Can you derive the derivative of
trigonometric functions?
3. Do you know how to find the
derivative of trigonometric
functions?
2. Can you solve problems on
maxima and minima using
trigonometric functions?

This document is the property of PHINMA EDUCATION


1
Course Code: MAT 171
Course Title: Differential Calculus
Teachers’ Guide Module #18

Name: _____________________________________________________________ Class number: _______


Section: ____________ Schedule:_______________________________________ Date: ______________

B.MAIN LESSON

Activity 2: Content Notes

 A function that is not algebraic is called a transcendental function.

 In the next three lessons, we will discuss the four types of transcendental functions,
namely,
 trigonometric,
 inverse trigonometric,
 logarithmic and exponential functions.

Our lesson for today is limited to the Trigonometric functions.

Trigonometric Functions
In the study of calculus, it is strongly recommended that you make a thorough review of the
fundamental facts, formulas and identities of trigonometry. Nevertheless, the following
fundamental identities are reproduced here for your easy reference and recall.

Pythagorean Identities Let us recall


the Trigonometric Identities
1. sin2 𝑥 + cos2 𝑥 = 1
2. tan2 𝑥 + 1 = sec 2 𝑥
3. cot 2 𝑥 + 1 = csc 2 𝑥

Reciprocal Relation
1
4. sin 𝑥 = csc 𝑥
1
5. cos 𝑥 = sec 𝑥
1
6. 𝑡𝑎𝑛𝑥 = 𝑠𝑒𝑐𝑥

Quotient Relation

𝑠𝑖𝑛𝑥
7. 𝑡𝑎𝑛𝑥 =
𝑐𝑜𝑠𝑥
𝑐𝑜𝑠𝑥
8. 𝑐𝑜𝑡𝑥 =
𝑠𝑖𝑛𝑥

This document is the property of PHINMA EDUCATION


2
Course Code: MAT 171
Course Title: Differential Calculus
Teachers’ Guide Module #18

Name: _____________________________________________________________ Class number: _______


Section: ____________ Schedule:_______________________________________ Date: ______________

Derivative of Trigonometric Functions


1. Derivative of 𝒔𝒊𝒏𝒙

The derivative of 𝑠𝑖𝑛𝑥 is obtained by considering the “Three-Step Rule (Lesson #4).

Example, consider 𝑦 = 𝑓(𝑥) = 𝑠𝑖𝑛𝑥

Solution: Given: 𝑦 = sin 𝑥


Step 1: 𝑓(𝑥 + ∆𝑥) − 𝑓(𝑥) = sin(𝑥 + ∆𝑥) − sin 𝑥 from the sum of two angles
sin(𝐴 + 𝐵) = 𝑆𝑖𝑛 𝐴 cos 𝐵 + cos 𝐴 sin 𝐵
= sin 𝑥 cos ∆𝑥 + cos 𝑥 sin ∆𝑥 − sin 𝑥
= cos 𝑥 sin ∆𝑥 − sin 𝑥 + sin 𝑥 cos ∆𝑥
= cos 𝑥 sin ∆𝑥 − sin 𝑥(1 − cos ∆𝑥)
1 ∆𝑥 1
From trigonometry: 𝑠𝑖𝑛2 𝐴 = 2 (1 − cos 2𝐴), hence, 𝑠𝑖𝑛2 2
= 2 (1 − cos ∆𝑥)
∆𝑥
(1 − cos ∆𝑥) = 2 𝑠𝑖𝑛2
2
∆𝒙
= 𝐜𝐨𝐬 𝒙 𝐬𝐢𝐧 ∆𝒙 − 𝐬𝐢𝐧 𝒙 (𝟐 𝒔𝒊𝒏𝟐 )
𝟐

∆𝑥 ∆𝑥
𝑓(𝑥+∆𝑥)−𝑓(𝑥) cos 𝑥 sin ∆𝑥 2 sin 𝑥(sin )(𝑠𝑖𝑛 )
2 2
Step 2: = −
∆𝑥 ∆𝑥 ∆𝑥
∆𝒙
𝐜𝐨𝐬 𝒙 𝐬𝐢𝐧 ∆𝒙 𝐬𝐢𝐧 ∆𝒙
𝟐
= ∆𝒙
− 𝐬𝐢𝐧 𝒙 ( ∆𝒙 )(𝒔𝒊𝒏 𝟐
)
𝟐
sin 𝛼
From Theorem: lim =1
𝛼→0 𝛼
sin ∆𝑥
lim =1
∆𝑥→0 ∆𝑥
1
𝑠𝑖𝑛 ∆𝑥
2
lim ∆𝑥 =1
∆𝑥→0
2
∆𝑥
lim 𝑠𝑖𝑛 ( 2 ) = 0
∆𝑥→0

𝑓(𝑥+∆𝑥)−𝑓(𝑥) 𝑑𝑦
Step 3: lim ∆𝑥
= 𝑑𝑥
= (cos 𝑥)(1) − (sin 𝑥)(1)(0)
∆𝑥→0
𝒅𝒚
Therefore: 𝒅𝒙
= 𝐜𝐨𝐬 𝒙
𝑑𝑦 𝑑
𝑑𝑥
= 𝑑𝑥
𝑠𝑖𝑛𝑥 = 𝑐𝑜𝑠𝑥

𝒅 𝒅𝒖
By chain rule, 𝒅𝒙
𝒔𝒊𝒏𝒖 = 𝒄𝒐𝒔𝒖 𝒅𝒙

This document is the property of PHINMA EDUCATION


3
Course Code: MAT 171
Course Title: Differential Calculus
Teachers’ Guide Module #18

Name: _____________________________________________________________ Class number: _______


Section: ____________ Schedule:_______________________________________ Date: ______________

2. Derivative of Cosine 𝒙

The derivative of 𝑐𝑜𝑠𝑥 may also be obtained directly from the “three-step rule”. However, it is
𝜋 𝜋
easily be verified by using the trigonometric identities, 𝑐𝑜𝑠𝑥 = sin ( 2 − 𝑥) , sinx = cos( 2 − 𝑥).
𝑑 𝑑 𝜋
𝑐𝑜𝑠𝑥 = sin ( − 𝑥)
𝑑𝑥 𝑑𝑥 2
𝜋
= [cos ( − 𝑥)] (−1)
2
= −𝑠𝑖𝑛𝑥
𝑑
𝑐𝑜𝑠𝑥 = −𝑠𝑖𝑛𝑥
𝑑𝑥
𝒅 𝒅𝒖
By chain rule, 𝒄𝒐𝒔𝒖 = −𝒔𝒊𝒏𝒖 .
𝒅𝒙 𝒅𝒙

The remaining trigonometric functions may be differentiated


by expressing them in terms of the 𝑠𝑖𝑛𝑒 𝑎𝑛𝑑 𝑐𝑜𝑠𝑖𝑛𝑒.

3. Derivative of 𝒕𝒂𝒏𝒙

𝑑 𝑑 𝑠𝑖𝑛𝑥
𝑑𝑥
𝑡𝑎𝑛𝑥 = 𝑑𝑥 𝑐𝑜𝑠𝑥

Differentiate by applying quotient rule


𝑑 𝑑
𝑑 𝑐𝑜𝑠𝑥 𝑠𝑖𝑛𝑥−𝑠𝑖𝑛𝑥 𝑐𝑜𝑠𝑥
𝑑𝑥 𝑑𝑥
𝑑𝑥
𝑡𝑎𝑛𝑥 = (𝑐𝑜𝑠𝑥)2
𝑐𝑜𝑠𝑥𝑐𝑜𝑠𝑥−(𝑠𝑖𝑛𝑥)(−𝑠𝑖𝑛𝑥)
= (𝑐𝑜𝑠𝑥)2
1
𝑐𝑜𝑠𝑥𝑐𝑜𝑠𝑥+𝑠𝑖𝑛𝑥𝑐𝑜𝑠𝑥
= cos2 𝑥
cos2 𝑥+sin2 𝑥
= cos2 𝑥
𝑑 1
𝑡𝑎𝑛𝑥 =
𝑑𝑥 cos2 𝑥

= sec 2 𝑥
𝒅 𝒅𝒖
By chain rule: 𝒕𝒂𝒏𝒙 = 𝐬𝐞𝐜 𝟐
𝒅𝒙 𝒅𝒙

This document is the property of PHINMA EDUCATION


4
Course Code: MAT 171
Course Title: Differential Calculus
Teachers’ Guide Module #18

Name: _____________________________________________________________ Class number: _______


Section: ____________ Schedule:_______________________________________ Date: ______________

4. Derivative of cot x
𝑑 𝑑 𝑐𝑜𝑠𝑥
𝑑𝑥
𝑐𝑜𝑡𝑥 = 𝑑𝑥 𝑠𝑖𝑛𝑥
Apply quotient rule
𝑑 𝑑
𝑠𝑖𝑛𝑥 𝑐𝑜𝑠𝑥−𝑐𝑜𝑠𝑥 𝑠𝑖𝑛𝑥
𝑑𝑥 𝑑𝑥
= (𝑠𝑖𝑛𝑥)2
𝑠𝑖𝑛𝑥(−𝑠𝑖𝑛𝑥)−𝑐𝑜𝑠𝑥𝑐𝑜𝑠𝑥
= sin2 𝑥
− sin2 𝑥−cos2 𝑥
= sin2 𝑥
−(sin2 𝑥+cos2 𝑥 1
= sin2 𝑥
But sin2 𝑥 + cos2 𝑥 = 1 , 𝑐𝑠𝑐𝑥 = 𝑠𝑖𝑛𝑥
𝑑 1
𝑑𝑥
𝑐𝑜𝑡𝑥 = − sin2 𝑥
𝑑
𝑑𝑥
𝑐𝑜𝑡𝑥 = − csc 2 𝑥
𝒅 𝒅𝒖
By chain rule: 𝒅𝒙
𝒄𝒐𝒕𝒖 = − 𝐜𝐬𝐜 𝟐 𝒖 𝒅𝒙

1
5. Derivative of 𝒔𝒆𝒄𝒙 Recall 𝑠𝑒𝑐𝑥 =
𝑐𝑜𝑠𝑥
𝑑 𝑑 1
𝑠𝑒𝑐𝑥 =
𝑑𝑥 𝑑𝑥 𝑐𝑜𝑠𝑥
𝑠𝑖𝑛𝑥
= cos2 𝑥
𝑠𝑖𝑛𝑥 1 𝑠𝑖𝑛𝑥
= But 𝑠𝑒𝑐𝑥 = , 𝑡𝑎𝑛𝑥 =
𝑐𝑜𝑠𝑥𝑐𝑜𝑠𝑥 𝑐𝑜𝑠𝑥 𝑐𝑜𝑠𝑥
𝑑
𝑑𝑥
𝑠𝑒𝑐𝑥 = 𝑠𝑒𝑐𝑥𝑡𝑎𝑛𝑥

𝒅 𝒅𝒖
Chain rule: 𝒅𝒙
𝒔𝒆𝒄𝒖 = 𝒔𝒆𝒄𝒖𝒕𝒂𝒏𝒖 𝒅𝒙

6. Derivative of 𝒄𝒔𝒄𝒙
𝑑𝑢
𝑑 𝑑 1 𝑑 1 −
𝑑𝑥
𝑑𝑥
𝑐𝑠𝑐𝑥 = 𝑑𝑥 𝑠𝑖𝑛𝑥 Differentiate using 𝑑𝑥 𝑢 = 𝑢2
−𝑐𝑜𝑠𝑥 −𝑐𝑜𝑠𝑥 1 𝑐𝑜𝑠𝑥
= sin2 𝑥
= 𝑠𝑖𝑛𝑥𝑠𝑖𝑛𝑥 But 𝑥 = 𝑠𝑖𝑛𝑥 , 𝑐𝑜𝑡𝑥 = 𝑠𝑖𝑛𝑥
𝑑
𝑑𝑥
𝑐𝑠𝑐𝑥 = −𝑐𝑠𝑐𝑥𝑐𝑜𝑡𝑥
𝒅 𝒅𝒖
Chain rule: 𝒅𝒙
𝒄𝒔𝒄𝒖 = −𝒄𝒔𝒄𝒖𝒄𝒐𝒕𝒖 𝒅𝒙

This document is the property of PHINMA EDUCATION


5
Course Code: MAT 171
Course Title: Differential Calculus
Teachers’ Guide Module #18

Name: _____________________________________________________________ Class number: _______


Section: ____________ Schedule:_______________________________________ Date: ______________

The “Chain Rule versions” of the


Derivative of the Six Trigonometric
Functions
𝒅 𝒅𝒖
1. 𝒔𝒊𝒏𝒖 = 𝒄𝒐𝒔𝒖
𝒅𝒙 𝒅𝒙
𝒅 𝒅𝒖
2. 𝒄𝒐𝒔𝒖 = −𝒔𝒊𝒏𝒖
𝒅𝒙 𝒅𝒙
𝒅 𝒅𝒖
3. 𝒕𝒂𝒏𝒖 = 𝐬𝐞𝐜 𝟐 𝒖
𝒅𝒙 𝒅𝒙
𝒅 𝒅𝒖
4. 𝒄𝒐𝒕𝒖 = − 𝐜𝐬𝐜 𝟐 𝒖
𝒅𝒙 𝒅𝒙
𝒅 𝒅𝒖
5. 𝒔𝒆𝒄𝒖 = 𝒔𝒆𝒄𝒖𝒕𝒂𝒏𝒖
𝒅𝒙 𝒅𝒙
𝒅 𝒅𝒖
6. 𝒄𝒔𝒄𝒖 = −𝒄𝒔𝒄𝒖𝒄𝒐𝒕𝒖
𝒅𝒙 𝒅𝒙

It is helpful to notice that the minus sign goes with


the derivative of the “co-functions”:
cosine, cosecant and cotangent.

This document is the property of PHINMA EDUCATION


6
Course Code: MAT 171
Course Title: Differential Calculus
Teachers’ Guide Module #18

Name: _____________________________________________________________ Class number: _______


Section: ____________ Schedule:_______________________________________ Date: ______________

Example 1. Find the derivative of the given function

a. 𝒚 = 𝐬𝐢𝐧(𝟓𝒙)

Solution: we note 𝑦 = sin(5𝑥), takes the form


𝑑𝑢
𝑦 = sin 𝑢 with u = 5x and = 5.
𝑑𝑥
𝑑𝑦 𝑑 𝑑 𝑑𝑢
𝑑𝑥
= cos(5𝑥) 𝑑𝑥 (5𝑥) 𝑑𝑥
𝑠𝑖𝑛𝑢 = 𝑐𝑜𝑠𝑢 𝑑𝑥
𝑑𝑦
= [cos(5𝑥)](5)
𝑑𝑥
𝒅𝒚
= 𝟓𝐜𝐨𝐬(𝟓𝒙)
𝒅𝒙

b. 𝒚 = 𝐜𝐨𝐬(𝟓𝒙)

𝑑𝑦 𝑑 𝑑𝑢
𝑑𝑥
= −[sin(5𝑥)](5) 𝑑𝑥
𝑐𝑜𝑠𝑢 = −𝑠𝑖𝑛𝑢 𝑑𝑥

= −𝟓𝐬𝐢𝐧(𝟓𝒙)

c. 𝒚 = 𝐭𝐚𝐧(𝟓𝒙)

𝑑𝑦 𝑑 𝑑𝑢
= [sec 2(5𝑥)](5) 𝑡𝑎𝑛𝑢 = sec 2 𝑢
𝑑𝑥 𝑑𝑥 𝑑𝑥

= 𝟓 𝐬𝐞𝐜 𝟐 (𝟓𝒙)

d. 𝒚 = 𝐜𝐨𝐭(𝟓𝒙)

𝑑𝑦 𝑑 𝑑𝑢
𝑑𝑥
= −[csc 2 (5𝑥)](5) 𝑑𝑥
𝑐𝑜𝑡𝑢 = − csc 2 𝑢 𝑑𝑥

= −𝟓 𝐜𝐬𝐜 𝟐 (𝟓𝒙)

e. 𝒚 = 𝐬𝐞𝐜(𝟓𝒙)

𝑑𝑦 𝑑 𝑑𝑢
𝑑𝑥
= [sec(5𝑥)tan(5𝑥)](5) 𝑑𝑥
𝑠𝑒𝑐𝑢 = 𝑠𝑒𝑐𝑢𝑡𝑎𝑛𝑢 𝑑𝑥

= 𝟓 𝐬𝐞𝐜(𝟓𝒙) 𝐭𝐚𝐧(𝟓𝒙)

𝑑 𝑑𝑢
f. 𝒚 = 𝐜𝐬𝐜(𝟓𝒙) = 𝑐𝑠𝑐𝑢 = −𝑐𝑠𝑐𝑢𝑐𝑜𝑡𝑢
𝑑𝑥 𝑑𝑥
𝒅𝒚
𝒅𝒙
= −𝟓 𝐜𝐬𝐜(𝟓𝒙) 𝒄𝒐𝒕(𝟓𝒙)

This document is the property of PHINMA EDUCATION


7
Course Code: MAT 171
Course Title: Differential Calculus
Teachers’ Guide Module #18

Name: _____________________________________________________________ Class number: _______


Section: ____________ Schedule:_______________________________________ Date: ______________
𝒅𝒚
Example 2. Find the 𝒅𝒙 if 𝒚 = 𝐬𝐢𝐧𝟑 (𝟓𝒙)
Solution: If we write 𝑦 = sin3 (5𝑥) as 𝑦 = [sin (5x)]3
then it takes the form 𝑦 = 𝑢𝑛 with 𝑢 = sin(5𝑥) and n=3.
This suggest the use of “ the power rule”.
𝑑𝑦 𝑑 𝑑 𝑛 𝑑𝑢
𝑑𝑥
= 3[sin2(5𝑥)] 𝑑𝑥 sin(5𝑥) By the power rule 𝑑𝑥
𝑢 = 𝑛𝑢𝑢−𝑛 𝑑𝑥 ”
𝑑
= 3 [sin2(5𝑥)][(cos(5𝑥)] 𝑑𝑥 5𝑥 Apply chain rule
= [3 sin2(5𝑥)][(cos(5𝑥)](5)
= 15 sin2 (5𝑥)cos(5𝑥) Simplify

We may perform some steps mentally and thus shorten our


solution. For instance in example 2, we may omit some steps
given above. Thus in practice, the problem is worked out simply
this way.

𝑦 = sin3 (5𝑥)
𝑑𝑦
𝑑𝑥
= [3 sin2 (5𝑥)[(cos(5𝑥)] (5) Chain rule ,power rule

= 15 sin2(5𝑥)cos(5𝑥)

𝒅𝒚
Example 3. Find 𝒅𝒙 if 𝒚 = 𝐭𝐚𝐧𝟒 (𝟓𝒙) Chain rule , power rule

Solution: 𝑦 = tan4 (5𝑥)


𝑑𝑦
𝑑𝑥
= 4 tan3 (5𝑥)(sec(5𝑥))(5)

= 𝟐𝟎 𝐭𝐚𝐧𝟑 (𝟓𝒙)𝐬𝐞𝐜(𝟓𝒙)

𝒅𝒚
Example 4. Find the 𝒅𝒙 if 𝒚 = 𝐜𝐨𝐬(𝟑𝒙)𝟐

Solution:
𝑦 = cos(3𝑥)2 Write the original equation
2
𝑦 = cos(9𝑥 )
𝑑𝑦 𝑑 𝑑𝑢
𝑑𝑥
= −(sin(9𝑥 2 ))(18𝑥) From 𝑑𝑥 𝑐𝑜𝑠𝑢 = −𝑠𝑖𝑛𝑢 𝑑𝑥

= −(𝟏𝟖𝒙)𝒔𝒊𝒏(𝟗𝒙𝟐 ) Simplify

This document is the property of PHINMA EDUCATION


8
Course Code: MAT 171
Course Title: Differential Calculus
Teachers’ Guide Module #18

Name: _____________________________________________________________ Class number: _______


Section: ____________ Schedule:_______________________________________ Date: ______________
𝒅𝒚
Example 5. Find 𝒅𝒙 for each of the following functions.
a.) 𝒚 = 𝐬𝐢𝐧𝟑 (𝒙𝟑 + 𝟏)
𝑑 𝑑𝑢
By the general power formula 𝑑𝑥
𝑢𝑛 = n𝑢𝑛−1 𝑑𝑥
we have,
𝑑𝑦 𝑑 𝑑𝑢
𝑑𝑥
= 3[sin2(𝑥 3 + 1)][cos(𝑥 3 + 1)](3𝑥 2 ) 𝑑𝑥
𝑠𝑖𝑛𝑢 = 𝑐𝑜𝑠𝑢 𝑑𝑥
= 𝟗𝒙𝟐 𝐬𝐢𝐧𝟐 (𝒙𝟑 + 𝟏)𝐜𝐨𝐬(𝒙𝟑 + 𝟏).

b.) 𝒚 = √𝟏 + 𝐬𝐞𝐜(𝟑𝒙)
𝑑 𝑑𝑢
𝑑𝑦 [1+sec(3𝑥)] 𝑑
𝑑𝑥
= 𝑑𝑥 𝑑𝑥
√𝑢 = 𝑑𝑥
2√𝑢
2√1+sec(3𝑥)

[sec(3𝑥) tan(3𝑥)](3) 𝑑 𝑑𝑢
= 𝑑𝑥
𝑠𝑒𝑐𝑢 = 𝑠𝑒𝑐𝑢𝑡𝑎𝑛𝑢 𝑑𝑥
2√1+sec(3𝑥)

𝟑 𝐬𝐞𝐜(𝟑𝒙)𝐭𝐚𝐧(𝟑𝒙)
=
𝟐√𝟏+𝐬𝐞𝐜(𝟑𝒙)

Example 6. Find y'' if 𝒔𝒊𝒏𝒚 + 𝒄𝒐𝒔𝒙 = 𝟏


𝑠𝑖𝑛𝑦 + 𝑐𝑜𝑠𝑥 = 1 Apply implicit differentiation
(𝑐𝑜𝑠𝑦)𝑦′ + (−𝑠𝑖𝑛𝑥) = 0 Differentiate both sides of the equation
𝑠𝑖𝑛𝑥
𝑦′ = 𝑐𝑜𝑠𝑦
(𝑐𝑜𝑠𝑦)(𝑐𝑜𝑠𝑥)−𝑠𝑖𝑛𝑥(−𝑠𝑖𝑛𝑦)
𝑦′′ = cos2 𝑦
𝑦′
𝑠𝑖𝑛𝑥
[𝑐𝑜𝑠𝑦𝑐𝑜𝑥+𝑠𝑖𝑛𝑥𝑠𝑖𝑛𝑦]( )
𝑠𝑖𝑛𝑦
𝑦′′ = cos2 𝑦
sin2 𝑥𝑠𝑖𝑛𝑦
𝑐𝑜𝑠𝑦𝑐𝑜𝑠𝑥+
𝑐𝑜𝑠𝑦
= cos2 𝑦

𝐜𝐨𝐬 𝟐 𝒚𝒄𝒐𝒔𝒙+𝐬𝐢𝐧𝟐 𝒙𝒔𝒊𝒏𝒚


= 𝐜𝐨𝐬 𝟑 𝒚

As we become familiar with the formulas and


their uses, we can perform some steps mentally
and will shorten our solution.

This document is the property of PHINMA EDUCATION


9
Course Code: MAT 171
Course Title: Differential Calculus
Teachers’ Guide Module #18

Name: _____________________________________________________________ Class number: _______


Section: ____________ Schedule:_______________________________________ Date: ______________

Application of Maxima and Minima Involving Trigonometric Functions


Although problems may be solved algebraically, another method of solving problems in maxima
and minima is to express the function in terms of trigonometric functions.

The basic idea is the same. Identify the constant terms and identify the variable to be
maximized or to be minimized, differentiate that variable then equate to zero.

Example 1. Find the shape of the rectangle of maximum perimeter


inscribed in a circle.

Solution:
Let P – Perimeter of rectangle
Recall mnemonics Soh–Cah–Toa
P = 2x + 2y
𝑦
𝑦 = 𝐷 sin 𝛳 Where: sin 𝛳 = 𝐷
𝑥
𝑥 = 𝐷 cos 𝛳 cos 𝛳 = 𝐷
P = 2(𝐷𝑐𝑜𝑠𝛳) + 2(𝐷 sin 𝛳) Substitute
= 2𝐷𝑐𝑜𝑠𝛳 + 2𝐷 sin 𝛳

Differentiate both sides


𝑑𝑃 𝑑𝑃
𝑑𝛳
= 2𝐷(− sin 𝛳) + 2𝐷 cos 𝛳 To maximize, Set 𝑑𝛳
=0
sin 𝛳
−2𝐷 sin 𝛳 + 2𝐷 cos 𝛳 = 0 − sin 𝛳 + cos 𝛳 = 0, sin 𝛳 = cos 𝛳, cos 𝛳
=1

sin 𝛳
Therefore: cos 𝛳
= tan 𝛳 = 1 𝛳 = 45°
𝑦 = 𝐷 sin 𝛳 = 𝐷 sin 45° = 0.707 𝐷
𝑥 = 𝐷 cos 𝛳 = 𝐷 cos 45° = 0.707𝐷

Since x = y , therefore it is a square.

This document is the property of PHINMA EDUCATION


10
Course Code: MAT 171
Course Title: Differential Calculus
Teachers’ Guide Module #18

Name: _____________________________________________________________ Class number: _______


Section: ____________ Schedule:_______________________________________ Date: ______________

Example 2. A cylinder is inscribed in a given sphere. Find the shape of the cylinder
if the convex surface area is a maximum.

Solution:
Let: A = Convex surface area
A = 𝜋 dh

ℎ = 𝐷 sin 𝛳 But sin 𝛳 =
𝐷
𝑥
𝑥 = 𝐷 cos 𝛳 cos 𝛳 = 𝐷
A = 𝜋 ( D cos 𝛳 )(𝐷 sin 𝛳 ) Substitute
𝐴 = 𝜋𝐷2 cos 𝛳 sin 𝛳

Differentiate by Product Rule


𝑑𝐴
= 𝜋𝐷 2 [cos 𝛳 cos 𝛳 + sin 𝛳(− sin 𝛳)]
𝑑𝛳
𝑑𝐴
𝑑𝛳
= 𝜋𝐷 2 [cos 𝛳2 − sin2 𝛳]

𝑑𝐴
To maximize, Set 𝑑𝜃
=0
𝜋𝐷 2 (cos 𝛳2 − sin2 𝛳) = 0
cos2 𝛳 = sin2 𝛳
cos 𝛳 = sin 𝛳
sin 𝛳
=1
cos 𝛳
tan 𝛳 = 1 𝛳 = 45°
ℎ = 𝐷 sin 𝛳 = 𝐷 sin 45° = 0.707 𝐷
𝑑 = 𝐷 cos 𝜃 = 𝐷 cos 45° = 0.707𝐷

Therefore, the diameter of the base of the cylinder is equal to the height of the cylinder
in order to be maximum

This document is the property of PHINMA EDUCATION


11
Course Code: MAT 171
Course Title: Differential Calculus
Teachers’ Guide Module #18

Name: _____________________________________________________________ Class number: _______


Section: ____________ Schedule:_______________________________________ Date: ______________

Activity 3: Skill-building Activities (with answer key) SOLVE THE FOLLOWING


PROBLEMS
Find the derivative of the given function:
1
1. 𝑦 = − 3 cot 3 (2𝑥) + cot(2𝑥) + 2𝑥
𝑠𝑒𝑐𝑥
2. 𝑦 = 1+𝑡𝑎𝑛𝑥

3. Find 𝑦 (4) if 𝑦 = sin(5𝑥)


4. The stiffness of a rectangular beam is proportional
to the breadth and the cube of the depth. Find the
shape of the stiffest beam that can be cut from a
log of a given size.

Activity 4: What I Know Chart, part 2


What I Learned
1. ______________________________________________________________________________
______________________________________________________________________________
2. ______________________________________________________________________________
______________________________________________________________________________
3. ______________________________________________________________________________
______________________________________________________________________________

Activity 5: Check for Understanding

Find the first derivative


1. 𝑦 = cot 5 (7𝑥)
2. 𝑦 = 𝑠𝑒𝑐 4 𝑥 − tan4 𝑥
3. Find the largest conical tent that can be constructed having a given slant height.

C. LESSON WRAP-UP

Activity 6: Thinking about Learning


A. Work Tracker
You are done with this session! Let’s track your progress. Shade the session number you just
completed.

This document is the property of PHINMA EDUCATION


12
Course Code: MAT 171
Course Title: Differential Calculus
Teachers’ Guide Module #18

Name: _____________________________________________________________ Class number: _______


Section: ____________ Schedule:_______________________________________ Date: ______________

B. Think about your Learning

1. What motivated you to finish the lesson today?


________________________________________________________________________________________
________________________________________________________________________________________
2. What could you have done better to improve your learning today?
________________________________________________________________________________________
________________________________________________________________________________________

FAQs
1.What type of function is a transcendental function?
A transcendental function cannot be expressed in terms of a finite sequence of the algebraic
operations of addition, multiplication and root extraction. Examples of transcendental functions
include the exponential function, the logarithm and the trigonometric functions.

2. What is meant by transcendental equation?

A transcendental equation is an equation containing a transcendental function of the variable


being solved for. Such equations often do not have closed-form solutions.

3. Why is trigonometry so hard?

In fact it's rather basic mathematics. However if all you know is arithmetic and basic linear
algebra then, yes trigonometry seems more difficult. The reason is that unlike linear algebra
(solving linear equations) trigonometry is non-linear which makes the identities between
multiples of angles non-trivial.

4. What is the rule of a right angled triangle?

A right triangle consists of two legs and a hypotenuse. The two legs meet at a 90° angle and the
hypotenuse is the longest side of the right triangle and is the side opposite the right angle. The
Pythagorean Theorem tells us that the relationship in every right triangle is: a2 + b2 = c2.

This document is the property of PHINMA EDUCATION


13
Course Code: MAT 171
Course Title: Differential Calculus
Teachers’ Guide Module #18

Name: _____________________________________________________________ Class number: _______


Section: ____________ Schedule:_______________________________________ Date: ______________

KEY TO CORRECTIONS

Activity 3. Find the derivative


𝑑𝑦
1. 𝑑𝑥
= 2 cot 4(2𝑥)
𝑑𝑦 𝑠𝑒𝑐𝑥(𝑡𝑎𝑛𝑥−1)
2. 𝑑𝑥
= (1+𝑡𝑎𝑛𝑥)2

3. 𝑦 (4) = 625sin(5𝑥)
4. 𝑑 = √3𝑏

Activity 5. Find the first derivative


1. 𝑦′ = −35 cot 4(7𝑥) csc 2(7𝑥)
𝑑𝑦
2. 𝑑𝑥
= 4 sec 2 𝑥𝑡𝑎𝑛𝑥
√2
3. Radius = x slant height
√3

This document is the property of PHINMA EDUCATION


14
MAT 171: Differential Calculus
Student’s Activity Sheets #14
Name: ______________________________________________________ Class number: ________
Section: ____________ Schedule: _______________________________ Date: _______________

Lesson Title: Inverse Trigonometric Functions Materials: Ballpoint, notebook, calculator


Textbook:
Lesson Objectives: Calculus by Ron Larson, 11th Ed.
At the end of the lesson, you should be able to: References:
1. Define Inverse Trigonometric Functions 1. The Calculus 7 by Louis Leithold
2. Derive and illustrate the derivative of the 2. Differential and Integral Calculus
Inverse Trigonometric Functions by Clyde E. Love and Earl Rainville
3. Solve problems using the derivative of Inverse 3. Calculus 11th Edition by James Stewart
Trigonometric Functions

“A beautiful day begins with a beautiful mind set.


The moment you start acting like life is a blessing,
it will start to feel like one. Train your mind to see
the good in everything.”

A. LESSON PREVIEW/REVIEW

Preview:
Inverse Trigonometric functions are simply defined as the
inverse functions of the basic trigonometric functions (sine,
cosine, tangent, cotangent, secant and cosecant functions).

They are also termed as arcus functions, anti-trigonometric functions or cyclometric functions. These
inverse functions in trigonometry are used to get the angle with any of the trigonometry ratios .

GOOD DAY! The following topics shall be accomplished


In this module:
1. Define, derive and illustrate Inverse Trigonometric Functions
2. Solve problems involving derivative of Inverse Trigonometric Functions

Activity 1: What I Know Chart, part 1

What I Know Questions: What I Learned (Activity 4)


1. Will you define Inverse
Trigonometric Functions?

2. Can you derive and illustrate


Inverse Trigonometric Functions?

3. Do you know how to solve


problems involving derivative of
Inverse Trigonometric Functions?

This document is the property of PHINMA EDUCATION


1
MAT 171: Differential Calculus
Student’s Activity Sheets #14
Name: ______________________________________________________ Class number: ________
Section: ____________ Schedule: _______________________________ Date: _______________

B.MAIN LESSON
Activity 2: Content Notes

Inverse Trigonometric Functions

The trigonometric functions are single-valued functions, while the inverse trigonometric functions are
multi-valued functions. This means that when an angle is given, its sine is uniquely determined. On
the other hand, if the sine is given, the angle is not uniquely determined. For example,
𝜋
y = sin = 1, y = arcsin 0 = 0, 𝜋, 2 𝜋, 3 𝜋 . . .
2

𝜋 𝜋
We recall from trigonometry, that 𝑦 = 𝑎𝑟𝑐𝑠𝑖𝑛𝑥 if and only if 𝑥 = 𝑠𝑖𝑛𝑦 and − 2 ≤ 𝑦 ≤ 2
. Note that
𝜋 𝜋
without restricting the values of y in the interval [- 2 , 2 ],
the equation 𝑦 = 𝑎𝑟𝑐𝑠𝑖𝑛𝑥 does not define a
function. The reason for this is that, for any value of x in the interval [-1, 1], those are infinitely many
values of y which will satisfy the equation 𝑦 = 𝑎𝑟𝑐𝑠𝑖𝑛𝑥.

However, with the restriction, we see that for each value of x in [-1, 1], there is a unique value of y. For
instance,

1 𝜋 𝜋
𝑦 = 𝑎𝑟𝑐𝑠𝑖𝑛 ( ) = , 𝑦 = arcsin(−1) = −
2 6 2

Definition of Inverse Trigonometric Functions are as follows:

Function Domain Range


𝜋 𝜋
1. 𝑦 = arcsin 𝑥 , if and only if sin 𝑦 = 𝑥 −1 ≤ 𝑥 ≤ 1 − ≤𝑦≤
2 2

2. 𝑦 = arccos 𝑥 , if and only if cos 𝑦 = 𝑥 −1 ≤ 𝑥 ≤ 1 0≤𝑦≤𝜋

𝜋 𝜋
3. 𝑦 = arctan 𝑥 , if and only if tan 𝑦 = 𝑥 −∞ ≤ 𝑥 ≤ ∞ − ≤𝑦≤
2 2

4. 𝑦 = arccot 𝑥, if and only if cot 𝑦 = 𝑥 −∞ ≤ 𝑥 ≤ ∞ 0≤𝑦≤𝜋

𝜋
5. 𝑦 = arcsec 𝑥 , if and only if sec 𝑦 = 𝑥 𝑥≥1 0≤𝑦≤
2

𝜋
6. 𝑦 = arccsc 𝑥 , if and only if csc 𝑦 = 𝑥 𝑥≥1 0≤𝑦≤
2

The term “arcsin 𝑥” is read as “arcsine of x” or sometimes “ the angle whose sine is x”, The
notation sin−1 𝑥 is often used for 𝑎𝑟𝑐𝑠𝑖𝑛𝑒 but in this module we shall use the “arc” notation.

This document is the property of PHINMA EDUCATION


2
MAT 171: Differential Calculus
Student’s Activity Sheets #14
Name: ______________________________________________________ Class number: ________
Section: ____________ Schedule: _______________________________ Date: _______________

The Graph of Inverse Trigonometric Functions

Note:
“When evaluating inverse trigonometric functions, remember that they denote
angles in radians measure.”

This document is the property of PHINMA EDUCATION


3
MAT 171: Differential Calculus
Student’s Activity Sheets #14
Name: ______________________________________________________ Class number: ________
Section: ____________ Schedule: _______________________________ Date: _______________

Derivative of Inverse Trigonometric Functions


a) Derivative of 𝐚𝐫𝐜 𝐬𝐢𝐧 𝒖

It will be more convenient to assume 𝑦 = arcsin 𝑥.

Where: 𝑠𝑖𝑛𝑦 = 𝑥

Differentiating in its implicit function, we find

𝑑𝑦
𝑐𝑜𝑠𝑦 𝑑𝑥
=1

𝑑𝑦 1
= 𝑐𝑜𝑠𝑦
𝑑𝑥

Because y = arcsin x, you know that sin y = x. This relationship between x and y can be
represented by a right triangle.

𝑜𝑝𝑝 𝑥
Sin y = ℎ𝑦𝑝
=1
𝐴𝑑𝑗 √1− 𝑥2
Cos y = =
ℎ𝑦𝑝 1

𝑑𝑦 1
Thus, = 𝑐𝑜𝑠𝑦,
𝑑𝑥

𝒅𝒚 𝟏
=
𝒅𝒙 √𝟏−𝒙𝟐

However, if u is any differentiable function of x, we then find


𝒅𝒖
𝒅 𝒅𝒙
𝒂𝒓𝒄 𝒔𝒊𝒏 𝒖 =
𝒅𝒙 √𝟏 − 𝒖𝟐

This document is the property of PHINMA EDUCATION


4
MAT 171: Differential Calculus
Student’s Activity Sheets #14
Name: ______________________________________________________ Class number: ________
Section: ____________ Schedule: _______________________________ Date: _______________

b) Derivative of 𝐚𝐫𝐜 𝐜𝐨𝐬 𝒖

Now let us take a look at the inverse cosine. We may assume again that

𝑦 = arccos 𝑥.
Where cos 𝑦 = 𝑥

Differentiating in its implicit function, we find

𝑑𝑦
−𝑠𝑖𝑛𝑦 𝑑𝑥 = 1
𝑑𝑦 1
𝑑𝑥
= − 𝑠𝑖𝑛𝑦

From cos y = x, draw the right triangle

√1−𝑥 2
Where Sin y =
1

𝑑𝑦 1
thus, = − 𝑠𝑖𝑛𝑦
𝑑𝑥

𝑑𝑦 1
𝑑𝑥
=−
√1−𝑥2

If u is any differentiable function of x we have,

𝒅𝒖
𝒅
𝒂𝒓𝒄 𝒄𝒐𝒔 𝒖 = − 𝒅𝒙
𝒅𝒙 √𝟏 − 𝒖𝟐

This document is the property of PHINMA EDUCATION


5
MAT 171: Differential Calculus
Student’s Activity Sheets #14
Name: ______________________________________________________ Class number: ________
Section: ____________ Schedule: _______________________________ Date: _______________

c. Derivative of 𝐚𝐫𝐜 𝐭𝐚𝐧 𝒖

In determining the derivative of arctan 𝑢, a similar approach must be done

Let 𝑦 = arctan 𝑥
tan 𝑦 = 𝑥

Differentiate
𝑑𝑦
sec 2 𝑦 𝑑𝑥 = 1

𝑑𝑦 1
𝑑𝑥
= sec2 𝑦

Draw the right triangle

𝑥 𝑜𝑝𝑝
From 𝑡𝑎𝑛𝑦 = =
1 𝑎𝑑𝑗

1
Therefore 𝑐𝑜𝑠𝑦 =
√1+𝑥 2

1 2 1 1
cos2 𝑦 = ( 2
) → sec2 𝑦 = 1+𝑥2
√1+𝑥

𝑑𝑦 1
Thus, 𝑑𝑥
= sec2 𝑦

𝑑𝑦 1
𝑑𝑥
= 1+𝑥2

If u is any differentiable function of x we have,

𝒅𝒖
𝒅
𝐚𝐫𝐜𝐭𝐚𝐧 𝒖 = 𝒅𝒙 𝟐
𝒅𝒙 𝟏+𝒖

This document is the property of PHINMA EDUCATION


6
MAT 171: Differential Calculus
Student’s Activity Sheets #14
Name: ______________________________________________________ Class number: ________
Section: ____________ Schedule: _______________________________ Date: _______________

Derivative of the Inverse Trigonometric Functions


Let u be a differentiable function of x
𝒅𝒖
𝒅 𝒅𝒙
1. 𝐚𝐫𝐜𝐬𝐢𝐧 𝒖 =
𝒅𝒙 √𝟏−𝒖𝟐
𝒅𝒖
𝒅 𝒅𝒙
2. 𝐚𝐫𝐜𝐜𝐨𝐬 𝒖 = −
𝒅𝒙 √𝟏+𝒖𝟐
𝒅𝒖
𝒅 𝒅𝒙
3. 𝐚𝐫𝐜𝐭𝐚𝐧 𝒖 =
𝒅𝒙 𝟏+𝒖𝟐
𝒅𝒖
𝒅 𝒅𝒙
4. 𝐚𝐫𝐜𝐜𝐨𝐭 𝒖 = − u>0
𝒅𝒙 𝟏+𝒖𝟐
𝒅𝒖
𝒅 𝒅𝒙
5. 𝐚𝐫𝐜𝐬𝐞𝐜 𝒖 = u>1
𝒅𝒙 |𝒖|√𝒖𝟐 −𝟏
𝒅𝒖
𝒅 𝒅𝒙
6. 𝐚𝐫𝐜𝐜𝐬𝐜 𝒖 = − u>1
𝒅𝒙 |𝒖|√𝒖𝟐 −𝟏

This document is the property of PHINMA EDUCATION


7
MAT 171: Differential Calculus
Student’s Activity Sheets #14
Name: ______________________________________________________ Class number: ________
Section: ____________ Schedule: _______________________________ Date: _______________

Example: 1. Find the derivative of the given functions.

a) 𝒚 = 𝐚𝐫𝐜𝐬𝐢𝐧(𝟓𝒙)
𝑑𝑢
𝑑𝑢 𝑑 𝑑𝑥
Let u = 5x , =5 From 𝑑𝑥 arcsin 𝑢 =
𝑑𝑥 √1−𝑢2

𝑑𝑦 5
=
𝑑𝑥 √1−(5𝑥)2
𝑑𝑦 5
=
𝑑𝑥 √1−25𝑥 2

b) 𝒚 = 𝐚𝐫𝐜𝐜𝐨𝐬(𝟓𝒙)
𝑑𝑢
𝑑𝑦 5 𝑑𝑢 𝑑 𝑑𝑥
=− Let 𝑢 = 5𝑥 , =5 From arccos 𝑢 =−
𝑑𝑥 √1−(5𝑥)2 𝑑𝑥 𝑑𝑥 √1−𝑢2
5
=−
√1−25𝑥 2

c) 𝒚 = 𝐚𝐫𝐜𝐭𝐚𝐧(𝟓𝒙)
𝑑𝑢
𝑑𝑦 5 𝑑𝑢 𝑑 𝑑𝑥
𝑑𝑥
= 1+(5𝑥)2
Let 𝑢 = 5𝑥 , 𝑑𝑥
=5 From 𝑑𝑥
arctan 𝑢 = 1+𝑢2
5
= 1+25𝑥2

d) 𝒚 = 𝐚𝐫𝐜𝐜𝐨𝐭(𝟓𝒙)
𝑑𝑢
𝑑𝑦 5 𝑑𝑢 𝑑 𝑑𝑥
=− Let 𝑢 = 5𝑥 , =5 From arccot 𝑢 =−
𝑑𝑥 1+(5𝑥)2 𝑑𝑥 𝑑𝑥 1+ 𝑢2
5
= − 1+25𝑥2

e) 𝒚 = 𝐚𝐫𝐜𝐬𝐞𝐜(𝟓𝒙)
𝑑𝑢
𝑑𝑦 5 𝑑𝑢 𝑑 𝑑𝑥
= Let 𝑢 = 5𝑥 , =5 From 𝑑𝑥 arcsec 𝑢 =
𝑑𝑥 (5𝑥)√(5𝑥)2 −1 𝑑𝑥 |𝑢|√𝑢2 −1
1
=
𝑥√25𝑥 2 −1

f) 𝒚 = 𝐚𝐫𝐜𝐜𝐬𝐜(𝟓𝒙)
𝑑𝑢
𝑑𝑦 5 𝑑𝑢 𝑑 𝑑𝑥
=− Let 𝑢 = 5𝑥 , =5 From 𝑑𝑥 arccsc 𝑢 = −
𝑑𝑥 |5𝑥|√(5𝑥)2 −1 𝑑𝑥 |𝑢|√𝑢2 −1
1
=−
𝑥√25𝑥 2 −1

g) 𝒚 = 𝒂𝒓𝒄𝒔𝒊𝒏√𝒙
1
2√𝑥 𝑑𝑢 1
𝑦′= Let 𝑢 = √𝑥, =2
√1−𝑥 𝑑𝑥 √𝑥
1 1
= =
2√𝑥 √1−𝑥 2√𝑥−𝑥 2

This document is the property of PHINMA EDUCATION


8
MAT 171: Differential Calculus
Student’s Activity Sheets #14
Name: ______________________________________________________ Class number: ________
Section: ____________ Schedule: _______________________________ Date: _______________

Example 2. Find the derivative of 𝒚 = 𝐚𝐫𝐜𝐬𝐢𝐧 𝒙 + 𝒙√𝟏 − 𝒙𝟐

Given: 𝑦 = arcsin 𝑥 + 𝑥√1 − 𝑥 2

𝑑𝑦 1 −2𝑥
= √1−𝑥 2 + 𝑥 ( ) + √1 − 𝑥 2 (1)
𝑑𝑥 2√1−𝑥 2

1 𝑥2
= √1−𝑥 2 − √1−𝑥 2 + √1 − 𝑥 2

1−𝑥 2 +1−𝑥 2
= √1−𝑥 2

2−2𝑥 2
= √1−𝑥 2

2(1−𝑥 2 )
= √1−𝑥 2

= 𝟐√𝟏 − 𝒙𝟐

𝒅𝒚
Example 3. If = 𝒂𝒓𝒄𝒕𝒂𝒏𝟒 (𝟑𝒙𝟓 ) , find 𝒅𝒙

Given: 𝑦 = 𝑎𝑟𝑐𝑡𝑎𝑛4 (3𝑥 5 ) apply: General Power rule, Chain rule

𝑑𝑦 𝑑 𝑑 𝑑𝑢
= 𝑑𝑥 arctan4(3𝑥 5 ) 𝑢𝑛 = 𝑛𝑢𝑛−1 𝑑𝑥
𝑑𝑥 𝑑𝑥

𝑑𝑢
𝑑𝑦 3 5 𝑑 5 𝑑 𝑑𝑥
= [(4 arctan (3𝑥 )] (arctan(3𝑥 ) 𝑎𝑟𝑐𝑡𝑎𝑛𝑢 =
𝑑𝑥 𝑑𝑥 𝑑𝑥 1+𝑢2

15𝑥 4 𝑑𝑢
= [4 arctan3 (3𝑥 5 )][1+(3𝑥 5 )2 ] Let 𝑢 = 3𝑥 5 , = 15𝑥 4
𝑑𝑥

𝟔𝟎𝒙𝟒 𝐚𝐫𝐜𝐭𝐚𝐧𝟑 (𝟑𝒙𝟓 )


= 𝟏+𝟗𝒙𝟏𝟎

This document is the property of PHINMA EDUCATION


9
MAT 171: Differential Calculus
Student’s Activity Sheets #14
Name: ______________________________________________________ Class number: ________
Section: ____________ Schedule: _______________________________ Date: _______________

Example 4. If = (𝟏 + 𝒙𝟐 )𝒂𝒓𝒄𝒕𝒂𝒏𝒙 − 𝒙 , find y'

Given: 𝑦 = (1 + 𝑥 2 )𝑎𝑟𝑐𝑡𝑎𝑛𝑥 − 𝑥

𝑑𝑦 𝑑 𝑑 𝑑
= (1 + 𝑥 2 ) 𝑑𝑥 𝑎𝑟𝑐𝑡𝑎𝑛𝑥 + (𝑎𝑟𝑐𝑡𝑎𝑛𝑥) 𝑑𝑥 (1 + 𝑥 2 ) − 𝑑𝑥 (𝑥)
𝑑𝑥

1
= (1 + 𝑥 2 ) (1+𝑥 2 ) + (𝑎𝑟𝑐𝑡𝑎𝑛𝑥)(2𝑥) − 1

= 𝟐𝒙 𝒂𝒓𝒄𝒕𝒂𝒏𝒙

𝒙
Example 5. If = 𝟐𝒂𝒓𝒄𝒔𝒊𝒏√𝟐 , find y'

𝑥 𝑥 √𝑥 1
Given:𝑦 = 2𝑎𝑟𝑐𝑠𝑖𝑛√2 Let 𝑢 = √2 = = √𝑥
√2 √2

𝑑𝑢 1 1 1
= ∙2 = 2√2
𝑑𝑥 √2 √𝑥 √𝑥

1
2√2√𝑥
=2 2
√1−(√𝑥)
[ 2 ]
1
√2√𝑥
= 𝑥
√1−2

1
√2√𝑥
= √2−𝑥
√2

1 √2
= ∙
√2√𝑥 √2−𝑥
1
=
√2𝑥−𝑥 2
𝒅𝒚 𝟏
=
𝒅𝒙 √𝟐𝒙−𝒙𝟐

This document is the property of PHINMA EDUCATION


10
MAT 171: Differential Calculus
Student’s Activity Sheets #14
Name: ______________________________________________________ Class number: ________
Section: ____________ Schedule: _______________________________ Date: _______________

Example 6. A statute 3m high is standing on a base of 4m high. If an observer’s eye is 1.5m


above the ground, how far should he stand from the base in order that the angle by the
statute is maximum?

Required: Distance from the base to the observer.

Let x be the distance from the base to the observer

Solution: 𝛳 = 𝐴 − 𝐵
5.5 5.5 2.5 2.5
From the figure: tan 𝐴 = 𝑥
, 𝐴 = arctan 𝑥
tan 𝐵 = 𝑥
, 𝐵 = arctan 𝑥

From 𝛳 =𝐴−𝐵
5.5 2.5
𝛳 = arctan 𝑥
− arctan 𝑥

Differentiate 𝛳 with respect to x.


− 5.5 2.5
𝑑𝛳 − 2 𝑑𝛳
𝑥2 𝑥
= 5.5 2
− 2.5 2
Set 𝑑𝑥
=0
𝑑𝑥 1+( ) 1+( )
𝑥 𝑥

−5.5 2.5
𝑥2 𝑥2
0= +
𝑥2 +(5.5)2 𝑥2 +(2.5)2
𝑥2 𝑥2

−5.5 2.5
0= + 2
𝑥 2 +30.25 𝑥 +6.25

5.5 2.5
𝑥 2 +30.25
= 𝑥 2 +6.25

5.5𝑥 2 + 34.375 = 2.5𝑥 2 + 75.625

(5.5 − 2.5)𝑥 2 = 75.625 − 34.375


41.25
𝑥2 = 3

𝒙 = 𝟑. 𝟕𝟎𝟖𝟎 the distance from the base to the observer

This document is the property of PHINMA EDUCATION


11
MAT 171: Differential Calculus
Student’s Activity Sheets #14
Name: ______________________________________________________ Class number: ________
Section: ____________ Schedule: _______________________________ Date: _______________

Activity 3: Skill-building Activities (with answer key)


DIFFERENTIATE!
Find the derivative of the given functions.
1 𝑑𝑦
1. 𝑦 = 𝑥√1 − 4𝑥 2 + 2 arcsin(2𝑥); Find 𝑑𝑥

𝑑𝑦
2. If = arcsin4 (3𝑥 5 ) ; find 𝑑𝑥

Activity 4: What I Know Chart, part 2


What I Learned
1. ______________________________________________________________________________
______________________________________________________________________________
2. ______________________________________________________________________________
______________________________________________________________________________
3. ______________________________________________________________________________
______________________________________________________________________________

Activity 5: Check for Understanding


.
𝑥 𝑥
1.Find the derivative of 𝑦 = − 𝑎𝑟𝑐𝑠𝑖𝑛 2
√4−𝑥 2

C. LESSON WRAP-UP

Activity 6: Thinking about Learning


A. Work Tracker
You are done with this session! Let’s track your progress. Shade the session number you just
completed.

B. Think about your Learning


1. What motivated you to finish the lesson today?
_____________________________________________________________________________________
_____________________________________________________________________________________
2. What could you have done better to improve your learning today?
_____________________________________________________________________________________
_____________________________________________________________________________________

This document is the property of PHINMA EDUCATION


12
MAT 171: Differential Calculus
Student’s Activity Sheets #14
Name: ______________________________________________________ Class number: ________
Section: ____________ Schedule: _______________________________ Date: _______________

FAQs
1.What does inverse mean in trigonometry?

In mathematics, there is always a way to calculate the opposite of an operation, or


an inverse. For trigonometric functions, the opposite is inverse trigonometric functions. When
two of the three side lengths of a right triangle are known, the inverse trigonometric function is
used to find the measure of an angle.

2. What are the uses of inverse trigonometric functions?

The inverse trigonometric functions are used to determine the angle measure when at
least two sides of a right triangle are known. The particular function that should be used
depends on what two sides are known.

KEY TO CORRECTIONS

Activity 3:
𝒅𝒚
1. 𝒅𝒙 = 𝟐√𝟏 − 𝟒𝒙𝟐

𝒅𝒚 𝟔𝟎𝒙𝟒 𝐚𝐫𝐜𝐬𝐢𝐧𝟒 (𝟑𝒙𝟓 )


2. 𝒅𝒙
=
√𝟏−𝟗𝒙𝟏𝟎

Activity 5.
𝒅𝒚 𝒙𝟐
1. 𝒅𝒙
= 𝟑
(𝟒−𝒙𝟐 )𝟐

This document is the property of PHINMA EDUCATION


13
MAT 171: Differential Calculus
Student’s Activity Sheet #15
Name: ______________________________________________________ Class number: ________
Section: ____________ Schedule: _______________________________ Date: _______________

Lesson Title: Derivative of Exponential and Materials: Ballpoint, Notebook, Calculator


Logarithmic Functions
Lesson Objectives: Textbook: Calculus by Ron Larson , 11th Ed
At the end of the lesson, you should be able to: References:
1. Define and State the Properties of 1.The Calculus 7 by Louis Leithold
Exponential and Logarithmic Functions 2. Calculus 7th Edition by James Stewart
2. Find the Derivative of Exponential and 3. Differential and Integral Calculus
Logarithmic Functions by Clyde E. Love and Earl Rainville

“Your attitude determines your direction.


Set a goal, work hard and never give up.”
A. LESSON PREVIEW/REVIEW

Introduction

GOOD DAY! The following topics shall be accomplished in


this module:
1. Properties of Exponential and Logarithmic Functions
2. Derivative of Exponential and Logarithmic Functions

Activity 1: What I Know Chart, part 1

What I Know Questions: What I Learned (Activity 4)


1. Can you define and state the
properties of exponential and
logarithmic functions?

2. Do you know how to find the


derivative of exponential and
logarithmic functions?

B.MAIN LESSON

Activity 2: Content Notes

Exponential and Logarithmic Functions


The function defined by the equation 𝑦 = log 𝑏 𝑥 where 𝑏 > 0 and 𝑏 ≠ 1 is called a logarithmic
function. We recall that 𝑦 = log 𝑏 𝑥 and 𝑥 = 𝑏 𝑦 are equivalent equations since they define the
same functions.

This document is the property of PHINMA EDUCATION


1
MAT 171: Differential Calculus
Student’s Activity Sheet #15
Name: ______________________________________________________ Class number: ________
Section: ____________ Schedule: _______________________________ Date: _______________

The function 𝑦 = 𝑎 𝑥 , 𝑎 > 0 is called an exponential function. The graph of the exponential as shown
in Figure 1 has the following properties: is one –valued function, continuous, y-intercept is equal to 1,
no negative value for any x, and having the asymptote as x-axis.

Logarithm to the base 10 are called common logarithm.

𝐲 = 𝐥𝐨𝐠 𝟏𝟎 𝒙
𝒚 = 𝐥𝐨𝐠 𝒙
If b = e, then from y = log10 𝑥 becomes 𝑦 = log 𝑒 𝑥 is written as 𝑦 =
ln 𝑥 where the symbol “𝑙𝑛” is customarily used in place of “𝑙𝑜𝑔𝑒 ”
logarithmic to the base 𝑒 are called Natural Napierian Logarithm.

The equation 𝑦 = ln 𝑥 and 𝑥 = 𝑒 𝑦 are also equivalent equations.

Note: 𝒚 = 𝐥𝐨𝐠 𝒆 𝒙 = 𝐥𝐧 𝒙
FIGURE 1
The modulus M of these functions is given by
𝟏
𝐥𝐨𝐠 𝟏𝟎 𝒆 = 𝑴 = 𝟎. 𝟒𝟑𝟒𝟐𝟗 And 𝐥𝐨𝐠 𝒆 𝟏𝟎 = 𝟐. 𝟑𝟎𝟐𝟓𝟗 = 𝑴
1
From property of logarithm log 𝑎 𝑏 = log
𝑏𝑎
𝟏
Thus, 𝐥𝐨𝐠 𝟏𝟎 𝒆 = 𝐥𝐨𝐠
𝒆 𝟏𝟎

Let 𝑙𝑛𝑥 = log 𝑒 𝑥.Then 𝑙𝑛𝑥 is called natural logarithm of x, see Figure 2.

The domain of log 𝑎 𝑥 is 𝑥 > 0 and the range is the set of real numbers.

For exponential and logarithmic, we will have similar inverse


relationships such as, for general case:

𝒍𝒐𝒈𝒂 (𝒂𝒙 ) = x And 𝒂𝒍𝒐𝒈𝒂 𝒙 = x


FIGURE 2
For the special case
ln (𝒆𝒙 ) = x, 𝒆𝐥𝐧 𝒙 = x

This document is the property of PHINMA EDUCATION


2
MAT 171: Differential Calculus
Student’s Activity Sheet #15
Name: ______________________________________________________ Class number: ________
Section: ____________ Schedule: _______________________________ Date: _______________

In your study of algebra and trigonometry, you learned the laws of exponents, radicals and logarithms.
Some of these laws serve as useful aids in this lesson and we list down here for easy reference.

Laws of Exponents:
E1 𝑎𝑚 𝑎𝑛 = 𝑎𝑚+𝑛
𝑎𝑚
E2 𝑎𝑛
= 𝑎𝑚−𝑛 if 𝑚 > 𝑛

E3 (𝑎𝑚 )𝑛 = 𝑎𝑚𝑛
E4 (𝑎𝑏)𝑛 = 𝑎𝑛 𝑏 𝑛
𝑎 𝑛 𝑎𝑛
E5 (𝑏 ) = 𝑏𝑛 𝑏≠0

Laws of Radicals
𝑛
R1 √𝑎𝑛 = 𝑎
𝑚 𝑚
𝑛 𝑛
R2 √𝑎𝑚 = ( √𝑎) = 𝑎𝑛
𝑛 𝑛 𝑛
R3 √𝑎 √𝑏 = √𝑎𝑏
𝑛
𝑛 𝑎 √𝑎
R4 √𝑏 = 𝑛
√𝑏

𝑚 𝑛 𝑚𝑛
R5 √ √𝑎 = √𝑎

Laws of Logarithms
L1 log 𝑏 𝑀𝑁 = log 𝑏 𝑀 + log 𝑏 𝑁
𝑀
L2 log 𝑏 𝑁 = log 𝑏 𝑀 − log 𝑏 𝑁

L3 log 𝑏 𝑁 𝑃 = 𝑃 log 𝑏 𝑁
L4 log 𝑏 𝑏 = 1
L5 𝑏 log𝑏 𝑁 = 𝑁

Derivative of Logarithm
𝑑𝑢
𝑑
1. log 𝑎 𝑢 = 𝑑𝑥 log 𝑎 𝑒 I𝑓 𝑎 = 10
𝑑𝑥 𝑢

𝑑𝑢
𝑑
2. 𝑑𝑥
log10 𝑢 = 𝑑𝑥
𝑢
∙ log10 𝑒 But 𝑀 = log10 𝑒 = 0.43429
𝑑𝑢
𝑑 𝑑𝑥
𝑑𝑥
log10 𝑢 = 𝑀 𝑢
𝑑𝑢
𝑑
3. 𝑑𝑥
ln 𝑢 = 𝑑𝑥
𝑢

This document is the property of PHINMA EDUCATION


3
MAT 171: Differential Calculus
Student’s Activity Sheet #15
Name: ______________________________________________________ Class number: ________
Section: ____________ Schedule: _______________________________ Date: _______________

Example 1. Differentiate
𝑑𝑢
𝑑 𝑑𝑥
1a) 𝒚 = 𝐥𝐨𝐠 𝟏𝟎 (𝟓𝒙 − 𝟕) From log10 𝑢 = 𝑀
𝑑𝑥 𝑢

Let u = (5x – 7)
𝑑𝑢
𝑑𝑥
=5
𝑑𝑦 5
𝑑𝑥
= (5𝑥−7) (𝑀) But 𝑀 = log10 𝑒 = 0.43429
𝒅𝒚 𝟓𝑴
=
𝒅𝒙 𝟓𝒙−𝟕
𝑑𝑢
𝟐 𝑑 𝑑𝑥
1b) 𝒚 = 𝐥𝐧(𝒙 + 𝟒) From ln 𝑢 =
𝑑𝑥 𝑢
2
Let u = (𝑥 + 4)
𝑑𝑢
𝑑𝑥
= 2x
𝒅𝒚 𝟐𝒙
𝒅𝒙
= (𝒙𝟐+𝟒)

𝟐𝒙−𝟓
Example 2. Differentiate 𝒚 = 𝐥𝐧(𝟑−𝟒𝒙)
𝑑𝑢
2𝑥−5 𝑑 𝑑𝑥
2a) Given: 𝑦 = ln( ) 𝑙𝑛𝑢 =
3−4𝑥 𝑑𝑥 𝑢
25−𝑥 𝑑𝑢
Let 𝑢 = 3−4𝑥
, Solve for 𝑑𝑥 , by quotient rule
𝑑𝑢 (3−4𝑥)(2)−(2𝑥−5)(−4)
= (3−4𝑥)2
𝑑𝑥
𝑑𝑢 6−8𝑥+8𝑥−20
𝑑𝑥
= (3−4𝑥)2
𝑑𝑢
𝑑𝑢 −14 𝑑 𝑑𝑥
𝑑𝑥
= (3−4𝑥)2 Apply the formula 𝑑𝑥 𝑙𝑛𝑢 = 𝑢
−14
𝑑𝑦 (3−4𝑥)2
Thus: 𝑑𝑥
= 2𝑥−5
3−4𝑥

𝑑𝑦 −14 (3−4𝑥)
𝑑𝑥
= (3−4𝑥)2 ∙ 2𝑥−5
𝒅𝒚 −𝟏𝟒
Therefore: 𝒅𝒙
= (𝟑−𝟒𝒙)(𝟐𝒙−𝟓)

2𝑥−5
You can solve the derivative of 𝑦 = ln(3−4𝑥) using the properties of logarithm,
𝑢
log 𝑣 = log 𝑢 − log 𝑣

This document is the property of PHINMA EDUCATION


4
MAT 171: Differential Calculus
Student’s Activity Sheet #15
Name: ______________________________________________________ Class number: ________
Section: ____________ Schedule: _______________________________ Date: _______________
2𝑥−5
2b) Given: 𝑦 = ln(3−4𝑥)

𝑦 = ln(2𝑥 − 5) − ln(3 − 4𝑥)


𝑑𝑢
𝑑𝑦 2 4 𝑑 𝑑𝑥
𝑑𝑥
= (2𝑥−5)
+ (3−4𝑥) From 𝑑𝑥
ln 𝑢 = 𝑢
6−8𝑥+8𝑥−20
= Simplify
(2𝑥−5)(3−4𝑥)
𝒅𝒚 −𝟏𝟒
𝒅𝒙
= (𝟐𝒙−𝟓)(𝟑−𝟒𝒙)
The same answer in problem 2a
𝒅𝒚
Example 3a) Find 𝒅𝒙 , if 𝒚 = 𝐥𝐧(𝟓𝒙 − 𝟕)𝟒 (𝟐𝒙 + 𝟑)𝟑
𝑑𝑢
𝑑
Given: 𝑦 = ln(5𝑥 − 7)4 (2𝑥 + 3)3 From 𝑙𝑛𝑢 = 𝑑𝑥
𝑑𝑥 𝑢
4 (2𝑥 3
Let 𝑢 = (5𝑥 − 7) + 3) Differentiate by product rule
𝑑𝑢
= (5𝑥 − 7)4 3(2𝑥 + 3)2 (2) + (2𝑥 + 3)3 4(5𝑥 − 7)3 (5)
𝑑𝑥

= 6(5𝑥 − 7)4 (2𝑥 + 3)2 + 20(2𝑥 + 3)3 (5𝑥 − 7)3


= 2(5𝑥 − 7)3 (2𝑥 + 3)2 [3(5𝑥 − 7) + 10(2𝑥 + 3)]
= 2(5𝑥 − 7)3 (2𝑥 + 3)2 (15𝑥 − 21 + 20𝑥 + 30) Simplify
𝑑𝑢
= 2(5𝑥 − 7)3 (2𝑥 + 3)2 (35𝑥 + 9)
𝑑𝑥
𝑑𝑢
𝑑𝑦 2(5𝑥−7)3 (2𝑥+3)2 (35𝑥+9) 𝑑 𝑑𝑥
𝑑𝑥
= (5𝑥−7)4 (2𝑥+3)3
From 𝑑𝑥
𝑙𝑛𝑢 = 𝑢
𝒅𝒚 𝟐(𝟑𝟓𝒙+𝟗)
Thus, 𝒅𝒙
= (𝟓𝒙−𝟕)(𝟐𝒙+𝟑)
𝒅𝒚
Example 3b) Another solution in solving 𝒅𝒙 of y = 𝐥𝐧(𝟓𝒙 − 𝟕)𝟒 (𝟐𝒙 + 𝟑)𝟑
Apply the property of logarithm: ln(𝑢𝑣) = ln 𝑢 + ln 𝑣
Given 𝑦 = ln(5𝑥 − 7)4 (2𝑥 + 3)3 Write the original function
𝑦 = ln(5𝑥 − 7)4 +(2𝑥 + 3)3 Rewrite, apply properties of logarithm
𝑦 = 4 ln(5𝑥 − 7) + 3 ln(2𝑥 + 3)
𝑑𝑢
𝑑𝑦 4(5) 6 𝑑
𝑑𝑥
= 5𝑥−7 + 2𝑥+3 From 𝑑𝑥 ln 𝑢 = 𝑑𝑥
𝑢

𝑑𝑦 20 6
𝑑𝑥
= 5𝑥−7
+ 2𝑥+3 Simplify

𝑑𝑦 40𝑥+60+30𝑥−42
𝑑𝑥
= (5𝑥−7)(2𝑥+3)

𝒅𝒚 𝟐(𝟑𝟓𝒙+𝟗)
Therefore: 𝒅𝒙
= (𝟓𝒙−𝟕)(𝟐𝒙+𝟑)

This document is the property of PHINMA EDUCATION


5
MAT 171: Differential Calculus
Student’s Activity Sheet #15
Name: ______________________________________________________ Class number: ________
Section: ____________ Schedule: _______________________________ Date: _______________

In example 2 and 3, be sure to see the benefit of applying logarithmic properties before differentiating.
It is convenient to use logarithms as aids in differentiating non-logarithmic functions. This process is
called logarithmic differentiation

In general, use logarithmic differentiation when


differentiating:
1) a function involving many factors, or
2) a function having both a variable base
and a variable exponent.

Example 4. Find the derivative of 𝒚 = 𝒙𝒙


Given 𝑦 = 𝑥 𝑥
ln 𝑦 = ln 𝑥 𝑥 Take the natural logarithm of both sides
ln 𝑦 = 𝑥𝑙𝑛𝑥 Apply logarithmic property
𝑦′ 1
= 𝑥 ( ) + ln 𝑥 Differentiate implicitly with respect to x
𝑦 𝑥
𝑦′
= 1 + ln 𝑥 Multiply both sides by y.
𝑦

𝒚′ = 𝒙(𝟏 + 𝐥𝐧 𝒙) solve for 𝒚′


Be sure to see that there is now simple differentiation rule for calculating the derivative of 𝒚 = 𝒙𝒙 . In
general when you use 𝑦 = 𝑢(𝑥)𝑣(𝑥) , we need to use logarithmic differentiation.

Derivative of Exponential Functions

Consider the exponential function 𝒚 = 𝒂𝒙


ln 𝑦 = ln 𝑎 𝑥 Take the natural logarithm of both sides
ln 𝑦 = xln 𝑎 Apply logarithmic property
𝑑𝑦
𝑑𝑥
𝑦
= ln 𝑎 Differentiate implicitly with respect to x
𝑑𝑦
= (ln 𝑎)(𝑦) From the given 𝑦 = 𝑎 𝑥
𝑑𝑥

𝒅𝒚
𝒅𝒙
= 𝒂𝒙 𝐥𝐧 𝒂
𝒅 𝒅𝒖
If 𝑢 = 𝑓(𝑥), then by chain rule 𝒂𝒖 = 𝒂𝒖 𝐥𝐧 𝒂
𝒅𝒙 𝒅𝒙

𝒅 𝒅𝒖
If = 𝑒 , then the formula is
𝑑 𝑢
𝑑𝑥
𝑒
𝑑𝑢
= 𝑒 𝑢 ln 𝑒 𝑑𝑥 ; but ln 𝑒 = 1 𝒆𝒖 = 𝒆𝒖
𝒅𝒙 𝒅𝒙

This document is the property of PHINMA EDUCATION


6
MAT 171: Differential Calculus
Student’s Activity Sheet #15
Name: ______________________________________________________ Class number: ________
Section: ____________ Schedule: _______________________________ Date: _______________

Example 1. Find the derivative of the given functions

1a ) 𝒚 = 𝟓𝟐𝒙
𝑑 𝑑𝑢
Let 𝑎 = 5 𝑢 = 2𝑥 From 𝑑𝑥 𝑎𝑢 = 𝑎𝑢 ln 𝑎 𝑑𝑥
𝑑𝑢
𝑑𝑥
=2
𝑑𝑦
𝑑𝑥
= 52𝑥 (ln 5)(2)

= (2 ln 5)52𝑥
= (𝐥𝐧 𝟓𝟐 )(𝟓𝟐𝒙 )

1b) 𝒚 = 𝒆𝟓𝒙
𝑑 𝑑𝑢
Let 𝑢 = 5𝑥 From 𝑑𝑥 𝑒 𝑢 = 𝑒 𝑢 𝑑𝑥
𝑑𝑢
𝑑𝑥
=5
𝑑𝑦
= (𝑒 5𝑥 )(5)
𝑑𝑥

= 𝟓𝒆𝟓𝒙

1c) 𝒚 = 𝒆𝒔𝒊𝒏𝒙
𝑑𝑢 𝑑 𝑑𝑢
Let 𝑢 = 𝑠𝑖𝑛𝑥 , 𝑑𝑥
= 𝑐𝑜𝑠𝑥 From 𝑑𝑥 𝑒 𝑢 = 𝑒 𝑢 𝑑𝑥
𝑑𝑦
= (𝑒 𝑠𝑖𝑛𝑥 )(𝑐𝑜𝑠𝑥)
𝑑𝑥
𝒅𝒚
𝒅𝒙
= (𝐜𝐨𝐬 𝒙)(𝒆𝒔𝒊𝒏𝒙 )

𝒅𝒚
Example 2. If 𝒚 = 𝐜𝐨𝐬 𝟒 𝒆−𝟒𝒙 ,find
𝒅𝒙
𝑑 𝑑𝑢
Given: 𝑦 = cos 4 𝑒 −4𝑥 By chain rule and recall 𝑑𝑥 cos u = (- sin u) 𝑑𝑥
𝑑𝑦 𝑑
𝑑𝑥
= [4 cos4−1 (𝑒 −4𝑥 )] 𝑑𝑥 cos(𝑒 −4𝑥 )

= 4[cos3(𝑒 −4𝑥 )][− sin(𝑒 −4𝑥 )][(𝑒 −4𝑥 )(−4)]


= [4 cos3 (𝑒 −4𝑥 )][− sin(𝑒 −4𝑥 )][(𝑒 −4𝑥 )(−4)]
= 16[cos3 (𝑒 −4𝑥 )[sin(𝑒 −4𝑥 )](𝑒 −4𝑥 ) Simplify
𝒅𝒚
𝒅𝒙
= 𝟏𝟔𝒆−𝟒𝒙 [𝐜𝐨𝐬𝟑 (𝒆−𝟒𝒙 )][𝐬𝐢𝐧(𝒆−𝟒𝒙 )]

This document is the property of PHINMA EDUCATION


7
MAT 171: Differential Calculus
Student’s Activity Sheet #15
Name: ______________________________________________________ Class number: ________
Section: ____________ Schedule: _______________________________ Date: _______________
𝒅𝒚 −𝟐
Example 3.1 Find 𝒅𝒙 , if 𝒚 = 𝒙𝟐 + 𝒙𝟒 𝒆𝐥𝐧 𝒙

−2
Given: 𝑦 = 𝑥 2 + 𝑥 4 𝑒 ln 𝑥 Apply chain rule and product rule
𝑑𝑦 𝑑 2 𝑑 ln 𝑥 −2 ln 𝑥 −2 𝑑 𝑑 𝑢 𝑑𝑢
= 𝑥 + [𝑥 4 𝑒 +𝑒 (𝑥 4 )] From 𝑒 = 𝑒𝑢
𝑑𝑥 𝑑𝑥 𝑑𝑥 𝑑𝑥 𝑑𝑥 𝑑𝑥
𝑑𝑦 −2 −2𝑥 −3 −2
= 2𝑥 + 𝑥 4 (𝑒 ln 𝑥 ) ( )+ (𝑒 ln 𝑥 )(4𝑥 3 )
𝑑𝑥 𝑥 −2
−2 −2 −2
= 2𝑥 + 𝑥 4 (𝑒 ln 𝑥 ) ( 𝑥 ) + 4𝑥 3 𝑒 ln 𝑥
−2 −2
= 2𝑥 − 2𝑥 3 𝑒 ln 𝑥 + 4𝑥 3 𝑒 ln 𝑥
−2 −2 −2
= 2𝑥 + 2𝑥 3 𝑒 ln 𝑥 But 𝑒 ln 𝑥 = 𝑒 log𝑒 𝑥 = 𝑥 −2
= 2𝑥 + 2𝑥 3 (𝑥 −2 )
= 2𝑥 + 2𝑥
𝒅𝒚
𝒅𝒙
= 𝟒𝒙

𝒅𝒚 −𝟐
Example 3.2 Find 𝒅𝒙, if 𝒚 = 𝒙𝟐 + 𝒙𝟒 𝒆𝒍𝒏𝒙

−2 −2
Given: 𝑦 = 𝑥 2 + 𝑥 4 𝑒 𝑙𝑛𝑥 But 𝑒 𝑙𝑛𝑥 = 𝑥 −2
𝑦 = 𝑥 2 + 𝑥 4 𝑥 −2
𝑦 = 𝑥2 + 𝑥2
𝑦 = 2𝑥 2
𝒅𝒚
𝒅𝒙
= 𝟒𝒙

𝒆𝒙
Example 4. Find the first derivative of the function 𝒚 = 𝒙𝟐+𝟏

𝑒𝑥
Given: 𝑦 = 𝑥 2 +1 Write the given and apply quotient rule
𝑑𝑢 𝑑𝑣
𝑑𝑦 (𝑥 2 +1)𝒅(𝑒 𝑥 )−𝑒 𝑥 𝒅(𝑥 2 +1) 𝑑 𝑢 𝑣
𝑑𝑥
−𝑢
𝑑𝑥
𝑑𝑥
= (𝑥 2 +1)2 𝑑𝑥 𝑣
= 𝑣2

𝑑𝑦 (𝑥 2 +1)(𝑒 𝑥 )−𝑒 𝑥 (2𝑥)


= (𝑥 2 +1)2
𝑑𝑥

𝒅𝒚 𝒆𝒙 𝒙𝟐 +𝒆𝒙 −𝟐𝒆𝒙 𝒙
𝒅𝒙
= (𝒙𝟐 +𝟏)𝟐

This document is the property of PHINMA EDUCATION


8
MAT 171: Differential Calculus
Student’s Activity Sheet #15
Name: ______________________________________________________ Class number: ________
Section: ____________ Schedule: _______________________________ Date: _______________
𝒅𝒚 𝒆𝒙 −𝒆−𝒙
Example 5. Find 𝒅𝒙, if 𝒚 = 𝒆𝒙 +𝒆−𝒙

𝑒 𝑥 −𝑒 −𝑥
Given: 𝑦 = 𝑒 𝑥 +𝑒 −𝑥
𝑑𝑦 (𝑒 𝑥 +𝑒 −𝑥 )𝒅(𝑒 𝑥 −𝑒 −𝑥 )−(𝑒 𝑥 −𝑒 −𝑥 )𝒅(𝑒 𝑥 +𝑒 −𝑥 )
= (𝑒 𝑥 +𝑒 −𝑥 )2
Apply quotient rule, chain rule
𝑑𝑥
𝑑𝑦 (𝑒 𝑥 +𝑒 −𝑥 )(𝑒 𝑥 +𝑒 −𝑥 )−(𝑒 𝑥 −𝑒 −𝑥 )(𝑒 𝑥 −𝑒 −𝑥 )
𝑑𝑥
= (𝑒 𝑥 +𝑒 −𝑥 )2

𝑒 2𝑥 +2+𝑒 −2𝑥 −(𝑒 2𝑥 −2+𝑒 −2𝑥 )


= (𝑒 𝑥 +𝑒 −𝑥 )2

𝑒 2𝑥 +2+𝑒 −2𝑥 −𝑒 2𝑥 +2−𝑒 −2𝑥 )


= (𝑒 𝑥 +𝑒 −𝑥 )2
𝒅𝒚 𝟒
𝒅𝒙
= (𝒆𝒙 +𝒆−𝒙 )𝟐 FIND THE FIRST DERIVATIVE!

Activity 3: Skill-building Activities (with answer key)

Find the first derivative


1. 𝑦 = ln √4 − 𝑥 2
𝑦
2. ln(𝑥 2 + 𝑦 2 ) + 4𝑎𝑟𝑐𝑡𝑎𝑛 𝑥 = 5

𝑑𝑦 𝑥+4
3. Find 𝑑𝑥 , if 𝑦 = ln √𝑥−4

Activity 4: What I Know Chart, part 2

What I Learned
1. ________________________________________________________________________________
___________________________________________________________________________________
2. ________________________________________________________________________________
___________________________________________________________________________________
3. ________________________________________________________________________________
___________________________________________________________________________________

Activity 5: Check for Understanding


Find the derivative of y with respect to x of the following. Use any method.
3
1. 𝑦 = ln √4𝑥 2 + 7𝑥
2. 𝑦 = sec 2(𝑒 −4𝑥 )

This document is the property of PHINMA EDUCATION


9
MAT 171: Differential Calculus
Student’s Activity Sheet #15
Name: ______________________________________________________ Class number: ________
Section: ____________ Schedule: _______________________________ Date: _______________

C. LESSON WRAP-UP

Activity 6: Thinking about Learning


A. Work Tracker
You are done with You are done with this session! Let’s track your progress. Shade the
session number you just completed.

1. What motivated you to finish the lesson today?


_____________________________________________________________________________________
_____________________________________________________________________________________
2. What could you have done better to improve your learning today?
_____________________________________________________________________________________
_____________________________________________________________________________________

FAQs

1.What are the uses of exponential functions in real life?

Exponential functions are often used to represent real-world applications, such as


bacterial growth/decay, population growth/decline, and compound interest.

2. What is the importance of logarithms?

Logarithms have a few specific properties that are quite useful in their own
right: Logarithms are a convenient way to express large numbers. The base-10 logarithm of a
number is roughly the number of digits in that number.

KEY TO CORRECTIONS

Activity 3 Activity 5.
𝟏 𝒅𝒚 𝟏 (𝟖𝒙+𝟕)
1. 𝒚′ = − 1. = 𝟑 (𝟒𝒙𝟐+𝟕𝒙)
(𝟒−𝒙𝟐 ) 𝒅𝒙
𝟐𝒚−𝒙 𝒅𝒚
2. 𝒚′ =
𝒚+𝟐𝒙
2. 𝒅𝒙
= −𝟖(𝒆−𝟒𝒙 )(𝐬𝐞𝐜 𝟐 𝒆−𝟒𝒙 ) 𝐭𝐚𝐧 𝒆−𝟒𝒙
𝒅𝒚 −𝟒
3. 𝒅𝒙
= (𝒙𝟐−𝟏𝟔)

This document is the property of PHINMA EDUCATION


10
MAT 171: Differential Calculus
Student’s Activity Sheet #15
Name: ______________________________________________________ Class number: ________
Section: ____________ Schedule: _______________________________ Date: _______________

TEACHER-LED ACTIVITIES
{These are standard instructions for teachers.}
A. If this session happens to be a face-to-face, in-classroom learning session:
1) Collect completed work in the SAS.
2) Allocate your contact time with students to individual or small group mentoring, monitoring,
and student consultations.
3) You may administer summative assessments (quizzes, demonstrations, graded recitation,
presentations, performance tasks) during face-to-face sessions.
4) You may also explore supplementary activities that foster collaboration, provided that social
distancing is observed.
5) You may provide supplementary content via videos, etc.

It is important to remember that students who cannot make it to face-to-face, in-classroom


sessions for health and safety reasons, should not be given lower grades for missing in-class
activities and should be given alternative summative tests.
B. If this session happens to be an at-home learning session for the students:
1) Check and grade collected SAS and other input from students.
2) Schedule phone calls/virtual calls/virtual chats to individual students or small groups of
students to monitor work, provide guidance, answer questions, and check understanding.

This document is the property of PHINMA EDUCATION


2
MAT 171: Differential Calculus
Student’s Activity Sheets #16
Name: ______________________________________________________ Class number: ________
Section: ____________ Schedule: _______________________________ Date: _______________

Lesson Title: The Hyperbolic Functions Materials: Ballpoint, notebook, scientific


calculator
Lesson Objectives: Textbook: Calculus by Ron Larson, 11th Ed.
At the end of the lesson, you should be able to: References:
1. Define hyperbolic functions and illustrate their 1. The Calculus 7 by Louis Leithold
properties 2. Differential and Integral Calculus
1. 2. Enumerate the hyperbolic differentiation by Clyde E. Love and Earl Rainville
formulas and differentiate hyperbolic functions. 3.Calculus 7th Edition by James Stewart

Start your day with positive attitude and plan to


make the best . of it. Smile to the world and the
world will smile back.

A. LESSON PREVIEW/REVIEW

Introduction

GOOD DAY!
The following topics shall be accomplished in this module:
1. Hyperbolic functions and their properties
2. Hyperbolic Differentiation Formulas
3. Differentiating Hyperbolic Functions

Activity 1: What I Know Chart, part 1

What I Know Questions: What I Learned (Activity 4)

1. Will you define hyperbolic


function and illustrate their
properties?

2. Can you enumerate the


Hyperbolic Differentiation
Formulas?

3. How do you differentiate


hyperbolic functions?

This document is the property of PHINMA EDUCATION


1
MAT 171: Differential Calculus
Student’s Activity Sheets #16
Name: ______________________________________________________ Class number: ________
Section: ____________ Schedule: _______________________________ Date: _______________

B.MAIN LESSON

Activity 2: Content Notes

Hyperbolic functions
The exponential functions 𝑒 𝑥 and 𝑒 −𝑥 occur frequently in applied mathematics, science and
engineering. Mathematicians try to form the combinations of them and give special names
called the “hyperbolic functions”

We define the hyperbolic functions as follows.


𝒆𝒙 − 𝒆−𝒙
1. 𝐬𝐢𝐧𝐡 𝒙 = 𝟐
𝒆𝒙 +𝒆−𝒙
2. 𝐜𝐨𝐬𝐡 𝒙 =
𝟐
𝐬𝐢𝐧𝐡 𝒙 𝒆𝒙 −𝒆−𝒙
3. 𝐭𝐚𝐧𝐡 𝒙 = =
𝐜𝐨𝐬𝐡 𝒙 𝒆𝒙 +𝒆−𝒙
𝐜𝐨𝐬𝐡 𝒙
4. 𝐜𝐨𝐭𝐡 𝒙 =
𝐬𝐢𝐧𝐡 𝒙
𝟏
5. 𝐬𝐞𝐜𝐡 𝒙 =
𝐜𝐨𝐬𝐡 𝒙
𝟏
6. 𝐜𝐬𝐜𝐡 𝒙 =
𝐬𝐢𝐧𝐡 𝒙

The notation 𝑠𝑖𝑛ℎ 𝑥 read “hyperbolic sine of x”, cosh 𝑥 as “the hyperbolic cosine of x”. The
others are read in the same manner. The hyperbolic cotangent, hyperbolic secant and
hyperbolic cosecant are defined as the reciprocals of the hyperbolic tangent, hyperbolic cosine
and hyperbolic sine, respectively.

Hyperbolic Identities
𝐜𝐨𝐬𝐡𝟐 𝒙 − 𝐬𝐢𝐧𝐡𝟐 𝒙 = 𝟏
𝐭𝐚𝐧𝐡𝟐 𝒙 + 𝐬𝐞𝐜𝐡𝟐 𝒙 = 𝟏
𝐜𝐨𝐭𝐡𝟐 𝒙 − 𝐜𝐬𝐜𝐡𝟐 𝒙 = 𝟏
𝒔𝒊𝒏𝒉 𝟐𝒙 = 𝟐 𝐬𝐢𝐧𝐡 𝒙 𝐜𝐨𝐬𝐡 𝒙
𝐜𝐨𝐬𝐡 𝟐𝒙 = 𝐜𝐨𝐬𝐡𝟐 𝒙 + 𝐬𝐢𝐧𝐡𝟐 𝒙
= 𝟏 + 𝟐 𝐬𝐢𝐧𝐡𝟐 𝒙
= 𝟐 𝐜𝐨𝐬𝐡𝟐 𝒙 − 𝟏

This document is the property of PHINMA EDUCATION


2
MAT 171: Differential Calculus
Student’s Activity Sheets #16
Name: ______________________________________________________ Class number: ________
Section: ____________ Schedule: _______________________________ Date: _______________

Derivative Formulas of Hyperbolic Functions

Let u be a differentiable functions of x.

𝒅 𝒅𝒖
H1. (𝐬𝐢𝐧𝐡 𝒖) = 𝐜𝐨𝐬𝐡 𝒖
𝒅𝒙 𝒅𝒙
𝒅 𝒅𝒖
H2. (𝐜𝐨𝐬𝐡 𝒖) = 𝐬𝐢𝐧𝐡 𝒖
𝒅𝒙 𝒅𝒙
𝒅 𝒅𝒖
H3. (𝐭𝐚𝐧𝐡 𝒖) = 𝐬𝐞𝐜𝐡𝟐 𝒖
𝒅𝒙 𝒅𝒙
𝒅 𝒅𝒖
H4. (𝐜𝐨𝐭𝐡 𝒖) = − 𝐜𝐬𝐜𝐡𝟐 𝒖
𝒅𝒙 𝒅𝒙
𝒅 𝒅𝒖
H5. (𝐬𝐞𝐜𝐡 𝒖) = − 𝐬𝐞𝐜𝐡 𝒖 𝐭𝐚𝐧𝐡 𝒖
𝒅𝒙 𝒅𝒙
𝒅 𝒅𝒖
H6. (𝐜𝐬𝐜𝐡 𝒖) = − 𝐜𝐬𝐜𝐡 𝒖 𝐜𝐨𝐭𝐡 𝒖
𝒅𝒙 𝒅𝒙

Example1. Find the derivative of the given function.

1.1 𝒚 = 𝐬𝐢𝐧𝐡(𝟓𝒙 + 𝟏)
𝑑 𝑑𝑢
From (sinh 𝑢) = cosh 𝑢
𝑑𝑥 𝑑𝑥

𝑑𝑦
𝑑𝑥
= [cosh(5𝑥 + 1)](5)
𝒅𝒚
𝒅𝒙
= 5 cosh (5x +1)

1.2 𝒚 = 𝐜𝐨𝐬𝐡(𝟓𝒙 + 𝟏)
𝑑 𝑑𝑢
From 𝑑𝑥 (cosh 𝑢) = sinh 𝑢 𝑑𝑥

𝑑𝑦
= [sinh(5𝑥 + 1)](5)
𝑑𝑥
𝒅𝒚
𝒅𝒙
= 𝟓 𝐬𝐢𝐧𝐡(𝟓𝒙 + 𝟏)

This document is the property of PHINMA EDUCATION


3
MAT 171: Differential Calculus
Student’s Activity Sheets #16
Name: ______________________________________________________ Class number: ________
Section: ____________ Schedule: _______________________________ Date: _______________

1.3 𝒚 = 𝐭𝐚𝐧𝐡(𝟓𝒙 + 𝟏)

𝑑 𝑑𝑢
From 𝑑𝑥
(tanh 𝑢) = sech2 𝑢 𝑑𝑥

𝑑𝑦
𝑑𝑥
= [sech2(5𝑥 + 1)](5)

𝒅𝒚
= 𝟓 𝐬𝐞𝐜𝐡𝟐 (𝟓𝒙 + 𝟏)
𝒅𝒙

1.4 𝒚 = 𝐜𝐨𝐭𝐡(𝟓𝒙 + 𝟏)

𝑑 𝑑𝑢
From 𝑑𝑥
(coth 𝑢) = − csch2 𝑢 𝑑𝑥

𝑑𝑦
𝑑𝑥
= −[csch2(5𝑥 + 1)](5)

𝒅𝒚
𝒅𝒙
= −𝟓[𝐜𝐬𝐜𝐡𝟐 (𝟓𝒙 + 𝟏)]

1.5 𝒚 = 𝐬𝐞𝐜𝐡(𝟓𝒙 + 𝟏)

𝑑 𝑑𝑢
From 𝑑𝑥
(sech 𝑢) = − sech 𝑢 tanh 𝑢 𝑑𝑥

𝑑𝑦
= −[sech(5𝑥 + 1) tanh(5𝑥 + 1)](5)
𝑑𝑥

𝒅𝒚
𝒅𝒙
= −𝟓 𝐬𝐞𝐜𝐡(𝟓𝒙 + 𝟏) 𝐭𝐚𝐧𝐡(𝟓𝒙 + 𝟏)

1.5 𝒚 = 𝐜𝐬𝐜𝐡(𝟓𝒙 + 𝟏)

𝑑 𝑑𝑢
From 𝑑𝑥
(csch 𝑢) = − csch 𝑢 coth 𝑢 𝑑𝑥

𝑑𝑦
= −[csch(5𝑥 + 1) coth(5𝑥 + 1)](5)
𝑑𝑥

𝒅𝒚
𝒅𝒙
= −𝟓 𝐜𝐬𝐜𝐡(𝟓𝒙 + 𝟏)𝐜𝐨𝐭𝐡(𝟓𝒙 + 𝟏)

This document is the property of PHINMA EDUCATION


4
MAT 171: Differential Calculus
Student’s Activity Sheets #16
Name: ______________________________________________________ Class number: ________
Section: ____________ Schedule: _______________________________ Date: _______________
𝒅𝒚
Example 2. Find 𝒅𝒙, If 𝒚 = 𝟑 𝐜𝐨𝐬𝐡𝟐 𝟒𝒙

Given: 𝑦 = 3 cosh2 4𝑥 Chain rule

𝑑𝑦 𝑑 𝑑𝑢
𝑑𝑥
= 6(cosh 4𝑥)(sinh 4𝑥)(4) 𝑑𝑥
cosh 𝑢 = sinh 𝑢 𝑑𝑥

𝑑𝑦
𝑑𝑥
= 24 cosh 4𝑥𝑠𝑖𝑛ℎ 4𝑥

𝑑𝑦
= 12(2 cosh 4𝑥 sinh 4𝑥)
𝑑𝑥

𝒅𝒚
= 𝟏𝟐(𝐬𝐢𝐧𝐡 𝟖𝒙)
𝒅𝒙

Example 3. Find the derivative of the function 𝒚 = 𝐬𝐢𝐧𝐡𝟓 𝒙 𝐜𝐨𝐬𝐡𝟓 𝒙

Given: 𝑦 = sinh5 𝑥 cosh5 𝑥 From the identities sinh 2x = 2 sinh x cosh x

sinh 2𝑥 5 sinh 2𝑥
𝑦=( 2
) 2
= sinh 𝑥 cosh 𝑥

1
𝑦= sinh5 2𝑥
32

𝑑𝑦 5
𝑑𝑥
= 32 (sinh4 2𝑥)(cosh 2𝑥)(2)

𝒅𝒚 𝟓
𝒅𝒙
= 𝟏𝟔 𝐬𝐢𝐧𝐡𝟒 𝟐𝒙 𝐜𝐨𝐬𝐡 𝟐𝒙

𝒅𝒚
Example 4. Find and simplify, if 𝒚 = 𝐚𝐫𝐜𝐬𝐢𝐧(𝐭𝐚𝐧𝐡 𝟒𝒙)
𝒅𝒙
𝑑𝑢
𝑑 𝑑𝑥
Given: 𝑦 = arcsin(tanh 4𝑥) Recall 𝑎𝑟𝑐𝑠𝑖𝑛𝑢 =
𝑑𝑥 √1−𝑢2

𝑑𝑢
Let 𝑢 = tanh 4𝑥 , 𝑑𝑥
= (sech2 4𝑥)(4) = 4 sech2 4𝑥

𝑑𝑦 4 sech2 4𝑥
= From identity tanh2 𝑥 + sech2 𝑥 = 1
𝑑𝑥 √1−tanh2 4𝑥

𝑑𝑦 4 sech2 4𝑥
= sech2 𝑥 = 1 − tanh2 𝑥
𝑑𝑥 √sech2 𝑥

4 sech2 4𝑥
= sech 4𝑥

= 𝟒 𝐬𝐞𝐜𝐡 𝟒𝒙

This document is the property of PHINMA EDUCATION


5
MAT 171: Differential Calculus
Student’s Activity Sheets #16
Name: ______________________________________________________ Class number: ________
Section: ____________ Schedule: _______________________________ Date: _______________

Activity 3: Skill-building Activities (with answer key)

1. Find the derivative of the function, 𝑦 = arctan(sinh 𝑥)

𝑑𝑦 sinh 𝑥
2.Find 𝑑𝑥, if 𝑦 = 1+cosh 𝑥

𝑑𝑦
3.Find 𝑑𝑥, if 𝑦 = 𝑥 sinh 𝑥 − cosh 𝑥

Activity 4: What I Know Chart, part 2


What I Learned
1. ______________________________________________________________________________
______________________________________________________________________________
2. ______________________________________________________________________________
______________________________________________________________________________
3. ______________________________________________________________________________
______________________________________________________________________________

Activity 5: Check for Understanding


Find the Derivative!
1.Find the derivative of the function
𝑦 = 𝑐𝑜𝑠ℎ4 𝑥 − sinh4 𝑥

2.Find the derivative of 𝑦 = ln cosh 𝑥.

C. LESSON WRAP-UP
Activity 6: Thinking about Learning

Work Tracker
You are done with this session! Let’s track your progress.
Shade the session number you just completed.

Think about your Learning

1. What motivated you to finish the lesson today?


________________________________________________________________________________________
________________________________________________________________________________________

2. What could you have done better to improve your learning today?
________________________________________________________________________________________
________________________________________________________________________________________

This document is the property of PHINMA EDUCATION


6
MAT 171: Differential Calculus
Student’s Activity Sheets #16
Name: ______________________________________________________ Class number: ________
Section: ____________ Schedule: _______________________________ Date: _______________

FAQs

1.What are hyperbolic functions?

In mathematics, hyperbolic functions are analogous to the ordinary


trigonometric functions defined for the hyperbola rather than on the circle: just as the points
(cos t, sin t) form a circle with a unit radius, the points (cosh t, sinh t) form the right half of the
equilateral hyperbola.

2. What is a hyperbolic curve?

A hyperbola is an open curve with two branches, the intersection of a plane with both
halves of a double cone. The plane does not have to be parallel to the axis of the cone;
the hyperbola will be symmetrical in any case.

KEY TO CORRECTIONS

Activity 3

𝒅𝒚
1. = sech x
𝒅𝒙
𝒅𝒚 𝟏
2. =
𝒅𝒙 𝟏+𝐜𝐨𝐬𝐡 𝒙
𝑑𝑦
3. 𝑑𝑥
= 𝒙 𝐜𝐨𝐬𝐡 𝒙

Activity 5

𝒅𝒚
1. 𝒅𝒙
= 𝟐 𝐬𝐢𝐧𝐡 𝟐𝒙

𝒅𝒚
2. = 𝐭𝐚𝐧𝐡 𝒙
𝒅𝒙

This document is the property of PHINMA EDUCATION


7
MAT 171: Differential Calculus
Student’s Activity Sheets #17
Name: ______________________________________________________ Class number: ________
Section: ____________ Schedule: _______________________________ Date: _______________

Lesson Title: Parametric Equations, Curvature, Materials: Ballpoint, notebook, calculator


& Partial Derivative Textbook: Calculus 11th Edition by Ron Larson
References:
Lesson Objectives: 1.The Calculus 7 by Louis Leithold
At the end of the lesson, you should be able to: 2. Calculus 7th Edition by James Stewart
1. Define Parametric Equation 2. Differential and Integral Calculus
2. Find the derivative of parametric equations. by Clyde E. Love and Earl Rainville
3. Define, illustrate and derive the arclength,
and curvature of a curve
4. Compute the values of curvature and radius
of curvature of a curve at any given point
5. Differentiate derivative and partial derivative

“The road to success isn’t straight. There are


curves and loops. You will have flat tires, but if
you have a spare called strength and a driver
called God, you will make it.”
A. LESSON PREVIEW/REVIEW

Introduction
GOOD DAY! The following topics shall be
accomplished in this module:
1. Definition of parametric equations
2. Derivative of parametric equations
3. Arc length and curvature of a curve
4. Values of curvature and the radius of curvature of a curve at any given point

Activity 1: What I Know Chart, part 1

What I Know Questions: What I Learned (Activity 4)


1. Can you define a parametric
equation?
2. How do you find the derivative
of parametric equations?
3. Can you define and illustrate an
arc length and curvature of a
curve?
4. How do you compute the values
of curvature and the radius of
curvature of a curve at any given
point?

This document is the property of PHINMA EDUCATION


1
MAT 171: Differential Calculus
Student’s Activity Sheets #17
Name: ______________________________________________________ Class number: ________
Section: ____________ Schedule: _______________________________ Date: _______________

B.MAIN LESSON

Activity 2: Content Notes

PARAMETRIC EQUATIONS
In Analytic Geometry, we have learned that a curve may also be described analytically by a pair
of equations of the form.

𝑥 = 𝑔(𝑡) or 𝑦 = ℎ(𝑡).

These equations are called parametric equations of the curve and the third variable t is called
a parameter.

For example, the equation of a circle is 𝑥 2 + 𝑦 2 = 𝑟 2 ; the origin is the center and of radius r.
The parametric equations of the circle are,
𝒙 = 𝒓 𝐜𝐨𝐬 𝜭
𝒚 = 𝒓 𝐬𝐢𝐧 𝜭
where the parameter ϴ is the angle between the x-axis and the radius to the point ( x, y ).

To verify the parametric equations of the circle, substitute 𝑟 cos 𝛳 and 𝑟 sin 𝛳 for x and y, that is,

𝑥2 + 𝑦2 = 𝑟2
(𝑟 cos 𝛳)2 + (𝑟 sin 𝛳)2 = 𝑟 2
𝑟 2 cos2 𝛳 + 𝑟 2 sin2 𝛳 = 𝑟 2
cos 2 𝛳 + sin2 𝛳 = 1

Derivatives in Parametric Form

Let 𝑦 = 𝑓(𝑥) be a function whose parametric representation is given in the form


𝑥 = 𝑔(𝑡) , 𝑦 = ℎ(𝑡)
We recall that,
𝑑𝑥
= rate of change of x with respect to t
𝑑𝑡
𝑑𝑦
𝑑𝑡
= rate of change of y with respect to t.

Then evidently the rate of change of y with respect to x of a function defined by 𝑥 = 𝑔(𝑡),
𝑦 = ℎ(𝑡) will be given by,
𝒅𝒚
𝒅𝒚 𝒅𝒕
= 𝒅𝒙
𝒅𝒙
𝒅𝒕

This document is the property of PHINMA EDUCATION


2
MAT 171: Differential Calculus
Student’s Activity Sheets #17
Name: ______________________________________________________ Class number: ________
Section: ____________ Schedule: _______________________________ Date: _______________

Next, we consider the problems of finding the second derivative of a function defined by the
parametric equations above. Recall that the second derivative is defined as,

𝒅𝟐 𝒚 𝒅 𝒅𝒚
𝒅𝒙𝟐
= 𝒅𝒙
(𝒅𝒙)

By the chain rule, we may write the equation above in the form,

𝒅𝟐 𝒚 𝒅 𝒅𝒚 𝒅𝒕
= ( ) ∙ 𝒅𝒙
𝒅𝒙𝟐 𝒅𝒕 𝒅𝒙

𝒅𝒚 𝒅𝟐 𝒚
Example 1. If 𝒙 = 𝒕𝟑 − 𝟏 , 𝒚 = 𝒕𝟐 + 𝒕, Find 𝒅𝒙 𝒂𝒏𝒅 𝒅𝒙𝟐
Solution:
Since 𝑥 = 𝑡 3 − 1, 𝑦 = 𝑡2 + 𝑡
Differentiate the given parametric equations.
𝑑𝑥 𝑑𝑦
𝑑𝑡
= 3𝑡 2 𝑑𝑡
= 2𝑡 + 1
𝑑𝑦
𝑑𝑦 𝑑𝑡
From 𝑑𝑥 = 𝑑𝑥
𝑑𝑡

𝒅𝒚 𝟐𝒕+𝟏
Thus, the first derivative is, =
𝒅𝒙 𝟑𝒕𝟐

𝑑𝑦 2𝑡+1
To find the second derivative of 𝑑𝑥
= 3𝑡 2

𝑑2 𝑦 𝑑 𝑑𝑦 𝑑𝑡
We then apply 𝑑𝑥 2
= [𝑑𝑡 (𝑑𝑥 )] (𝑑𝑥)
𝑑2 𝑦 𝑑 2𝑡+1 𝑑𝑡 𝑑𝑡 1 1
𝑑𝑥 2
= 𝑑𝑡 ( 3𝑡 2 ) ∙ 𝑑𝑥 But 𝑑𝑥 = 𝑑𝑥 = 3𝑡 2
𝑑𝑡

(3𝑡 2 )(2)−(2𝑡+1)(6𝑡) 1
= ∙
9𝑡 4 3𝑡 2
−6𝑡 2 −6𝑡 1
= ∙ 2
9𝑡 4 3𝑡
−6𝑡(𝑡+1)
= 27𝑡 6
𝑑2 𝑦 −2(𝑡+1)
=
𝑑𝑥 2 9𝑡 5

Therefore the second derivative of y with respect to x is,


𝒅𝟐 𝒚 −𝟐(𝒕+𝟏)
𝒅𝒙𝟐
= 𝟗𝒕𝟓

This document is the property of PHINMA EDUCATION


3
MAT 171: Differential Calculus
Student’s Activity Sheets #17
Name: ______________________________________________________ Class number: ________
Section: ____________ Schedule: _______________________________ Date: _______________

𝒅𝒚 𝒅𝟐 𝒚
Example2. From the parametric equations: 𝒙 = 𝟐 𝐬𝐢𝐧 𝜭, and 𝒚 = 𝟏 − 𝟒 𝐜𝐨𝐬 𝜭, Find 𝒅𝒙 and 𝒅𝒙𝟐

Given: 𝑥 = 2 sin 𝛳, 𝑦 = 1 − 4 cos 𝛳


𝑑𝑥 𝑑𝑦
𝑑𝛳
= 2 cos 𝛳, 𝑑𝛳
= 4 sin 𝛳

𝑑𝑦
𝑑𝑦 𝑑𝛳
Then 𝑑𝑥
= 𝑑𝑥
𝑑𝛳

𝒅𝒚 4 sin 𝛳
= 2 cos 𝛳 = 𝟐 𝐭𝐚𝐧 𝜭
𝒅𝒙
Then solve for the second derivative

𝑑2 𝑦 𝑑𝛳 𝑑𝜃 1
= 2 sec 2 𝛳 ∙ But =
𝑑𝑥 2 𝑑𝑥 𝑑𝑥 2 cos 𝛳
1 1
= 2 sec 2 𝛳 ∙ ( ) sec 𝛳 =
2 cos 𝛳 cos 𝛳

= (sec 2 𝛳)(sec 𝛳)
𝒅𝟐 𝒚
𝒅𝒙𝟐
= 𝐬𝐞𝐜 𝟑 𝜭

CURVATURE
Differential of arc length
Let 𝑦 = 𝑓(𝑥) be a continuous function. Let P(x, y) and Q(𝑥 + ∆𝑥, 𝑦 + ∆𝑦) be on the curve of f(x).
Denote ∆𝑠 be the arc length from P to Q. The rate of change of the arc s from P to Q per unit
change in y are given respectively by,

𝒅𝒔 ∆𝒔 𝒅𝒚 𝟐
= 𝐥𝐢𝐦 = ±√𝟏 + ( ) ,
𝒅𝒙 ∆𝒙→𝟎 ∆𝒙 𝒅𝒙

𝒚 = 𝒇(𝒙)
𝒅𝒔 ∆𝒔 𝒅𝒙 𝟐
= 𝐥𝐢𝐦 = ±√𝟏 + ( )
𝒅𝒚 ∆𝒚→𝟎 ∆𝒚 𝒅𝒚

When the curve is given by parametric


equations 𝑥 = 𝑓(𝑢) and 𝑦 = 𝑔(𝑢), the
rate of change of s with respect to u is
given by,

𝒅𝒔 𝒅𝒙 𝒅𝒚𝟐 𝟐
= √( ) + ( ) .
𝒅𝒖 𝒅𝒖 𝒅𝒖

This document is the property of PHINMA EDUCATION


4
MAT 171: Differential Calculus
Student’s Activity Sheets #17
Name: ______________________________________________________ Class number: ________
Section: ____________ Schedule: _______________________________ Date: _______________

Here the plus or minus sign is to be taken accordingly as s increases or decreases as u


increases.

If the equation of the curve is given in the polar form 𝑟 = 𝑓(𝛳), then the differential of arc length
is given by,

𝒅𝒓 𝟐
𝒅𝒔 = √𝒓𝟐 + ( ) 𝒅𝜭
𝒅𝜭

this can be obtained by use of the familiar relations between rectangular and polar coordinates,
that is,

𝒙 = 𝒓 𝐜𝐨𝐬 𝜭 and 𝒚 = 𝒓 𝐬𝐢𝐧 𝜭.

Definition of curvature

We have in our previous discussion that the concept of the derivative is related to the tangent to
a curve. Another concept of geometric interest is the curvature when we consider any smooth
curve.

Curvature measures the rate at which the tangent lines turns per unit distance move along the
curve; or it measures the rate of change of direction of the curve.

Let P and Q be two points on a curve, separated by an arc length as ∆𝑠 (see figure). Then the
∆𝛼
average curvature of the arc from P to Q is expressed by the fraction ∆𝑠 . Where, ∆𝛼 is the
angle turned through by the tangent line moving from P to Q.

This document is the property of PHINMA EDUCATION


5
MAT 171: Differential Calculus
Student’s Activity Sheets #17
Name: ______________________________________________________ Class number: ________
Section: ____________ Schedule: _______________________________ Date: _______________
∆𝛼 𝑑𝛼 𝑑𝛼
The curvature k at point P is defined as 𝐾 = lim = = curvature at P. To calculate , we
∆𝑠→0 ∆𝑠 𝑑𝑠 𝑑𝑠

|𝑦 ′′ |
consider the slope of the line at P which is the angle of inclination 𝛼, that is 𝑑𝛼 = 𝑑𝑥 .
1+(𝑦 ′ )2

But 𝑑𝑠 = √1 + (𝑦 ′ )2 𝑑𝑥. Hence, by division,

the curvature at a point P( x, y ) of the curve 𝑦 = 𝑓(𝑥) is

∆𝛼 𝑑𝛼 |𝑦 ′′ |𝑑𝑥 1
𝒌 = lim = = ∙
∆𝑠→0 ∆𝑠 𝑑𝑠 1+(𝑦 ′)2 √1+(𝑦 ′)2 𝑑𝑥

𝒅𝟐 𝒚
𝒚′′ 𝒅𝒙𝟐
𝒌= 𝟑 or 𝒌= 𝟑
[𝟏+(𝒚′ )𝟐 ]𝟐 𝒅𝒚 𝟐 𝟐
[𝟏+( ) ]
𝒅𝒙

Radius of Curvature
The reciprocal of curvature is called the radius of curvature and is denoted by R. That is,

𝟏
𝐑=
𝐊
𝟑
𝒅𝒚 𝟐 𝟐
[𝟏+( ) ]
𝒅𝒙
𝑹= 𝟐
𝒅 𝒚
𝒅𝒙𝟐
𝟑
[𝟏+(𝒚′ )𝟐 ]𝟐
or 𝑹=
𝒚′′

𝑑𝑦 𝑑2 𝑦
Where 𝑦′ = 𝑑𝑥 and 𝑦′′ = 𝑑𝑥 2 .

When the equation of a curve is given parametrically in the form 𝑥 = 𝑔(𝑡), 𝑦 = ℎ(𝑡)
The radius of curvature can be shown to be,
𝟑
𝟐 𝟐 𝟐

[(𝒈′ ) +(𝒉 ) ]

𝑹= ′′
𝒈′𝒉 −𝒈′′𝒚′

𝑑𝑥 𝑑2 𝑥 𝑑𝑦 𝑑2 𝑦
Where 𝑔′ = 𝑑𝑡
, 𝑔′′ = 𝑑𝑡 2
, ℎ′ = 𝑑𝑡
, ℎ′′ = 𝑑𝑡 2

This document is the property of PHINMA EDUCATION


6
MAT 171: Differential Calculus
Student’s Activity Sheets #17
Name: ______________________________________________________ Class number: ________
Section: ____________ Schedule: _______________________________ Date: _______________

Example 1. Find the curvature of the curve 𝒚 = 𝒙𝟑 at ( 1, 1).

Solution:
Given: 𝑦 = 𝑥 3
𝑑𝑦 𝑑𝑦
𝑑𝑥
= 3𝑥 2 Solve the first derivative, at (1, 1), 𝑑𝑥
= 3(1)2 = 3
𝑑2 𝑦 𝑑2 𝑦
𝑑𝑥 2
= 6𝑥 Solve the second derivative, at (1, 1) 𝑑𝑥 2
= 6(1) = 6

From the formula of curvature:


𝒅𝟐 𝒚
𝒅𝒙𝟐 𝑑𝑦 𝑑2 𝑦
𝒌= 𝟑 Substitute =3; =6
𝑑𝑥 𝑑𝑥 2
𝒅𝒚 𝟐 𝟐
[𝟏+( ) ]
𝒅𝒙

6
= 3
[1+(3)2 ]2
6
= 3
(10)2

𝒌 = 𝟎. 𝟏𝟖𝟗𝟕

Example 2. Find the radius of curvature of the curve 𝒚 = 𝟐𝒙𝟑 + 𝟑𝒙𝟐 at (𝟏, 𝟓).

Solution:

Given: 𝑦 = 2𝑥 3 + 3𝑥 2

𝑦′ = 6𝑥 2 + 6𝑥 At (1,5)

𝑦′ = 6(1) + 6(1) = 12

𝑦′′ = 12𝑥 + 6 at (1,5)

𝑦′′ = 12(1) + 6 = 18
3
2 2
[1+(𝑦 ′ ) ]
𝑅= 𝑦′′

3
[1+(12)2 ]2
= 18
𝑹 = 𝟗𝟕

This document is the property of PHINMA EDUCATION


7
MAT 171: Differential Calculus
Student’s Activity Sheets #17
Name: ______________________________________________________ Class number: ________
Section: ____________ Schedule: _______________________________ Date: _______________

Example 2. Find the curvature of the parabola 𝒚𝟐 = 𝟏𝟐𝒙 at ( 3, 6 ).

Solution
Given: 𝑦 2 = 12𝑥
Find the first derivative of y with respect to x.
𝑑𝑦
2𝑦 𝑑𝑥 = 12
𝑑𝑦 12 6
= =
𝑑𝑥 2𝑦 𝑦

𝑑𝑦
Solve for at (3, 6 )
𝑑𝑥
𝑑𝑦 6
Thus = =1
𝑑𝑥 6

𝑑𝑦 6
From = Find the second derivative of y with respect to x,
𝑑𝑥 𝑦

𝑑2 𝑦 6𝑦′
𝑑𝑥 2
= − 𝑦2

𝑑2 𝑦
Solve 𝑑𝑥 2
at ( 3, 6)

𝑑2 𝑦 −6(1) 1
= =−
𝑑𝑥 2 (6)2 6
𝑑2 𝑦
𝑑𝑥2
From the formula 𝒌 = 3
𝑑𝑦 2 2
[1+( ) ]
𝑑𝑥

1
But 𝑦′ = 1, 𝑦′′ = −
6
1

6
𝒌= 3
[1+(1)2 ]2

𝒌 = −𝟎. 𝟎𝟓𝟖𝟗

This document is the property of PHINMA EDUCATION


8
MAT 171: Differential Calculus
Student’s Activity Sheets #17
Name: ______________________________________________________ Class number: ________
Section: ____________ Schedule: _______________________________ Date: _______________

PARTIAL DERIVATIVES
In the preceding lessons, we have learned and used differentiation of functions where we had
one dependent variable and one independent variable. These functions are called functions of
one variable. The functional notations regarding these types of functions were as follows:

𝒚 = 𝒇(𝒙) for explicit functions,


𝒇(𝒙, 𝒚) = 𝟎 for implicit functions.

In this lesson, we shall study differentiation of functions of several variables. Examples of these
functions are the following familiar formulas:

𝟏
𝑽 = 𝝅𝒓𝟐 𝒉 (1) 𝑨 = 𝟐 𝒂𝒃 𝐬𝐢𝐧 𝜭 (2)

Formula (1) expresses the volume V of a right circular cylinder in terms of the base radius r and
the altitude or height h. That is, V is a function of two variables, r and h. Formula (2) shows that
the area A of an oblique triangle is a function of three variables, namely a, b and ϴ.

Partial Derivative

Let 𝑧 = 𝑓(𝑥, 𝑦) be a function of two independent variables x and y. If y is held constant, then 𝑧
becomes temporarily a function of the single variable x. From this point of view, we can
complete the derivative of 𝑧 with respect to 𝑥 by employing the rules for ordinary differentiation
of functions with single variable. The derivative found in this manner is called the partial
derivative of 𝑧 with respect to x and the process involved is called partial differentiation. The
𝜕𝑧 𝜕𝑓
derivative of 𝑧 with respect to 𝑥 is denoted by any of the following symbols ,
𝜕𝑥 𝜕𝑥
, 𝑧-
𝑥, 𝑓𝑥 (𝑥, 𝑦), 𝑓𝑥 .

Similarly, if x is held constant, then z becomes temporarily a function of y. As a result, we can


compute the partial derivative of z with respect to y and this derivative may be denoted by any
of the following symbols.

𝜕𝑧 𝜕𝑓
,
𝜕𝑦 𝜕𝑦
, 𝑧𝑦 , 𝑓𝑦 (𝑥, 𝑦), 𝑓𝑦
𝜕𝑧 𝜕𝑧
It should be noted that the symbol 𝜕𝑥 ( or 𝜕𝑦
) cannot be thought of as a fraction since neither of
𝝏
the symbols 𝜕𝑧 𝑎𝑛𝑑 𝜕𝑥 (or 𝜕𝑧 𝑎𝑛𝑑 𝜕𝑦 ) has a separate meaning. The symbol 𝝏𝒙
alone means to
𝝏
differentiate partially with respect to x whatever follows it. The symbol is interpreted in like
𝝏𝒚
manner.

This document is the property of PHINMA EDUCATION


9
MAT 171: Differential Calculus
Student’s Activity Sheets #17
Name: ______________________________________________________ Class number: ________
Section: ____________ Schedule: _______________________________ Date: _______________

The Definition of Partial Derivative are as follows:

If the function of two independent variables 𝑧 = 𝑓(𝑥, 𝑦), we define the first partial derivative as,

𝝏𝒛 𝒇(𝒙+∆𝒙,𝒚 )−𝒇(𝒙,𝒚) 𝝏𝒛 𝒇(𝒙,𝒚+∆ 𝒚 )−𝒇(𝒙,𝒚)


(1) = 𝐥𝐢𝐦 (2) 𝝏𝒚 = 𝐥𝐢𝐦
𝝏𝒙 ∆𝒙→𝟎 ∆𝒙 ∆𝒚→𝟎 ∆𝒚
This is the partial derivative of z with respect to
This is the partial derivative of z with respect to
the independent variable x where y is kept fixed
or a constant. y where x is kept fixed or as constant

The Process of Solving Partial Derivative

To solve for the partial derivative with respect to any independent variable, we differentiate the given
functional equation using the same set of differentiation formulas that we have used before. The only
difference is that there are more than one independent variable to be found. However, the particular
derivative with respect to a particular independent variable will be obtained easily by treating the other
independent variable as plain constant.

One will note therefore, that differentiating a function of two or more independent variables is just like
differentiating functions of only one variable.
𝑑
Furthermore, the operator symbol for ordinary differentiation will be replaced by the new
𝑑𝑥
𝝏
symbol using independent variable x.
𝝏𝒙

𝝏𝒛 𝝏𝒛
Example 1. Find (a) 𝝏𝒙 and (b) 𝝏𝒚 for 𝒛 = 𝟑𝒙𝟐 − 𝟒𝒙𝒚 + 𝒚𝟐
𝜕𝑧
a) To find 𝜕𝑥 for 𝑧 = 3𝑥 2 − 4𝑥𝑦 + 𝑦 2 , consider y to be held constant and differentiate with
respect to x.
𝜕𝑧 𝜕
𝜕𝑥
= 𝜕𝑥 (3𝑥 2 − 4𝑥𝑦 + 𝑦 2 )

= 6𝑥 − 4𝑦 + 0
𝝏𝒛
Therefore: 𝝏𝒙 = 𝟔𝒙 − 𝟒𝒚 is the partial derivative with respect to x
𝜕𝑧
b) To find 𝜕𝑦
for 𝑧 = 3𝑥 2 − 4𝑥𝑦 + 𝑦 2 , consider x to be held constant and differentiate with
respect to y.

𝜕𝑧 𝜕
𝜕𝑦
= 𝜕𝑦
(3𝑥 2 − 4𝑥𝑦 + 𝑦 2 )
𝜕𝑧
𝜕𝑦
= 0 − 4𝑥 + 2𝑦
𝝏𝒛
Therefore: 𝝏𝒚 = −𝟒𝒙 + 𝟐𝒚 is the partial derivative with respect to y

This document is the property of PHINMA EDUCATION


10
MAT 171: Differential Calculus
Student’s Activity Sheets #17
Name: ______________________________________________________ Class number: ________
Section: ____________ Schedule: _______________________________ Date: _______________

Example 2. Find 𝒇𝒙 , 𝒇𝒚 , 𝒇𝒛 Given: 𝒇(𝒙, 𝒚, 𝒛) = 𝟑𝒙𝟐 𝒚 + 𝒚𝒛𝟐 + 𝟐𝒛𝟑


Solution:
𝜕
a) 𝑓𝑥 = 𝜕𝑥 (3𝑥 2 𝑦 + 𝑦𝑧 2 + 2𝑧 3 )

𝑓𝑥 = 6𝑥𝑦 + 0 + 0 , Consider y and z to be constant, differentiate with respect to x


Therefore: 𝒇𝒙 = 𝟔𝒙𝒚 The partial derivative with respect to x

𝜕
b) 𝑓𝑦 = (3𝑥 2 𝑦 + 𝑦𝑧 2 + 2𝑧 3 )
𝜕𝑦

𝑓𝑦 = 3𝑥 2 + 𝑧 2 + 0 , Consider x and z to be constant, differentiate with respect to y


𝒇𝒚 = 𝟑𝒙𝟐 + 𝒛𝟐 The partial derivative with respect to y

𝜕
c) 𝑓𝑧 = (3𝑥 2 𝑦 + 𝑦𝑧 2 + 2𝑧 3 )
𝜕𝑥

𝑓𝑧 = 0 + 2𝑦𝑧 + 6𝑧 2 , Consider x and y to be constant, differentiate with respect to z


𝒇𝒛 = 𝟐𝒚𝒛 + 𝟔𝒛𝟐 The partial derivative with respect to z.

Higher Partial Derivative Symbols

Just like in functions of one independent variable, we can also find higher derivative in functions of two
or more independent variables. But before we go to actual solving for higher partial derivative, we have
to be familiar with higher derivatives symbols. For example, if we are given the function 𝑦 = 𝑓(𝑥, 𝑦)

𝝏𝒇 𝝏𝒇
First derivative symbols: 𝒇𝒙 , 𝒇 𝒚 , 𝝏𝒙
, 𝝏𝒚
, etc.

𝝏𝒇𝟐 𝝏𝒇𝟐
Second derivative symbols: 𝒇𝒙𝒙 , 𝒇𝒚𝒚 , 𝒇𝒙𝒚 etc. 𝝏𝒙𝟐
, 𝝏𝒚𝟐
etc.

𝝏𝒇
Example 1. If 𝒛 = 𝒙𝒚𝟐 + 𝒙𝟑 𝒚, find 𝝏𝒙
, 𝒛𝒙 , 𝒛𝒙𝒙 , 𝒛𝒙𝒙𝒚

Given: 𝑧 = 𝑥𝑦 2 + 𝑥 3 𝑦

Solution:
𝑧𝑥 = 𝑦 2 + 3𝑥 2 𝑦 Consider y to be constant and differentiate with respect to x
𝑧𝑥𝑥 = 6𝑥𝑦 Consider x to be constant and differentiate with respect to y
𝒛𝒙𝒙𝒚 = 𝟔𝒙

This document is the property of PHINMA EDUCATION


2
MAT 171: Differential Calculus
Student’s Activity Sheets #17
Name: ______________________________________________________ Class number: ________
Section: ____________ Schedule: _______________________________ Date: _______________

Activity 3: Skill-building Activities (with answer key)


Differentiate!
𝑑𝑦 𝑑2 𝑦
1. Find 𝑑𝑥
and 𝑑𝑥 2
and simplify whenever possible.

Given parametric equations:


x = 𝑡3 + 1 , 𝑦 = 𝑡2 + 1

2. If 𝑥 = 𝛳 − sin 𝛳 and 𝑦 = 1 − cos 𝛳,


𝑑𝑦 𝑑2 𝑦
Find 𝑑𝑥 and 𝑑𝑥 2

3. Find the radius of the curvature of a parabola


𝑦 2 − 4𝑥 = 0 at (4, 4)

4. If 𝑧 = 𝑥 2 𝑦 + 4𝑥 + 3𝑦,
𝜕𝑧 𝜕𝑧
Find and
𝜕𝑥 𝜕𝑦

Activity 4: What I Know Chart, part 2

What I Learned
1. ________________________________________________________________________________
___________________________________________________________________________________
2. ________________________________________________________________________________
___________________________________________________________________________________
3. ________________________________________________________________________________
___________________________________________________________________________________

Activity 5: Check for Understanding

𝑑𝑦 𝑑2 𝑦
1. Find 𝑑𝑥 and 𝑑𝑥 2
and simplify whenever possible.

. Given: 𝑥 = 𝑡 3 + 1, 𝑦 = 4𝑡 2 − 4𝑡

2. A parabola has an equation of 𝑦 2 = 4𝑥


a) What is the slope of the curve at ( 1, 2)
b) What is the radius of curvature at ( 1, 2 )

3. If 𝑧 = 𝑥 sin 𝑦 + 𝑦 sin 𝑥
𝜕𝑧 𝜕𝑧
Find 𝜕𝑥 and 𝜕𝑦

This document is the property of PHINMA EDUCATION


3
MAT 171: Differential Calculus
Student’s Activity Sheets #17
Name: ______________________________________________________ Class number: ________
Section: ____________ Schedule: _______________________________ Date: _______________

C. LESSON WRAP-UP

Activity 6: Thinking about Learning

Work Tracker
You are done with this session! Let’s track your progress.
Shade the session number you just completed.

Think about your Learning

1. What motivated you to finish the lesson today?


________________________________________________________________________________________
________________________________________________________________________________________

2. What could you have done better to improve your learning today?
________________________________________________________________________________________
________________________________________________________________________________________

FAQs
1. Where do we use parametric equations?

Parametric equations are commonly used to express the coordinates of the points that make up
a geometric object such as a curve or surface, in which case the equations are collectively
called a parametric representation or parameterization (alternatively spelled as parametrisation)
of the object.

2. Why do we need parametric equations?

One of the advantages of parametric equations is that they can be used to graph curves
that are not functions, like the unit circle. Another advantage is that the parameter can be used
to represent something useful and therefore provide us with additional information about the
graph.

3. What is the difference between radius and radius of curvature?

The difference between radius and radius of curvature is that the radius refers to the
distance between the center of a circle or any other point on the circumference of the circle and
surface of the sphere. While on the other hand, the radius of curvature is the radius of the circle
that touches the curve at a given point.

This document is the property of PHINMA EDUCATION


4
MAT 171: Differential Calculus
Student’s Activity Sheets #17
Name: ______________________________________________________ Class number: ________
Section: ____________ Schedule: _______________________________ Date: _______________

4. How do you define curvature?

Curvature is defined as 1 : the act of curving : the state of being curved. 2 : a measure or
amount of curving specifically : the rate of change of the angle through which the tangent to a
curve turns in moving along the curve and which for a circle is equal to the reciprocal of the
radius.

5. What is the difference between derivative and partial derivative?


A derivative is applied to functions that have only one independent variable. A partial
derivative is applied to functions that have more than one independent variable. The idea is that
you find the derivative with respect to one of the variables and assume that all of the other
variables are actually constants.

6. Why do we use partial derivatives?

Partial differentiation is used to differentiate mathematical functions having more than one
variable in them. In ordinary differentiation, we find derivative with respect to one variable only.
A function contains only one variable. So partial differentiation is more general than ordinary
differentiation.

KEY TO CORRECTIONS

Activity 3.
𝒅𝒚 𝟐 𝒅𝟐 𝒚 −𝟐
1. 𝒅𝒙
= 𝟑𝒕 , 𝒅𝒙𝟐 = 𝟗𝒕𝟒
𝒅𝒚 𝐬𝐢𝐧 𝜭 𝒅𝟐 𝒚 𝟏
2. = 𝟏−𝐜𝐨𝐬 𝜭 , = − (𝟏−𝐜𝐨𝐬 𝜭 )𝟐
𝒅𝒙 𝒅𝒙𝟐
3. 𝑹 = −𝟐𝟐. 𝟑𝟔
𝝏𝒛 𝝏𝒛
4. 𝝏𝒙 = 𝟐𝒙𝒚 + 𝟒 , 𝝏𝒚
= 𝒙𝟐 + 𝟑

Activity 5.
𝒅𝟐 𝒚 −𝟖(𝒕−𝟏)
1. =
𝒅𝒙𝟐 𝟗𝒕𝟓

2. 𝒚′ = 𝟏 , 𝑹 = −𝟓. 𝟔𝟔
𝝏𝒛
3. 𝝏𝒙
= 𝐬𝐢𝐧 𝒚 + 𝒚 𝐜𝐨𝐬 𝒙
𝝏𝒛
𝝏𝒚
= 𝒙 𝐜𝐨𝐬 𝒚 + 𝐬𝐢𝐧 𝒙

This document is the property of PHINMA EDUCATION


5

You might also like